Sunteți pe pagina 1din 104

1|Page Philippians 4:13 jpmrnmd

GYNECOLOGY – Mastery Review


“Master your Ultra Instinct”
Ultimate Reviewer ~800 items for Major Exams | Revalida | Board Exams
Take Note: 8. Curdy discharge
“This Mastery Review is intended for
remembering important concepts in  Candidiasis
GYNECOLOGY, it might not be the same
examination but it will surely give you a 9. Sexually associated infection
grasp of the special topics and Must know
things especially if you are cramming”  Bacterial Vaginosis

**Combined with all the latest Recalls and 10. Strawberry cervix
Major Examination available from
Prelims/Midterms/Finals 2016 / 2017 and 2018  Trichomonas

A. pH >5 Write A if TRUE and B if FALSE


B. pH <4.5
11. The average age of menopause for
1. Physiologic vaginal discharge Filipina is 47-48 years of age.

 pH <4.5  TRUE

2. Bacterial vaginosis 12. The age of menopause is NOT


genetically programmed.
 pH >5
 FALSE
3. Trichomonas infection

 pH >5 13. Premature ovarian failure may be


treated with estrogen only.
4. Atrophic Vaginal discharge
 TRUE
 pH >5
14. Estradiol is reduced to greater extent
5. Fungal Infection than estrone in menopause patients.

 pH >5  TRUE

MATCHING TYPE: 15. Nocturnal sleep disruption is the


hallmark feature of declining estrogen.
A. Bacterial Vaginosis
B. Trichomonas  FALSE
C. Candidiasis
16. Recurrent urinary tract infection in a
6. Shift in vaginal flora postmenopausal patient may be related to
estrogen deficiency.
 Bacterial Vaginosis
 TRUE
7. Clue cells

 Bacterial Vaginosis

“Never design your LIFE like a GARDEN Where anyone can walk, rather Design your LIFE like the SKY Where everyone desires
to REACH”

Special Credits to: | Darna | Totoro | LittleSnoozy | Alma | RatioTeam | RecallsTeam | Astral | SKCCSLAS | Katz
Disclaimer: Some of the items and answers here are not mine but was verified so, If you find discrepancies please feel free to
correct, this serves only as a guide for easy and efficient reviewing.
God bless!
2|Page Philippians 4:13 jpmrnmd
GYNECOLOGY – Mastery Review
“Master your Ultra Instinct”
Ultimate Reviewer ~800 items for Major Exams | Revalida | Board Exams
17. Loss of cortical bone is greater with distensibility of the vagina of a
estrogen deficiency than loss of trabecular bone. reproductively mature woman.

 FALSE  Correct statement - pg 213


Comprehensive Gynaecology
18. Osteopenia is defined by T score of -1 to
-2.5 standard deviations. 25. The vagina of a child is 4-5cm long and has
an alkaline Neutral pH.
 TRUE
 Incorrect - pg 213 Comprehensive
19. Taking bisphosphonates have a Gynaecology
significant effect on prevention of osteoporosis.
26. During the physical examination and rectal
 TRUE examination of the prepubertal child, no pelvic
masses except the cervix should be palpable.
20. Older asymptomatic women will benefit
in the quality of life with the use of hormone  Correct statement - pg 213
replacement therapy. Comprehensive Gynaecology

 TRUE 27. The normal prepubertal uterus and ovaries


are NOT palpable by rectal exam?
Write: A – Correct Statement
B – Incorrect Statement  Incorrect - pg 213 Comprehensive
Gynaecology
21. In the field of paediatric gynaecology, most
diagnostic errors result from errors of omission 28. The relative size ratio of cervix to uterus is
during the examination rather than errors of 2:5 2:1 in a child.
commission.
 Incorrect - pg 213 Comprehensive
 Correct statement - pg 213 Gynaecology
Comprehensive Gynaecology
29. It is estimated that 80% to 90% of outpatient
22. Many gynaecologic conditions in children visits of children to gynecologists involve the
can be diagnosed by history Inspection alone classic symptoms of vulvovaginitis: introital
irritation and discharge.
 Incorrect - pg 213 Comprehensive
Gynaecology  Correct statement - pg 213
Comprehensive Gynaecology
23. The vaginal epithelium of the prepubertal
child appears redder and thinner than vaginal 30. Positive identification of gonorrhea or
epithelium of a woman in her reproductive years chlamydia in a child with premenarchal
vulvogaginitis is considered diagnostic of sexual
 Correct statement - pg 213 molestation.
Comprehensive Gynaecology
 Correct statement - pg 213
 24. The prepubertal vagina is also Comprehensive Gynaecology
narrower, thinner, and lacking in the

“Never design your LIFE like a GARDEN Where anyone can walk, rather Design your LIFE like the SKY Where everyone desires
to REACH”

Special Credits to: | Darna | Totoro | LittleSnoozy | Alma | RatioTeam | RecallsTeam | Astral | SKCCSLAS | Katz
Disclaimer: Some of the items and answers here are not mine but was verified so, If you find discrepancies please feel free to
correct, this serves only as a guide for easy and efficient reviewing.
God bless!
3|Page Philippians 4:13 jpmrnmd
GYNECOLOGY – Mastery Review
“Master your Ultra Instinct”
Ultimate Reviewer ~800 items for Major Exams | Revalida | Board Exams
31. The term complex hyperplasia is
synonymous with? C. Adenocarcinoma - Berek and Novaks

A. Simple Hyperplasia 36. Which histologic subtype of endometrial


carcinoma has the poorest prognosis?
B. Atypical Hyperplasia
A. Adenocarcinoma
C. adenomatous hyperplasia - Novaks Pg
1835 B. Adenosquamous adenocarcinoma

D. Cystic C. Squamous carcinoma

D. ethinyl estradiol D. Papilary adenocarcinoma

33. In the 2009 FIGO surgical staging a patient


with endometrial carcinoma involving the 37. The histologic criteria of diagnosing
cervical stroma is stage as? leiomyosarcoma is?

A. Ia - Berek and Novaks A. more than 10 mitotic figures per 10 high


power field
B. II
B. more than 10 mitotic figures per 1 high power
C. IIa field

D. IIIa C. Cytologic atypia

34. A 52 year old G1P1 patient came to your D. Vascular Involvement


clinic for post menopausal bleeding. On pelvic
exam her uterus was slightly enlarged. 38. The primary treatment for sarcoma of the
Transvaginal Ultrasound revealed a thickened uterus is?
endometrial lining of 1-2 cm. What will be your
next step in the management of this case? A. Radiation therapy

A. Endometrial Biopsy - Berek and Novaks B. Chemotherapy

B. Total Hysterectomy, bilateral salphingo- C. Radiation followed by surgery


oophrectomy
D. Surgery - Berek and Novaks
C. Radical Hysterectomy, bilateral salphingo-
oophrectomy 39. Which of the following is a risk factor for the
development of endometrial carcinoma?
D. Total Hysterectomy
35. Which is the most common histologic A. HPV infection
subtype in endometrial carcinoma?
B. Late menopause
A. Endometrial Carcinoma
41. Bacterial Vaginosis
B. Papilary Adenocarcinoma

“Never design your LIFE like a GARDEN Where anyone can walk, rather Design your LIFE like the SKY Where everyone desires
to REACH”

Special Credits to: | Darna | Totoro | LittleSnoozy | Alma | RatioTeam | RecallsTeam | Astral | SKCCSLAS | Katz
Disclaimer: Some of the items and answers here are not mine but was verified so, If you find discrepancies please feel free to
correct, this serves only as a guide for easy and efficient reviewing.
God bless!
4|Page Philippians 4:13 jpmrnmd
GYNECOLOGY – Mastery Review
“Master your Ultra Instinct”
Ultimate Reviewer ~800 items for Major Exams | Revalida | Board Exams
 Metronidazole 500mg twice daily for 7 B. Secondary Amenorrhea
days C. Precocious Puberty

42. Vaginal Candidiasis 53. Onset of female breast

 Fluconazole 150mg single dose A. Menarche


B. Telarche
43. Trichomoniasis C. Adrenarche
D. Rubarche
 Metronidazole 2gm Single dos
Rationale:
44. Gonorrhea  THELARCHE - onset of female
BREAST development
 Azithromycin 1gm single dose  RUBARCHE - appearance of SEXUAL
HAIR
45. Chlamydia  ADRENARCHE - onset of androgen-
dependent body changes such as
 Azithromycin 1gm single dose GROWTH OF AXILLARY & PUBIC
HAIR, BODY ODOR, ACNE
46. Clue Cells  MENARCHE - onset of menstruation

 Bacterial vaginosis 54. In Compartment 1, as to anatomical


category of the etiology of amenorrhea, the
47. Hyphae and Pseudohypae anatomical structure to examine is:

 Active Candidiasis A. Ovaries


B. Uterus
48. Cottage Cheese-like discharge C. Pituitary gland
D. Hypothalamus
 Vaginal Candidiasis
 Rationale:
49. Strawberry cervix COMPARTMENT 1 “The Outflow
Tract” uterus, cervix, and vagina
 Trichomoniasis
 T-esticular feminization
50. Mucopurulent discharge  I-mperforated hymen
 A-sherman’s syndrome
 Gonorrhea  R-adiotherapy
 U-terine agenesis
51. Secondary amenorrhea, pituitary?
COMPARTMENT 2 “The Ovaries”
 Sheehan’s
 S-urgery
52. Absence of menses regardless of  O-varian failure
presence of secondary sexual  C-hemo/radiotherapy
characteristics by age of 16?  R-esistant ovary
syndrome
A. Primary Amenorrhea

“Never design your LIFE like a GARDEN Where anyone can walk, rather Design your LIFE like the SKY Where everyone desires
to REACH”

Special Credits to: | Darna | Totoro | LittleSnoozy | Alma | RatioTeam | RecallsTeam | Astral | SKCCSLAS | Katz
Disclaimer: Some of the items and answers here are not mine but was verified so, If you find discrepancies please feel free to
correct, this serves only as a guide for easy and efficient reviewing.
God bless!
5|Page Philippians 4:13 jpmrnmd
GYNECOLOGY – Mastery Review
“Master your Ultra Instinct”
Ultimate Reviewer ~800 items for Major Exams | Revalida | Board Exams
 O-varian agenesis/ B. Bulimia
dysgenesis C. Both
 P-olycystic ovary D. Neither
syndrome  Rationale: Most common cause
of amenorrhea is Anorexia
COMPARTMENT 3 “The Anterior nervosa
Pituitary Gland”
57. What is the most probable cause of
 T- secondary amenorrhea if estradiol level is
Non-functional tumor normal and pelvic ultrasound is normal?
 I-
syndrome A. PCO
 R-adiotherapy B. Hyperandrogenism
 I-nfection - Tuberculosis C. POF
 S-urgery D. Hypothalamuc disorder

58. Secondary to necrosis of pituitary...


COMPARTMENT 4 “The Hypothalamus”
 Sheehan’s
 C-
syndrome 59. Most common cause of GnRH-
 Acquired independent precocious puberty is
 P-ituitary stalk
disconnection syndrome A. Granulosa
 E-xcessive weight loss B. Theca
 C-ranial radiotherapy C. Leydig
 E-xtreme exercise D. Sertoli

55. The outflow tract obstruction that leads to 60. Presence of ovaries will cause regression
primary amenorrhea is/are of mesonephric ducts and development of
paramesonehric ducts into:
A. Perforated hymen
B. Transverse septum in vagina A. Female genital tract
C. Hydro/pyosalphinx B. Male genital tract
D. Cervicitis C. Promordial follicle

 Rationale: Imperforated 61. 23 y/o nulligravid diagnosed with pituitary


microadenoma, 2yrs married wants to get
because of the closed outflow pregnant. What will be the first line of
tract treatment

56. The singer, Ms. Caren Carpenters, once A. transphenoidal resection of


had an extreme weight loss and distorted microadenoma
implaceablr attitude towards eating. This B. bromocriptine
psychiatric condition associated with C. Cergolide
amenorrhea is known as: D. Periodic Progestin withdrawal

A. Anorexia nervosa

“Never design your LIFE like a GARDEN Where anyone can walk, rather Design your LIFE like the SKY Where everyone desires
to REACH”

Special Credits to: | Darna | Totoro | LittleSnoozy | Alma | RatioTeam | RecallsTeam | Astral | SKCCSLAS | Katz
Disclaimer: Some of the items and answers here are not mine but was verified so, If you find discrepancies please feel free to
correct, this serves only as a guide for easy and efficient reviewing.
God bless!
6|Page Philippians 4:13 jpmrnmd
GYNECOLOGY – Mastery Review
“Master your Ultra Instinct”
Ultimate Reviewer ~800 items for Major Exams | Revalida | Board Exams
62. Ultrasound diagnosis of PCO is made in A. Methylation
the basis of w/c of the ff.? B. Aromatization
C. 5-a reductase action
A. Finding of enlarged ovaries D. Halogenation
(>10cm3) and produce 10 or more
peripherally crowded cystic 68. Other name of PCOS?
structures (2-8mm)
B. Enlarged ovary >3cm w/ multiple  Stein leventhal syndrome
cystic structures in ovarian cortex
C. A multiloculated ovary each lobule 69. Tumor associated with excessive estrogen
measuring >10cm production. Which may cause pseduo
D. NOTA precocious puberty, post menoposal
bleeding?
63. Congenital abnormality involving the
hypothalamus causing idiopathic A. Granulosa cell - theca cell
hypogonadotrophic hypogonads with B. Sertoli ledig cell
anosmia C. Gonadoblastoma
D. Immature teratoma
A. Kallman syndrome
B. Sheehan's syndrome 70. Non-epithelial ovarian neoplasm that
C. PCOS resembles fetal testis and mimic tumors of
D. Asherman's syndrome the adrenal glands:

64. Alternative to Clomiphene? A. Granulosa-theca cell tumor


B. Sertoli-leydig cell tumor
 Letrozole C. Gonadoblastoma
D. Immature Teratoma
65. Treatment for Metabolic Syndrome in
PCOS is best achieved: 71. Testosterone (>200 ng/ml), temporal
balding, clitoral hypertrophy, voice
A. Diet + Metformin deepening, breast atrophy
B. Bariatic Surgery
C. OCP A. Dysgerminoma
D. Insulin B. Sertoli Leydig cell
C. Granulosa - theca cell
66. Although therapy for PCOS is directed at D. Endodermal sinus tumor
woman's specific complaint, which of the ff
should be the mainstay of treatments 72. True of Dysgerminoma?

A. Lifestyle change including weight A. Peak age over 45yo


reduction and fitness B. CA125 increase 50% of case
B. Metformin C. More of solid rather than cystic
C. OCP D. Causes increase in AFP
D. Finasteride
73. A cystic benign tumor can undegro
67. Ovarian testosterone converted to malignant transformation, and estimated to
estrogen thru: occur in <2% of these tumors, the

“Never design your LIFE like a GARDEN Where anyone can walk, rather Design your LIFE like the SKY Where everyone desires
to REACH”

Special Credits to: | Darna | Totoro | LittleSnoozy | Alma | RatioTeam | RecallsTeam | Astral | SKCCSLAS | Katz
Disclaimer: Some of the items and answers here are not mine but was verified so, If you find discrepancies please feel free to
correct, this serves only as a guide for easy and efficient reviewing.
God bless!
7|Page Philippians 4:13 jpmrnmd
GYNECOLOGY – Mastery Review
“Master your Ultra Instinct”
Ultimate Reviewer ~800 items for Major Exams | Revalida | Board Exams
malignant transformation usually occur at  Early Menarche and Late
what part of dermoid element? Menopause

A. Endodermal layer 80. Risk for endometrial ca?


B. Mesodermal layer
C. Ectodermal layer  Unopposed Estrogen
D. Squamous Epithelium
81. Endometrial thickness of 4mm or less on
74. Most common primitive germ cell tumor, ultrasound in an asymptomatic
which account to 20-30% ovarian tumor postmenopausal woman.
during pregnancy. Counter part of
testicular seminoma. A. Maybe a normal finding
B. Risk of endometrial ca is increased
A. Endoderml sinus tumor C. Currettage can be performed
B. Dysgerminoma D. Indication of further investigation
C. Immature teratoma
82. 33 years old nulligravid underwent
75. The minimal invasive surgery for endometrial curretage d/t AUB. Result
dysgerminoma adenomatous atypical hyperplasia who is
still desirous of pregnanct. What is the
A. Oopherocystectomy management?
B. Oopherectomy
C. TAHBSO A. TAHBSO
D. TAHBSO with BND B. observe
C. High dose progestin
76. Mixed germ cell malignancy,m/c
combination? 83. According to 1988 FIGO staging of
endometrial CA is based on:
 Dysgerminoma and
Endodermal Sinus Tumor A. Clinical findings
B. Intraoperative findings
77. 2yo male; seminalysis revealed low sperm C. Surgicopathologic findings
motility D. Histopathologic findings

A. Kartageners synd esp w/ absent 84. Endometrial adenocarcinoma and


sperm cilia rhabdomyosarcoma of the uterus?
B. Chem & rad exposure
C. Cryptorchidism A. Endometrial stromal sarcoma
D. Pituitary tumor B. Endolympathic stromal
C. Leiomyosarcoma
78. Most common cause of infertility D. Malignant mixed mullerian tumor

A. Ovarian dysfunction 85. Groove sign?


B. Tubal factor
C. Uterine factor  lymphogranuloma venereum

79. Increased risk for endometrial carcinoma?

“Never design your LIFE like a GARDEN Where anyone can walk, rather Design your LIFE like the SKY Where everyone desires
to REACH”

Special Credits to: | Darna | Totoro | LittleSnoozy | Alma | RatioTeam | RecallsTeam | Astral | SKCCSLAS | Katz
Disclaimer: Some of the items and answers here are not mine but was verified so, If you find discrepancies please feel free to
correct, this serves only as a guide for easy and efficient reviewing.
God bless!
8|Page Philippians 4:13 jpmrnmd
GYNECOLOGY – Mastery Review
“Master your Ultra Instinct”
Ultimate Reviewer ~800 items for Major Exams | Revalida | Board Exams
86. Precludes to PID?
93. The CA125 is considered elevated
A. Use of IUD
B. Ectopic pregnancy A. 15-20
B. 20-25
87. Benign Bilateral Ovarian Tumor with no C. 25-30
subcapsular cyst? D. 30-35
E. >35
 Stromal Hyperthecosis
 Rationale: Results of the CA
88. High risk in developing to full blown cancer 125 test are measured in units
except? per milliliter (U/mL). The normal
value is less than 35 U/mL
 CIN 1
94. Figo staging for ovarian cancer with
89. A 25 year old G1P1 (1001) came back for bilateral extension beyond the pelvis plus
follow up. She underwent a cytology exam, extension into peritoneal fluid.
a week before her test revealed ASCUS.
What diagnostics? A. IC
B. IIa
A. HPV DNA testing C. IIIb
B. Repeat Pap Smear D. IIIc
C. Colposcopy
D. AOTA 95. True of CA125?

90. Pathophysiology of osteoporosis are the ff A. Specificity is better for high


except: values in post menopausal
women
A. Low calcium intake B. Specific for epithelial ovarian
B. High estrogen level neoplasm
C. Aging C. For follow up/ evaluation in post op
D. Hereditary D. A and C

91. Consist of epithelial cells resembling those 96. Indications for conservative treatment of
of endometrium epithelial ovarian ca except

A. Mucinous A. Stage 1A
B. Serous B. Multiparity
C. Endometrioid C. Well differentiated
D. Clear cell D. Negative for cytology

92. Histologic criteria for diagnosis of 97. Which of the ff is considered as a minimum
borderline malignant criteria in the diagnosis of PID

A. Atypicality A. Hypogastric pain


B. Epithelial pleomorphism B. Rebound tenderness
C. No stromal invasion C. Cervical wiggling tenderness
D. All of the above D. All

“Never design your LIFE like a GARDEN Where anyone can walk, rather Design your LIFE like the SKY Where everyone desires
to REACH”

Special Credits to: | Darna | Totoro | LittleSnoozy | Alma | RatioTeam | RecallsTeam | Astral | SKCCSLAS | Katz
Disclaimer: Some of the items and answers here are not mine but was verified so, If you find discrepancies please feel free to
correct, this serves only as a guide for easy and efficient reviewing.
God bless!
9|Page Philippians 4:13 jpmrnmd
GYNECOLOGY – Mastery Review
“Master your Ultra Instinct”
Ultimate Reviewer ~800 items for Major Exams | Revalida | Board Exams
103. Patient sought consult d/t vaginal
98. Most accurate method for diagnosis PID is discharge, you ask the patient the following
thru? questions EXCEPT?

A. positive fever, high ESR, adnexal A. Color of discharge?


tenderness B. Is discharge foul smelling?
B. direct visualization of infected C. Is it related to a specific type of
upper genital organs food
C. increased ESR D. Does it cause itchiness
D. positive free peritoneal fluid by
ultrasound 104. Clinical significance of Breast Self-Exam

99. The specimen of this procedure is  Identify cancer in young


collected by swabbing endocervix and women who are not typically
ectocervix. candidates for mammography

A. Papsmear 105. Minimum criteria to dx PID


B. Colposcopy
C. Cervical biopsy A. Hypogastric pain
D. Cyotheraphy B. Cervical tenderness

100. A 45 yo, nulligravid obese hypertensive 106. 3-alphadiol-G measurement


complaining of menstrual bleeding
consuming 5 baby pads per day A. Skin
B. Hair
A. Pap test C. Wedge resection
B. Colposcopy D. All of the above
C. Endometrial biopsy
D. Cryotherapy 107. 20 years old woman check up
complaining of appearance of mustache.
101. Treatment for small CIN I. Blood assay revealed, testosterone of
1mg, DHEAS of 8mg, DHEA of 0.6mg, and
A. Freezing androsteronedione of 1mg. Increased hair
B. Pap smear growth is attributed to the increased
C. Colposcopy production of androgen in the?
D. Cervical biopsy
E. Endometrial biopsy A. Ovary
B. Adrenals
102. The best position to obtain vaginal C. Skin
specimen in a 2 year old child. Choices:
108. A 53 y/o patient 4 years menopause
A. Dorsal noticed temporal balding and deepening of
B. Lithotomy the voice pelvic exam unremarkable with
C. Knee chest testosterone levels 3mg and dhea 24mg
D. Frog leg what is your diagnosis?

A.Idiopathic Hirsutisim
B. PCOS

“Never design your LIFE like a GARDEN Where anyone can walk, rather Design your LIFE like the SKY Where everyone desires
to REACH”

Special Credits to: | Darna | Totoro | LittleSnoozy | Alma | RatioTeam | RecallsTeam | Astral | SKCCSLAS | Katz
Disclaimer: Some of the items and answers here are not mine but was verified so, If you find discrepancies please feel free to
correct, this serves only as a guide for easy and efficient reviewing.
God bless!
10 | P a g e Philippians 4:13 jpmrnmd
GYNECOLOGY – Mastery Review
“Master your Ultra Instinct”
Ultimate Reviewer ~800 items for Major Exams | Revalida | Board Exams
C. LOHD  Enlarged ovaries (>10cm) 10 or
D. Hilus tumor more peripherally oriented
E. Sertoli leydig tumor cystic structures (2-8mm)
surrounding dense stroma
109. Not complete. Testosterone level
>3ng/ml with bilateral enlargement of 116. Arises from walthard nest of the ovary
ovaries?
A. serous
 Stromal hyperthecosis B. Mucinous
C. Endometriod
110. The patient will be taller than her D. Brenner
playmates of same age of 8 but eventually
wii be short stature after puberty? 117. Onset of female breast development

A. Cushing syndrome A. Menarche


B. Early onset of PCOS B. Thelarche
C. LODH C. Adrenarche

111. Therapy for LOHD to bring back 118. Absence of menses regarless of
ovulation? presence of secondary characteristics by
Age 16?
A. Suppress testosterone production of
the ovaries A. Primary amenorrhea
B. Suppress adrostenedione and
normalize hydroxyprogesterone 119. Best time to measure serum
levels progesterone in women taking clomiphene
C. Correct hirsutism citrate?

112. This vaginal cancer is common among A. Exactly at midluteal phase


women who were exposed to B. 1 week after last intake clomiphene
Diethysilbesterol in utero citrate
C. 2 weeks after last intake clomiphene
A. Sarcoma botryoides citrate
B. Yolk sac tumors D. 3 days after increase of basal body
C. DES tumors temperature
D. Clear cell CA
120. 5. The use of gonadotropin therapy is
113. Treatment of choice for bacterial indicated for ovulation induction when:
vaginosis?
A. Estrogen concentration or levels are
 Metronidazole 500 mg BID for 7 days too low
B. Lack of withdrawal bleeding after
114. Other name for PCOS progesterone bleeding
C. Clomiphene citrate and letrozole fails
 Stein leventhal syndrome D. A and B
E. A, B and C
115. Ultrasound diagnosis of PCOS

“Never design your LIFE like a GARDEN Where anyone can walk, rather Design your LIFE like the SKY Where everyone desires
to REACH”

Special Credits to: | Darna | Totoro | LittleSnoozy | Alma | RatioTeam | RecallsTeam | Astral | SKCCSLAS | Katz
Disclaimer: Some of the items and answers here are not mine but was verified so, If you find discrepancies please feel free to
correct, this serves only as a guide for easy and efficient reviewing.
God bless!
11 | P a g e Philippians 4:13 jpmrnmd
GYNECOLOGY – Mastery Review
“Master your Ultra Instinct”
Ultimate Reviewer ~800 items for Major Exams | Revalida | Board Exams
 Rationale: Gonadotropin therapy is 123. The problem on infertility is associated
indicated for ovulation induction with?
when estrogen levels are low. Low
serum E2 levels (usually < 30 A. Age of the woman
pg/mL) or lack of withdrawal B. Disorders of ovulation
bleeding after progestogen C. Age of the husband
administra-tion D. AOTA

Apart from this indication in usually 124. The most common cause of menstrual
amenorrheic women, it is appropriate to disorder among women presenting with
use gonadotropins in clomiphene/letrozole infertility is?
failures
A. PCOS
121. The highest percentage of potential risk B. Mullerian duct agenesis
complication in singleton pregnancy after C. Perimenopausal
IVF is? D. Pelvic inflammatory disease

A. Preterm birth 125. Regarding etiologic factor of infertility,


B. SGA which of the ff. is CORRECT?
C. Neonatal ICU admission
D. Low birth weight A. Tubal and peritoneal factors account for
10% of cases of female infertility
122. Best therapy offered in cases of B. Cervical factor is estimated to be a
proximal and distal obstruction of the tube cause of infertility in no more than 2% of
is? infertile couples
C. Uterine pathologies as etiologic factor
A. IUI are diagnosed in as many as 90% of
B. IVF infertile patients
C. Ovulatory drug D. Leiomyomas have not been shown to
D. Operative reconstructive therapy be a direct cause of infertility

 Rationale: Distal obstruction is 126. To monitor ovulation, the basal body


much more common than proximal temperature (BBT) is being monitored. It
obstruction if there is extensive usually increases when circulating levels
damage, the chances for of?
conception after tubal
reconstruction are very unlikely. A. Progesterone increase
Women with extensive tubal B. Lutenizing hormone surge
disease have a greater chance of C. Estradiol increase
conceiving with an IVF procedure, D. Follicle stimulating hormone increase
and thus the extent and location of
the intrinsic and extrinsic tubal 127. Which one of the ff. is TRUE regarding
disease should be ascertained by ovulation?
HSG and possibly laparoscopy in
an effort to determine whether A. associated with decreasing levels of
tubal reconstruction or IVF offers estrogens
the better prognosis B. may lead to the formation of corpus
luteum

“Never design your LIFE like a GARDEN Where anyone can walk, rather Design your LIFE like the SKY Where everyone desires
to REACH”

Special Credits to: | Darna | Totoro | LittleSnoozy | Alma | RatioTeam | RecallsTeam | Astral | SKCCSLAS | Katz
Disclaimer: Some of the items and answers here are not mine but was verified so, If you find discrepancies please feel free to
correct, this serves only as a guide for easy and efficient reviewing.
God bless!
12 | P a g e Philippians 4:13 jpmrnmd
GYNECOLOGY – Mastery Review
“Master your Ultra Instinct”
Ultimate Reviewer ~800 items for Major Exams | Revalida | Board Exams
C. stimulated by high levels of
progesterone 133. Cervical malignancy not affected by
D. followed by increasing levels of usual sexual factors but risk of
estrogens development is increased with use of
OCP?
128. Couple with infertility is defined as
failure to achieve pregnancy after  Adenocarcinoma
unprotected coitus for a period of? 134. Histologic type of Cervical CA that is
most common seen in pregnancy.
A. 3 years
B. 6 years A. Squamous CA
C.1 month B. Adenosquamous CA
D. 1 year C. Adenocarcinoma
D. Endometriod cervical CA
129. The rise in the basal body temperature
indicates that? 135. The findings of hydronephrosis in
cervical ca is assign?
A. Woman has PID
B. Ovulation has taken place A. IIIA
C. Follicular secretion of estrogen B. IIIB
D. Fertilization has occurred C. IIIC
D. IIIA2
130. Which of the ff. hormones is the MOST
reliable predictor of ovulation? 136. Clinically visible lesion limited to the
cervix
A. LH
B. FSH A. 1 A2
C. Estrogen B. 1 B2
D. Progesterone C. 2A
D. 2B
131. Angelica, 20y/o primary infertile, BMI 35,
complained of amenorrhea for 6 months. 137. Barrel shaped cervix?
The LH/FSH ratio is 3.0. Your initial
treatment is? A. Verrucous
B. Exophytic
A. Spironolactone C. Endophytic
B. Clomiphene citrate D. AOTA
C. Human menopausal gonadotrophin
D. Progesterone 138. This node is assessed in advance
cervical carcinoma?
132. Treatment of patients in the reproductive
age group who are currentlydesirous to get A. Internal inguinal node
pregnant is by: B. External inguinal node
C. Inguinal node
A. Induction of ovulation D. Left scalene node
B. Progesterone only pills
C. Triphasic pills 139. Mode of spread of CCA if metastasize to
D. Estrogen + progesterone pills liver lungs and bone?

“Never design your LIFE like a GARDEN Where anyone can walk, rather Design your LIFE like the SKY Where everyone desires
to REACH”

Special Credits to: | Darna | Totoro | LittleSnoozy | Alma | RatioTeam | RecallsTeam | Astral | SKCCSLAS | Katz
Disclaimer: Some of the items and answers here are not mine but was verified so, If you find discrepancies please feel free to
correct, this serves only as a guide for easy and efficient reviewing.
God bless!
13 | P a g e Philippians 4:13 jpmrnmd
GYNECOLOGY – Mastery Review
“Master your Ultra Instinct”
Ultimate Reviewer ~800 items for Major Exams | Revalida | Board Exams
A. hematogenous  Vascular metastasis
B. Lymphatics
C. Vascular 148. Danazol
D. Do not metastasize
 Androogen
140. Staging of cervical carcinoma is
dependent prilamrily on which of the ff?

A. Pelvic exam 149. Leuprolide


B. Opertative findings
C. CT/MRI findings  GNRH agonist
D. CxR findings
150. Mefenamic acid
141. Female 20 weeks AIG with cervical
carcinoma in situ, management?  NSAIDs

A. Observe & deliver…


B. teletherapy after abortion 151. Dienogest
C. Hysterectomy
D. Chemoradiation  Progestogen

142. Meigs-wertheim hysterectomy is also 152. Medroxyprogesterone acetate


known as?
 Progestogen
A. class I
B. class II 153. Endometriosis is the presence and
C. Modified radical hysterectomys growth of the endometrial glands and
D. Radical hysterectomy stroma in an aberrant or heterotopic
location?
143. Thigh
 TRUE
 Vascular metastasis
154. The most popular theory in the etiology
144. Ovary of endometriosis is the retrograde
menstruation theory?
 Retrograde menstruation
 TRUE

145. Episiotomy 155. The induction substance in the


metaplasia theory of endometriosis could
 Iatrogenic be a combination of menstrual debris and
estrogen?
146. Pelvic lymph node
 TRUE
 Lymphatic metastasis
156. The theory that can explain
endometriosis in the Cesarean section
147. Lung scar is the vascular metastasis theory?

“Never design your LIFE like a GARDEN Where anyone can walk, rather Design your LIFE like the SKY Where everyone desires
to REACH”

Special Credits to: | Darna | Totoro | LittleSnoozy | Alma | RatioTeam | RecallsTeam | Astral | SKCCSLAS | Katz
Disclaimer: Some of the items and answers here are not mine but was verified so, If you find discrepancies please feel free to
correct, this serves only as a guide for easy and efficient reviewing.
God bless!
14 | P a g e Philippians 4:13 jpmrnmd
GYNECOLOGY – Mastery Review
“Master your Ultra Instinct”
Ultimate Reviewer ~800 items for Major Exams | Revalida | Board Exams
 FALSE  TRUE
165. Cytokines and growth factor that have
157. Natural killer cells have decreased been implicated in pathogenesis of
cytotoxicity against endometrial and metaplasia?
hematopoetic cells in women with
endometriosis?  FALSE

 TRUE 166. If there’s obstruction in the outflow tract,


endometriosis will happen. This can be
158. The most common site of endometriosis explain by retrograde menstruation?
is the uterosacral ligament?
 TRUE
 FALSE
167. Local production of estrogen and
159. The classic symptoms of endometriosis progesterone explains why progression of
are cyclic pelvic pain and infertility? lesion may occur even w/ ovarian
suppression?
 TRUE
 FALSE
160. The extent of pelvic pain is directly
related to the amount of endometriosis in 168. New lesion are described a whitish &
the female pelvis? p____

 TRUE  FALSE

161. Medical therapy for endometriosis 169. Chronic pelvic pain experienced by
suppresses symptoms, prevents patient w/ endometriosis is secondary
progression and provides a long term cure dysmenorrhea?
of the disease?
 TRUE
 FALSE
170. Serial pelvic examination is a good
162. GnRH agonist used for endometriosis indicator of progression of the disease?
would decrease the patient’s bone mineral
content?  FALSE

 TRUE 171. Ovaries are the most common site of


endometriosis?
163. Adenomyosis is the growth of
endometrial glands and stroma into the  TRUE
uterine myometrium to a depth of 1.5 mm
from the basalis layer of endometrium? 172. Primary short-term goal in patients w/
endometriosis is to prevent progression or
 FALSE recurrence of the disease process?

164. Endometriosis is 30-45% in women w/  FALSE


infertility?

“Never design your LIFE like a GARDEN Where anyone can walk, rather Design your LIFE like the SKY Where everyone desires
to REACH”

Special Credits to: | Darna | Totoro | LittleSnoozy | Alma | RatioTeam | RecallsTeam | Astral | SKCCSLAS | Katz
Disclaimer: Some of the items and answers here are not mine but was verified so, If you find discrepancies please feel free to
correct, this serves only as a guide for easy and efficient reviewing.
God bless!
15 | P a g e Philippians 4:13 jpmrnmd
GYNECOLOGY – Mastery Review
“Master your Ultra Instinct”
Ultimate Reviewer ~800 items for Major Exams | Revalida | Board Exams
183. The statements/s are true regarding
173. The coelomic epithelium endometriosis?
retains the ability for METAPLASIA
multipotential development A. It is benign
174. This theory usually B. It has a wide spectrum of clinical
LYMPHATIC ANF
explains the rare and remote problems
VASCULAR
sites of endometriosis C. Because of laparoscopy, there an
METASTASIS
increasing awareness of mild
175. A woman may develop endometriosis
the condition earlier in life GENETIC D. ABC
and usually have more PREDISPOSITION E. A and B
advanced disease.
176. The local production of 184. This is the typical patient with
estrogen through aromatase endometriosis?
activity explains why IMMUNOLOGIC
progression of lesion may CHANGES A. Mid 30’s
occur even with ovarian B. With secondary dysmenorrhea
suppression C. Multiparrous
177. Secondary to D. ABC
implantation of endometrial RETROGRADE E. A and B
cells shed during a woman’s MENSTRUATION
monthly period 185. This is considered as the most popular
theory for the development of endometriosis?
MATCHING TYPE:
A. Danazol A. Metaplasia
B. Oral contraceptives B. Retrograde menstruation
C. GnRH agonist C. Iatrogenic dissemination
D. NSAIDs D. Vascular metastasis
E. Dienogest E. Immunologic changes
178. COX-2 inhibitors are
under this category NSAIDs 186. Endometrial of the forearm and thigh
can be explained by this theory?
179. With a mildly
androgenic and anabolic DANAZOL A. Metaplasia
effect B. Retrograde menstruation
180. Cause a dramatic C. Iatrogenic dissemination
reduction in serum estrone, D. Vascular metastasis
GnRH AGONIST E. Immunologic changes
estradiol, testosterone, and
androstenedione
181. Considered to be the 187. This/these is/are considered as the
most economical regimen in ORAL primary immunologic change involve in the
the treatment of CONTRACEPTIVES pathogenesis of endometriosis?
endometriosis
182. Is a progestogen that A. Cytokine alteration
has an antiproliferative effect DIENOGEST B. Growth factors proliferation
on the endometrial cells C. Peritoneal macrophage function
alteration
D. ABC

“Never design your LIFE like a GARDEN Where anyone can walk, rather Design your LIFE like the SKY Where everyone desires
to REACH”

Special Credits to: | Darna | Totoro | LittleSnoozy | Alma | RatioTeam | RecallsTeam | Astral | SKCCSLAS | Katz
Disclaimer: Some of the items and answers here are not mine but was verified so, If you find discrepancies please feel free to
correct, this serves only as a guide for easy and efficient reviewing.
God bless!
16 | P a g e Philippians 4:13 jpmrnmd
GYNECOLOGY – Mastery Review
“Master your Ultra Instinct”
Ultimate Reviewer ~800 items for Major Exams | Revalida | Board Exams
E. A and B 192. A conservative surgery for
188. The ff. Statements is/are true regarding endometriosis involves the ff?
the theory of genetic predisposition as a
pathogenesis of endometriosis? A. Resections of endometrial implants
B. Lysis of adhesions
A. There is an increase in the incidence of C. Oophorocystectomy
endometriosis in relatives of women with D. ABC
disease E. A and B
B. The expression of this genetic liability
most likely depends on an interaction 193. Feminism of undifferentiated external
with environmental and epigenetic? genitalia happens in the absence of androgen
C. Women who have a family history of stimulation?
endometriosis are likely to develop the
disease earlier in life and a less disease A. True
D. ABC B. False
E. A and B
193. The sperm are attracted to an egg
189. The ff statements is/are true regarding through the process known as?
the deep lesions of endometriosis?
A. Capacitation
A. It represents a more progressive form of B. Chemotaxis
disease C. Acrosomal Reaction
B. Distinguishing these lesions is important D. NOTA
for therapy
C. It is a penetration of >5mm in the 194. Teratogen exposure after 49th day of
affected area/organ gestation may injure or kill the embryo or cause
D. ABC developmental and growth retardation but
E. A and B usually will not be responsible for specific
malformations?
190. This/these is/are considered as rare
sites of endometriosis? A. True
B. False
A. Appendix
B. Pelvic lymph node 195. The period of embryonic development is
C. Umbilicus said to be complete when the embryo attains a
D. ABC crown rump length of?
E. B and C
A. 10mm
191. The classic symptom/s of endometriosis B. 20mm
include? C. 30mm
D. 40mm
A. Chronic pelvic pain
B. Infertility 196. Fetal intrauterine produces urine
C. Abnormal uterine bleeding throughout pregancy that contribute to amniotic
D. ABC fluid?
E. A and B
F. A. True
B. False

“Never design your LIFE like a GARDEN Where anyone can walk, rather Design your LIFE like the SKY Where everyone desires
to REACH”

Special Credits to: | Darna | Totoro | LittleSnoozy | Alma | RatioTeam | RecallsTeam | Astral | SKCCSLAS | Katz
Disclaimer: Some of the items and answers here are not mine but was verified so, If you find discrepancies please feel free to
correct, this serves only as a guide for easy and efficient reviewing.
God bless!
17 | P a g e Philippians 4:13 jpmrnmd
GYNECOLOGY – Mastery Review
“Master your Ultra Instinct”
Ultimate Reviewer ~800 items for Major Exams | Revalida | Board Exams
204. The AMH is produced by the sertoli cells
and it acts systemically?

197. Early in the embryo life, there are two A. FALSE


sets of paired genital ducts that develop in each 205. The round ligament is formed 8weeks
sex. These are the Wolffian ducts and the AOG?
Mullerian ducts?
A. True
A. True
B. False 206. Failure of the sinovaginal bulb result to
agenesis of vagina?
198. In the presence of ovaries or of gonadal
genesis, the mesonephric duct regress, and the A. True
paramesonephric ducts develop into?
207. The major effect of synthetic progestin
Female Genital Tract component of OCP is:

198. What necessary for the development of A. Makes cervical mucus slippery and
the testis? elastic
B. Maintains the endometrium
A. Y chromosome C. Inhibit Ovulation
D. Prevent unscheduled bleeding
199. This hormones or substances are
involved in sexual diff of male genesis? 208. What is the estrogen component used in
contraceptive preparations that is orally
A. Testosterone & its metabolite active and highly potent?
Dihydrotestosterone
A. 17b-estradiol
200. The hormone testosterone stimulates B. Mestranol
the development of what organ? C. Esthinyl estradiol
D. Estrone valerate
A. Prostate gland
209. This OCP formulation contains a low
201. Some seminiferous tubules are progestin without any estrogen:
produced at what AOG?
A. Monophasic
A. 7th-8th weeks B. Minipill
C. Biphasic
202. Two functional chromosome X is D. Triphasic
necessary for the development of functional
ovary? 210. The ff. symptoms are related to estrogen
component of OCP, EXCEPT:
A. True
A. Nausea
203. New oogonia are formed at birth? B. Breast tenderness
C. Fluid retention
A. FALSE D. Weight gain

“Never design your LIFE like a GARDEN Where anyone can walk, rather Design your LIFE like the SKY Where everyone desires
to REACH”

Special Credits to: | Darna | Totoro | LittleSnoozy | Alma | RatioTeam | RecallsTeam | Astral | SKCCSLAS | Katz
Disclaimer: Some of the items and answers here are not mine but was verified so, If you find discrepancies please feel free to
correct, this serves only as a guide for easy and efficient reviewing.
God bless!
18 | P a g e Philippians 4:13 jpmrnmd
GYNECOLOGY – Mastery Review
“Master your Ultra Instinct”
Ultimate Reviewer ~800 items for Major Exams | Revalida | Board Exams
216. The ff. contraceptive could be given for
211. What does WHO category 4 means in a patient taking anticonvulsant drugs,
WHO medical eligibility criteria: EXCEPT:

A. can used the method without A. DMPA


restriction B. IUD
B. Advantage generally outweigh C. Subdermal implant
theoretical or proven risk D. Low dose OCP
C. Theoretical or proven risk usually
outweigh the advantages 217. Hormonal contraceptive are female sex
D. Condition represents an steroids which include/s
unacceptable health risk if
method is used A. Synthetic estrogen
B. Synthetic progestin
212. Which of the ff. is relative C. Synthetic androgen
contraindication for OCP used: D. AOTA
E. A and B only
A. Gestational DM
B. Undiagnosed AUB 218. The following are anti-fertility effects of
C. Suspected pregnancy OCP, except:
D. Impaired liver function
A. Increases basal FSH and LH
213. How is contraceptive patch used: B. Little estradiol is produced
C. There is no midcycle LH surge
A. 1 patch each week for 4 weeks D. Ovarian follicles do not mature
B. 1 patch each week for 3 weeks
C. 1 patch is used for 1 month 219. Relative contraindication for OCP use
D. 1 patch every other week for 1
month A. Undiagnosed AUB
B. Markedly increase liver function test
214. What is the dose of DMPA used for C. Known or suspected Breast CA
contraception? D. Migraine and vascular headache

A. 150 ug 220. Prolong used of OCP increase the risk


B. 150 mg of which of the carcinoma:
C. 150 gm
D. 150 kg A. Endometrial carcinoma
B. Cervical carcinoma
215. The ff. are the effects of subdermal C. Breast carcinoma
implant D. Ovarian carcinoma

A. Endometrial injury 221. The following drugs reduces the


B. Infertility effectiveness of OCP, EXCEPT:
C. Irregular bleeding
D. AOTA A. Phenobarbital
E. A and C only B. Rifampicin
C. Tetracycline
D. Acetaminophen

“Never design your LIFE like a GARDEN Where anyone can walk, rather Design your LIFE like the SKY Where everyone desires
to REACH”

Special Credits to: | Darna | Totoro | LittleSnoozy | Alma | RatioTeam | RecallsTeam | Astral | SKCCSLAS | Katz
Disclaimer: Some of the items and answers here are not mine but was verified so, If you find discrepancies please feel free to
correct, this serves only as a guide for easy and efficient reviewing.
God bless!
19 | P a g e Philippians 4:13 jpmrnmd
GYNECOLOGY – Mastery Review
“Master your Ultra Instinct”
Ultimate Reviewer ~800 items for Major Exams | Revalida | Board Exams
 TRUE
222. This type of OCP formulation contains 228. More than 80 points of blood loss is
the same dose of estrogen and considered to be menorrhagia.
progesterone given through a 21-day
cycle  TRUE

A. Monophasic 229 .Von Willebrand disease is the second


B. Biphasic most common cause of AUB in adolescents.
C. Triphasic
D. Multiphasic  FALSE

223. Injectable hormonal contraceptives like 230. In a reproductive age woman, the first
DMPA is given: consideration is she has AUB is
pregnancy.
A. 150mg IM per month
B. 150mg IM every 3 months  TRUE
C. 150mg IM weekly
D. 150mg IM every 6 month 231. Among all the age group, adolescents
have the highest chance of AUB due to
224. In subdural implants, what is the amount genital tract infection.
of progestin released during the first 6 to
12 months?  TRUE

A. 36mg/day
B. 80mg/day 232. Ovulatory AUB will not slough uniformly.
C. 0.35 ug/day
D. 0.25 ug/day  FALSE

225. Old women, who are smokers and have 233. In anovulatory AUB, it is caused by
poorly controlled HPN are given the prostaglandin secretion.
following family planning EXCEPT:
 FALSE
A. Combined OCP
B. Injectable 234. Endometrial biopsy must be done in all
C. Subdermal patch patients.
D. IUD
 FALSE
226. When is OCP given in spontaneous
abortion or induced abortion? 235. There is no need to perform CBC if the
bleeding of the patient is not profuse.
A. Immediately
B. 2-3 weeks  TRUE
C. 1 week
D. on the first day of menstruation 236. GnRH agonist can be used to stop the
bleeding in anovulatory AUB.
227. A regular menstrual cycle is a
progesterone withdrawal effect.  FALSE

“Never design your LIFE like a GARDEN Where anyone can walk, rather Design your LIFE like the SKY Where everyone desires
to REACH”

Special Credits to: | Darna | Totoro | LittleSnoozy | Alma | RatioTeam | RecallsTeam | Astral | SKCCSLAS | Katz
Disclaimer: Some of the items and answers here are not mine but was verified so, If you find discrepancies please feel free to
correct, this serves only as a guide for easy and efficient reviewing.
God bless!
20 | P a g e Philippians 4:13 jpmrnmd
GYNECOLOGY – Mastery Review
“Master your Ultra Instinct”
Ultimate Reviewer ~800 items for Major Exams | Revalida | Board Exams
 TRUE
237. For a woman to menstruate, the
endometrium must be stimulated with 246. NSAIDs may be used to stop
estrogen and progesterone the bleeding in cases of anovulatory
AUB
 FALSE
 TRUE
238. Blood loss of 60 points is
considered menorrhagia.

 FALSE

239. VWD most common cause of


AUB in adolescent

 TRUE

240. 4. In reproductive age group,


the presence of AUB, pregnancy is
considered

 TRUE

241. 5. Infection of the genital tract


as a cause of AUB is most common in
the perimenopausal patients

 FALSE

242. 6. In anovulatory AUB, there is


no uniform sloughing to the basalis layer

 TRUE

243. 7. Ovulatory DUB is usually


presented with menorrhagia

 TRUE

244. 8. Endometrial biopsy must be


done if AUB is seen in an older patient

 TRUE

245. 9. The aim of therapy in


ovulatory Dub is to reduce the amount of
excessive bleeding.

“Never design your LIFE like a GARDEN Where anyone can walk, rather Design your LIFE like the SKY Where everyone desires
to REACH”

Special Credits to: | Darna | Totoro | LittleSnoozy | Alma | RatioTeam | RecallsTeam | Astral | SKCCSLAS | Katz
Disclaimer: Some of the items and answers here are not mine but was verified so, If you find discrepancies please feel free to
correct, this serves only as a guide for easy and efficient reviewing.
God bless!
21 | P a g e Philippians 4:13 jpmrnmd
GYNECOLOGY – Mastery Review
“Master your Ultra Instinct”
Ultimate Reviewer ~800 items for Major Exams | Revalida | Board Exams

247. Optional in examining child


a. Vagina
b. Vulva
c. Rectal

248. Source of light in vaginal examination


a. Perilight
b. Droplight
c. Otoophthalmoscope
d. Flashlight

249. The pH of the vagina of prepubertal child


a. Acidic
b. Alkaline

250. Ratio of cervix to uterus in prepubertal


a. 3:1

“Never design your LIFE like a GARDEN Where anyone can walk, rather Design your LIFE like the SKY Where everyone desires
to REACH”

Special Credits to: | Darna | Totoro | LittleSnoozy | Alma | RatioTeam | RecallsTeam | Astral | SKCCSLAS | Katz
Disclaimer: Some of the items and answers here are not mine but was verified so, If you find discrepancies please feel free to
correct, this serves only as a guide for easy and efficient reviewing.
God bless!
22 | P a g e Philippians 4:13 jpmrnmd
GYNECOLOGY – Mastery Review
“Master your Ultra Instinct”
Ultimate Reviewer ~800 items for Major Exams | Revalida | Board Exams
b. 1:2
c. 2:2
d. 2:1

251. Best method of examining a child who’s 4-5 years old?


a. Frog position
b. Mother's lap
c. Knee chest
d. Lithotomy

Infant

2-
4-
*The methods mentioned above are all part of the 1st aspect of pelvic inspection which is the
evaluation of the external genitalia
*Knee chest position, on the other hand, is a method utilized in the 2nd phase of the examination
which involves evaluation of the vagina
252. Most common prepubertal infection
a. Vulvovaginitis
b. Adhesive vulvitis
c. Lichen sclerosus
d. D. Foreign body

253. Creates “flat appearance” of vulvar surface


a. Vulvovaginitis
b. Adhesive vulvitis

“Never design your LIFE like a GARDEN Where anyone can walk, rather Design your LIFE like the SKY Where everyone desires
to REACH”

Special Credits to: | Darna | Totoro | LittleSnoozy | Alma | RatioTeam | RecallsTeam | Astral | SKCCSLAS | Katz
Disclaimer: Some of the items and answers here are not mine but was verified so, If you find discrepancies please feel free to
correct, this serves only as a guide for easy and efficient reviewing.
God bless!
23 | P a g e Philippians 4:13 jpmrnmd
GYNECOLOGY – Mastery Review
“Master your Ultra Instinct”
Ultimate Reviewer ~800 items for Major Exams | Revalida | Board Exams
c. Lichen sclerosus
d. D. Foreign body

254. Foul, purulent bloody discharge


a. Vulvovaginitis
b. Adhesive vulvitis
c. Lichen sclerosus
d. Foreign body

Classic symptom (Foreign Bodies)


• • Foul, bloody vaginal discharge
• • Discharge is often purulent and maybe without blood

255. Thinning of vulvar epithelium with loss of the rete pegs


a. Vulvovaginitis
b. Adhesive vulvitis
c. Lichen sclerosus
d. D. Foreign body

Lichen Sclerosus (LS)


• • Formerly called lichen sclerosus atrophicus
• • Skin dystrophy
• • Thinning of vulvar epithelium with loss of rete pegs
• • May be associated with autoimmune phenomena
• • Most common symptoms: pruritus and vulvar discomfort

256. Drug of choice for adhesive vulvitis?


a. Antibiotic
b. Estrogen cream
c. Resection
d. D. Laser therapy

According to doc’s lecture, you don’t force it to separate because it will only result to bleeding.
Treatment of choice is topical estrogen. After some time, it will separate by itself.

“Never design your LIFE like a GARDEN Where anyone can walk, rather Design your LIFE like the SKY Where everyone desires
to REACH”

Special Credits to: | Darna | Totoro | LittleSnoozy | Alma | RatioTeam | RecallsTeam | Astral | SKCCSLAS | Katz
Disclaimer: Some of the items and answers here are not mine but was verified so, If you find discrepancies please feel free to
correct, this serves only as a guide for easy and efficient reviewing.
God bless!
24 | P a g e Philippians 4:13 jpmrnmd
GYNECOLOGY – Mastery Review
“Master your Ultra Instinct”
Ultimate Reviewer ~800 items for Major Exams | Revalida | Board Exams
Disadvantage of unwashed specimen in CAUSE SEVERE UTERINE CONTRACTIONS
intrauterine insemination? DUE TO PROSTAGLANDIN RELEASE
a. Cause severe uterine contractions - Sperm must be washed in order to be used in
due to prostaglandin release IUI. Raw semen cannot be inserted directly into
b. Affects sperm motility a woman’s uterus. This is because semen
c. Antisperm antibodies affect success of contains chemical called prostaglandins.
fertility Prostaglandin causes muscular contraction and
d. Affects sperm concentration are responsible fro cramps during menstruation
and pregnancy.
- If raw semen is inserted directly into your
uterus, rather than going through the cervix first,
it could cause severe pain and cramping. It
could also cause your uterus to collapse,
causing severe complications.
- Neat semen, UNWASHED cannot be used
because of a risk of infection or an allergic
reaction. The sperm need to be prepared in the
laboratory either by being washed or by the
swim up method.
- Sperm washing can remove dead sperm and
those with poor motility. This leaves behind
sperm that can swim faster and that are more
likely to fertilize your egg. Sperm washing can
also get rid of the WBC, mucous and seminal
fluid surrounding the sperm which can also
interfere with infertility.

257. Disadvantage of unwashed specimen in intrauterine insemination?


a. Excessive uterine cramps due to prostaglandin release
b. Degrade seminal motility
c. Production of antibody
d. Decrease capacitation

258. Azoospermia is the absence of sperm in the PRETESTICULAR AZOOSPERMIA


semen, the condition where in the hypothalamic - Congenital, acquired and idiopathic etiologies
pituitary axis fails to stimulate spermatogenesis of HYPOGONADOTROPHIC HYPOGONADISM
within the testis is called. (low LH,FSH, Testosterone and prolactin levels)
a. Testicular azoospermia - Gonadotropin-releasing hormone therapy -
b. Pre testicular azoospermia indicated for infertile men with hypothalamic
c. Gonadal failure dysfunction
d. Hypergonadotrophic hypogonadism - Alternative treatment - hCG • Gonadal failure is
the hallmark of Testicular azoospermia.
• Klinefelter syn., Microdeletion of Y
chromosome, radiation therapy, chemotherapy,
testicular torsion, or mumps orchitis, testicular
maldescent
• hypergonadotrophic hypogonadism ( elevated
LH,FSH, Low testosterone)

“Never design your LIFE like a GARDEN Where anyone can walk, rather Design your LIFE like the SKY Where everyone desires
to REACH”

Special Credits to: | Darna | Totoro | LittleSnoozy | Alma | RatioTeam | RecallsTeam | Astral | SKCCSLAS | Katz
Disclaimer: Some of the items and answers here are not mine but was verified so, If you find discrepancies please feel free to
correct, this serves only as a guide for easy and efficient reviewing.
God bless!
25 | P a g e Philippians 4:13 jpmrnmd
GYNECOLOGY – Mastery Review
“Master your Ultra Instinct”
Ultimate Reviewer ~800 items for Major Exams | Revalida | Board Exams
• The hypothalamic-pituitary axis and
spermatogenesis are normal. No sperm appear
in the ejaculate secondary to congenital
absence or obstruction of the vas deferens or
ejaculatory ducts, acquired obstruction of these
ducts, or ductal dysfunctions, including
retrograde ejaculation.
• Confirmed by vasography
• Vasectomy - most common cause of
postesticular azoospermia
• Treatment: Epididymal aspiration,
Microsurgical vasoepididymostomy

Testicular Azoospermia
Post Testicular Azoospermia

259. Approximately azoospermia and severe TRUE


oligospermia (<5mL), microdeletion of Y - two most commonly implicated candidate gene
chromosome can be passed on? families are the RNA binding motif (RBM) and the
deleted in azoospermia (DAZ).
TRUE - these micro deletions in the Y chromosome occur
in 2 % of fertile men, 10 to 20% of men with
idiopathic azoospermia or severe oligospermia and
in 7% infertile men
- These microdeletions can be trans bitted to the
MALE offspring, who may then suffer from infertility.
- Testicular azoospermia is treated by surgical
retrieval of spermatozoa with subsequent
fertilization of the oocyte by Intracytopalsmic Sperm
Injection.

260. Characteristic of a normal semen sample. 35M/mL


a. Poor agglutination - Normal Seminal fluid analysis (WHO) • Volume -
b. 35M/mL concentration >2mL
c. 5% sperm morphology • Sperm Concentration - >20 M/mL
d. 10% progressive • Sperm Motility - 50% progressive, 25% rapidly
e. Volume of 1 mL progressive
• Morphology (strict criteria) > 15% normal forms
• WBC - >1M/mL

261. Normal sperm retains its fertilizing ability 72 hours


for how many hours for the time of ejaculation?
a. 12
b. 24
c. 48
d. 72

“Never design your LIFE like a GARDEN Where anyone can walk, rather Design your LIFE like the SKY Where everyone desires
to REACH”

Special Credits to: | Darna | Totoro | LittleSnoozy | Alma | RatioTeam | RecallsTeam | Astral | SKCCSLAS | Katz
Disclaimer: Some of the items and answers here are not mine but was verified so, If you find discrepancies please feel free to
correct, this serves only as a guide for easy and efficient reviewing.
God bless!
26 | P a g e Philippians 4:13 jpmrnmd
GYNECOLOGY – Mastery Review
“Master your Ultra Instinct”
Ultimate Reviewer ~800 items for Major Exams | Revalida | Board Exams

262. Characteristics of normal seminal fluid All of the Above


analysis:
a. Period of abstinence of 2-3 days
b. Liquefaction that occurs 15-20 mins
after ejaculation
c. Sperm motility declines 2 hrs after
ejaculation
d. All of the above

263. Quantity and Quality of sperm in sperm 3 months


fluid analysis reflects sperm produced __ month - time for the cycle spermatozoa formation to be
earlier. completed
a. 2
b. 3
c. 4
d. 5

264. Cause of abnormal seminal value in Leydig Cell failure - azoospermia


seminal analysis EXCEPT: Retrograde ejaculation - Coital disorder, no
a. Retrograde ejaculation ejaculate ,postesticular azoospermia
b. Absence of vas deferens and seminal Absence of vas deferens and seminal vesicle -low
vesicle volume ejaculate postesticular azoospermia
c. Leydig cell failure Hypogonadal - hypogonadotrophic hypogonadism,
d. Hypogonadal endocrine, pretesticular azoospermia

265. Normal findings in a semen analysis Sperm concentration of 35M/mL


a. Formations of agglutination - Normal Seminal fluid analysis (WHO) • Volume -
b. Sperm concentration of 35M/ mL >2mL
c. 5% morphology • Sperm Concentration - >20 M/mL
d. 10% normal motility • Sperm Motility - 50% progressive, 25% rapidly
e. Volume of 1 mL progressive
• Morphology (strict criteria) > 15% normal forms
• WBC - >1M/mL

266. The diagnosis of cystic fibrosis is highly Congenital Bilateral absence of vas deferens
associated with male infertility, its clinical (CBAVD)
presentation would be? - is found in 1% to 2 % of infertile men and 95% of
a. Obstructive azoospermia men with Cystic Fibrosis, Post Testicular
b. Non-obstructive azoospermia Azoospermia
c. Severe oligospermia
d. Congenital bilateral absence of vas
deferens

“Never design your LIFE like a GARDEN Where anyone can walk, rather Design your LIFE like the SKY Where everyone desires
to REACH”

Special Credits to: | Darna | Totoro | LittleSnoozy | Alma | RatioTeam | RecallsTeam | Astral | SKCCSLAS | Katz
Disclaimer: Some of the items and answers here are not mine but was verified so, If you find discrepancies please feel free to
correct, this serves only as a guide for easy and efficient reviewing.
God bless!
27 | P a g e Philippians 4:13 jpmrnmd
GYNECOLOGY – Mastery Review
“Master your Ultra Instinct”
Ultimate Reviewer ~800 items for Major Exams | Revalida | Board Exams

267. Antiandrogenic drug that is associated with Cimetidine


seminal abnormalities as well as impotence? - as a weak antiandrogen, it has been associated
a. Nitrofurantoin with seminar abnormalities as well as gynecomastia
b. Tetracycline and impotence
c. Cimetidine - Nitrofurantoin - spermatogenic arrest and
d. Ketoconazole decreased sperm counts at high dosage
- Tetracycline- chlortetracycline and minocyline in
particular bind to spermatozoa and interfere with
sperm motility
- Ketoconazole - single oral doses reduce serum
testosterone concentrations although the clinical
importance of this is unknown. Gynecomastia, loss
of libido and impotence have also been reported.

268. Choice of family planning for lactating Progesterone


woman - Progestin-only contraceptives are preferable to
a. Combo OCP estrogen-containing methods if initiated during the
b. Implant first six months after delivery. Progestin only
c. Injectible OCP contraceptives do not appear to affect milk volume,
d. progesterone composition, or to cause deleterious effects in the
infant. Ideally for women who desire a form of
contraception in addition to lactation-induced
amenorrhea, progestin-only methods should be
started at six weeks postpartum if the woman is
fully breastfeeding. Since contraception protection
is provided by lactation amenorrhea, the six week
delay will decrease infant exposure to exogenous
hormones and decrease the incidence of irregular
postpartum bleeding. Milk volume may decrease
with the use of estrogen; however, no detrimental
effects have been shown on infant growth or
development.
- Combo OCP - For women who are planning to
gradually wean their infant, use of COCs may
provide an easier transition to bottle-feeding. COCs
should be used with caution by women who are not
able to obtain supplemental milk.

269. Billings ovulation method Wet days


a. Dry days - Cervical mucus is essential for fertility. It protects
b. Wet days and nourishes the sperm so they retain their
c. fertilizing capacity. It forms channels which help the

“Never design your LIFE like a GARDEN Where anyone can walk, rather Design your LIFE like the SKY Where everyone desires
to REACH”

Special Credits to: | Darna | Totoro | LittleSnoozy | Alma | RatioTeam | RecallsTeam | Astral | SKCCSLAS | Katz
Disclaimer: Some of the items and answers here are not mine but was verified so, If you find discrepancies please feel free to
correct, this serves only as a guide for easy and efficient reviewing.
God bless!
28 | P a g e Philippians 4:13 jpmrnmd
GYNECOLOGY – Mastery Review
“Master your Ultra Instinct”
Ultimate Reviewer ~800 items for Major Exams | Revalida | Board Exams
d. d. 4 days after peak days sperm travel through your reproductive system to
meet and fertilize the egg. And it acts as a filter,
destroying imperfect sperm

270. Procedures used in female sterilization Tubal ligation


-Your doctor will inflate your abdomen with gas
and make a small incision to access your
reproductive organs with the laparoscope. Then
they will seal your fallopian tubes. The doctor may
do this by:
• cutting and folding the tubes
• removing sections of the tubes
• blocking the tubes with bands or clips
Tubal implant- A very small spring-like coil is
placed into each fallopian tube. The coils cause
scar tissue to form in the tubes, blocking the
tubes. This method does not involve cuts or
incisions. Instead, a healthcare provider uses a
thin tube to thread the small coils through the
vagina and uterus into the fallopian tubes, where
the coils will remain.
Hysterosalpingography
271. Type of contraception wherein the male Pull out method
withdraws
272. Male factor in infertility, what result of Mumps Orchitis
abnormality? (something ganito ung thought ng Mumps orchitis rarely leads to sterility, but it may
question, d ko maaalala) antibody formation contribute to subfertility. It can lead to
a. Kallmann oligospermia, azoospermia, and asthenospermia
b. Kartgener (defects in sperm movement). Unilateral orchitis
c. Mumps orchitis can drastically, but only briefly diminish the sperm
count and alter the mobility, and morphology of
the sperm. Impairment of fertility is estimated to
occur in about 13% of patients while 30–87% of
patients with bilateral mumps orchitis experience
infertility.The link between mumps orchitis and
anti-sperm antibodies has been unclear while the
antibodies are suspected to impair fertility.
- Kallmann syndrome is a condition characterized
by delayed or absent puberty and an impaired
sense of smell.
This disorder is a form of hypogonadotropic
hypogonadism, which is a condition resulting from
a lack of production of certain hormones that
direct sexual development. These hormones are
normally made in a part of the brain called the
hypothalamus. Males born with hypogonadotropic
hypogonadism often have an unusually small
penis (micropenis) and undescended testes

“Never design your LIFE like a GARDEN Where anyone can walk, rather Design your LIFE like the SKY Where everyone desires
to REACH”

Special Credits to: | Darna | Totoro | LittleSnoozy | Alma | RatioTeam | RecallsTeam | Astral | SKCCSLAS | Katz
Disclaimer: Some of the items and answers here are not mine but was verified so, If you find discrepancies please feel free to
correct, this serves only as a guide for easy and efficient reviewing.
God bless!
29 | P a g e Philippians 4:13 jpmrnmd
GYNECOLOGY – Mastery Review
“Master your Ultra Instinct”
Ultimate Reviewer ~800 items for Major Exams | Revalida | Board Exams
(cryptorchidism). At puberty, most affected
individuals do not develop secondary sex
characteristics, such as the growth of facial hair
and deepening of the voice in males, the start of
monthly periods (menstruation) and breast
development in females, and a growth spurt in
both sexes. Without treatment, most affected men
and women are unable to have biological children
(infertile).

273. The following are parameters for normal


semen analysis, except: - Normal Seminal fluid analysis (WHO) • Volume -
a. Volume 2-5ml >2mL
b. Liquefaction: >60%? • Sperm Concentration - >20 M/mL
c. 1st hour: >60% 2nd: >50% • Sperm Motility - 50% progressive, 25% rapidly
d. Morphology: 50% progressive
• Morphology (strict criteria) > 15% normal forms

274. If abnormal semenalysis, when to repeat: 4-6 weeks


a. 1-2 weeks -time to produce new sperm (journal)
b. 3-4 weeks * 75 days = 10.7 weeks (handout)

275.No sperm on ejaculation secondary to Post testicular Azoospermia


congenital absence or obstruction of vas - CBAVD
deferens…is known as. - acquired obstruction of ducts
a. Testicular azoospermia - Ductal dysfunction
b. Pre testicular azoospermia - Retrograde ejaculation
c. Post testicular azoospermia
d. Gonadal failure Pretesticular azoospermia - hypogonadotrophic
hypogonadism
276. Male infertility is considered an inherited Congenital Bilateral absence of vas deferens
disorder in infertile couple. Congenital absent (CBAVD)
or atrophy of vas deferens accounts how many - is found in 1% to 2 % of infertile men and 95% of
percentage in all male infertility. men with Cystic Fibrosis, Post Testicular
a. 1 Azoospermia
b. 2
c. 3
d. 5

277. Intrauterine Insemination is used treat Sperm Motility


oligospermia. It is limited by decrease in the - a retrospective analysis of 1841 couples
number of sperm and __. undergoing 4,056 cycles of IUI for male factor
a. Sperm motility infertility found that pregnancy rates were related to
b. Poor sperm morphology total motile sperm count.

“Never design your LIFE like a GARDEN Where anyone can walk, rather Design your LIFE like the SKY Where everyone desires
to REACH”

Special Credits to: | Darna | Totoro | LittleSnoozy | Alma | RatioTeam | RecallsTeam | Astral | SKCCSLAS | Katz
Disclaimer: Some of the items and answers here are not mine but was verified so, If you find discrepancies please feel free to
correct, this serves only as a guide for easy and efficient reviewing.
God bless!
30 | P a g e Philippians 4:13 jpmrnmd
GYNECOLOGY – Mastery Review
“Master your Ultra Instinct”
Ultimate Reviewer ~800 items for Major Exams | Revalida | Board Exams
c. Anti sperm antibody - Placement of about 0.3 to 0.5 ml of washed
d. A and B processed and concentrated sperm
e. A, B, and C

288. Increased in women who use diaphragm TSS


contraceptive Diaphragm is a soft rubber dome shaped barrier that
a. Toxic shock syndrome covers the cervix in order to prevent sperm from
b. UTI entering into the uterus.Spermicide is added to the
c. STD diaphragm to kill sperm that may get around the
d. Cervical Neoplasm barrier. The most common side effect is vaginal
irritation. Latex allergies, history of TSS or
irregularities of the vagina or cervix could create
additional risks.
289. Which of the following is an advantage of Offers greater protection from STD
female condom over male condom Disadvantage
a. Ease of application Perhaps the biggest disadvantage of female
b. Ease of removal condoms is that they have to be inserted into the
c. Offers greater protection from vagina. For women or their partners who are not
STD familiar with their anatomy, this can prove
d. Feel of condom during intercourse. challenging or uncomfortable. Many woman also
worry if they have the condom inserted correctly.
According to Planned Parenthood, it's also possible
for the condom to slip all the way inside of the
vagina.

290. All reported series of pregnancies with Spontaneous abortion


any type of IUD in situ include an increase in
a. Spontaneous abortion
b. Congenital abnormalities
c. Septic abortion
d. Postmaturity

291. Efficiency of sterilization is greatest in Female with 4 children


what demographic
a. Female <30 years old
b. 25 y/o female who did not wish to
get pregnant
c. Couple married for >10 years
d. Female with 4 children

292. Which is a late sequelae tubal Increase menstrual irregularities and pain
sterilization? - The most common late sequelae/outcomes of
a. Increase risk ovarian cancer female sterilization are hormonal imbalance, loss of
b. Decrease sexual libido ovarian function, pelvic pain, and menorrhea
c. Increase menstrual irregularities (hypermenorrhea). Hypermenorrhea is defined as
and pain heavy bleeding, a menstrual period that lasts longer
d. Increase risk PID than 10 days, menstrual flow that includes large

“Never design your LIFE like a GARDEN Where anyone can walk, rather Design your LIFE like the SKY Where everyone desires
to REACH”

Special Credits to: | Darna | Totoro | LittleSnoozy | Alma | RatioTeam | RecallsTeam | Astral | SKCCSLAS | Katz
Disclaimer: Some of the items and answers here are not mine but was verified so, If you find discrepancies please feel free to
correct, this serves only as a guide for easy and efficient reviewing.
God bless!
31 | P a g e Philippians 4:13 jpmrnmd
GYNECOLOGY – Mastery Review
“Master your Ultra Instinct”
Ultimate Reviewer ~800 items for Major Exams | Revalida | Board Exams
blood clots (and that is not your norm), and heavy
periods that interfere with your regular lifestyle.
Women experiencing pelvic pain and menorrhea are
often suggested/undergo hysterectomy

293. The most common cause for infertility MALE FACTOR


a. Tubalobstruction - Relative Prevalence of etiologies of Fertility (%) •
b. Male factor Male factor
c. Hyperthyroidism • Female factor
d. Hyperandrogens • Both
• Unexplained fertility

- Increasing factor of male due to vices, exposure to


environmental factors
294. The initial evaluation in an infertile couple SEMEN ANALYSIS
should include: - initial evaluation, history, physical exam • Irreg
a. Ovarian biopsy menses, no ovulation by tests
b. Semen analysis • HSG
c. Dilatation and curettage • Structural, Hysteroscopy
d. Diagnostic laparoscopy • Abnormal semen analysis

295. Which of the following is basic SEMEN ANALYSIS


investigation for the male infertility
a. Semen analysis
b. Sperm penetration assay of cervical
mucous
c. Sperm penetration assay of hamster
ova
d. Sperm antibodies test

296. According to WHO, sperm count 20M/mL


shouldn’t be less than:
a. 50M/mL
b. 60M/mL
c. 90M/mL
d. 20M/mL

297. Sims Hunter test is POST COITAL TEST


a. Useful index of tubal patency - cervical factor - assess quality of cervical mucus,
b. Post coital test the presence and number of motile sperm in the
c. Means of determining the number of female reproductive tract and its interaction between
abnormal sperm morphology cervical mucuc and sperm.
d. Means of looking into the uterine - Hyterosalpingography - tubal latency
cavity

“Never design your LIFE like a GARDEN Where anyone can walk, rather Design your LIFE like the SKY Where everyone desires
to REACH”

Special Credits to: | Darna | Totoro | LittleSnoozy | Alma | RatioTeam | RecallsTeam | Astral | SKCCSLAS | Katz
Disclaimer: Some of the items and answers here are not mine but was verified so, If you find discrepancies please feel free to
correct, this serves only as a guide for easy and efficient reviewing.
God bless!
32 | P a g e Philippians 4:13 jpmrnmd
GYNECOLOGY – Mastery Review
“Master your Ultra Instinct”
Ultimate Reviewer ~800 items for Major Exams | Revalida | Board Exams
- Laparoscopy visualization of all pelvic organs and
permits detection of intramural and subserosal
uterine fibroids, peritubal and periovarian adhesions
and endometriosis.

298. Greater than normal incidence of sperm ASTHENOSPERMIA- reduced sperm motility
with decrease motility is - Azoospermia - no sperm in semen
a. Azoospermia - Oligospermia - reduced sperm numbers - Mild to
b. Asthenospermia mod ; 5-20 M/mL
c. Oligospermia - Severe: <5M/mL
d. Normospermia
- Normozoospermia- all semen parameters are
normal
- Teratozoospermia - increased abnormal forms of
sperm
- Oligoasthenoteratozoospermia - sperm variables all
subnormal
- Aspermia - no ejaculate
- Leucocytospermia - increased white cells in semen
- Necroozoospermia- all sperm are non viable or non
motile

299.Infertility is considered primary if, G0P0


a. G1 P0 (0010) - no pregnancy in 12 months of unprotected
b. G0 P0 intercourse
c. G1 P1 (1001)
d. G2 P2

300. Presence of fewer than 20 million Sperm Oligospermia - reduced sperm numbers
.mL of semen - Azoospermia - no sperm in semen
a. Oligospermia - Normozoospermia- all semen parameters are
b. Asthenospermia normal
c. Normospermia - Teratozoospermia - increased abnormal forms of
d. Teratospermia - Mild to mod ; 5-20 sperm
M/mL - Oligoasthenoteratozoospermia - sperm variables all
- Severe: <5M/mL subnormal
- Aspermia - no ejaculate
- Leucocytospermia - increased white cells in semen
- Necroozoospermia- all sperm are non viable or non
motile

301. Possible cause of azoospermia is HYPOGONADOTROPHIC HYPOGONADISM


a. Hypogonadotrophic - pretesticular azoospermia
hypogonadism
b. Maturation Arrest
c. Hypospermatogenesis
d. Metabolic abonormalities of the
sperm

“Never design your LIFE like a GARDEN Where anyone can walk, rather Design your LIFE like the SKY Where everyone desires
to REACH”

Special Credits to: | Darna | Totoro | LittleSnoozy | Alma | RatioTeam | RecallsTeam | Astral | SKCCSLAS | Katz
Disclaimer: Some of the items and answers here are not mine but was verified so, If you find discrepancies please feel free to
correct, this serves only as a guide for easy and efficient reviewing.
God bless!
33 | P a g e Philippians 4:13 jpmrnmd
GYNECOLOGY – Mastery Review
“Master your Ultra Instinct”
Ultimate Reviewer ~800 items for Major Exams | Revalida | Board Exams

302. The following value of semen analysis VOLUME TEST 1ML


indicates abnormal semen quality: - WHO - Volume ; >2mL
a. Volume test 1 mL - Concentration: > 20 M/mL
b. Sperm Count of >40M/mL - Motility : >50% progressive, >25% rapidly
c. Motility >60% progressive
d. Liquefaction complete in 3 mins - Morphology (strict): >15 normal form
- WBC; >1M/mL
- Liquefaction : after 15-20 mins (RCJ), 20-30 mins
(NER)

303. Main symptoms of hyperprolactinemia are galactorrhea and amenorrhea


 Ans. TRUE
304. Pathologic causes of hyperprolactinemia due to pituitary tumors, includes prolactinoma cushing
syndrome
 Ans. TRUE
305. Prolactinoma are present in approximately 10% of the population
 Ans. TRUE
***Typo.error lang ata ung 1.0% kasi sa samplex ng friend ko binago niya yung question into 10%
306. Prolactin is synthesized by the decidualized stroma of the endometrium
 Ans. TRUE
307. Diazepam, Narcotics, Anti-hypertensive drugs are pharmacologic agents that cause
hyperprolactinemia
 Ans. TRUE
308. Women who have hyperprolactinemia, galactorrhea, and amenorrhea can have prolactinomas
despite low levels of estrogen.
 Ans. TRUE
309. Hyperprolactinemia can occur in individuals with primary hypothyroidism
 Ans. TRUE
310. Estrogen replacement therapy or oral contraceptives can be used for treatment of
hyperprolactinemia and hypoestogenism
 Ans. TRUE
311. Most microadenoma enlarge with time while almost at macroadenoma do not.
 Ans. FALSE
312. Serotonine is principal prolactin inhibitor
 Ans. FALSE

313. A 34 y/o G1P1, complained of galatorrhea for 8 months. Medication must ask except:
A. Antihyperyensive
B. OCP
C. Tricyclic antidepressant
D. Tranquilizer

“Never design your LIFE like a GARDEN Where anyone can walk, rather Design your LIFE like the SKY Where everyone desires
to REACH”

Special Credits to: | Darna | Totoro | LittleSnoozy | Alma | RatioTeam | RecallsTeam | Astral | SKCCSLAS | Katz
Disclaimer: Some of the items and answers here are not mine but was verified so, If you find discrepancies please feel free to
correct, this serves only as a guide for easy and efficient reviewing.
God bless!
34 | P a g e Philippians 4:13 jpmrnmd
GYNECOLOGY – Mastery Review
“Master your Ultra Instinct”
Ultimate Reviewer ~800 items for Major Exams | Revalida | Board Exams
314. Diagnostic test for hyperprolactinemia include:
A. urine test
B. Liver function test
C. thyroid function test
D. cardio 2d echo
315: Dopa receptor agonist NOT used in Dopamine agonist, bromocriptine mesylate, is
hyperprolactinemia: often the initial drug of choice and may require
A. Bromocriptine high doses to achieve clinical improvement and
B. Cabergoline shrinkage of prolactinomas.
C. OCP Cabergoline is more effective and causes fewer
D. Pergolide adverse effects than bromocriptine. However, it is
much more expensive. It is often used in patients
who cannot tolerate the adverse effects of
bromocriptine or in those who do not respond to
bromocriptine.
Oral contraceptives are hormonal preparations
that may contain combinations of the hormones
estrogen and progestin or progestin alone.
Pergolide a drug previously used for the
treatment of hyperprolactinemia was withdrawn
from the US market March 29, 2007, because of
heart valve damage resulting in cardiac valve
regurgitation
MATCHING TYPE
316. A soft saucer-shaped device made from polyurethane foam containing nanoxynol-9
D. Sponge
317. This consist of an active agent and a carrier
B. Spermicides
318. Thin dome-shaped membrane of latex rubber or silicone with a flexible spring modeled into the
rim
E. Diaphragm
319. A cup shaped silicone or rubber device that fits around the cervix
A. Cervical cap
320. Consist of a soft, loose-fitting polyurethane with 2 flexible rings
C. Female condom
321. In lactating mothers, the following form of Method of avoiding pregnancy based on natural
family planning can be advised, EXCEPT: post partum infertility
a. Combination OCP Full lactation with no regular supplemental feeding
b. Implants not even water!
c. Injectable contraceptives
d. Progestin only OCPs
322. In billing’s or ovulation method, unprotected After menstruation (dry days)
intercourse may be allowed during: 4 days after peak wet days
a. Wet days
b. Dry days
c. 4th day after the peak day
d. All of the above
e. B & C only

“Never design your LIFE like a GARDEN Where anyone can walk, rather Design your LIFE like the SKY Where everyone desires
to REACH”

Special Credits to: | Darna | Totoro | LittleSnoozy | Alma | RatioTeam | RecallsTeam | Astral | SKCCSLAS | Katz
Disclaimer: Some of the items and answers here are not mine but was verified so, If you find discrepancies please feel free to
correct, this serves only as a guide for easy and efficient reviewing.
God bless!
35 | P a g e Philippians 4:13 jpmrnmd
GYNECOLOGY – Mastery Review
“Master your Ultra Instinct”
Ultimate Reviewer ~800 items for Major Exams | Revalida | Board Exams
323. The following procedures are used for Hysterectomy answer (ayaw maedit huhu
female sterilization, EXCEPT: Other commonly used procedures are:
a. Hysteroscopy Tubal sterilization at time of laparotomy for CS
b. Laparoscopy operation
c. Hysterectomy Postpartum minilaparotomy soon after vaginal
d. Interval minilaparatomy delivery
324. Withdrawal of the penis from the vagina Failure rate 6.7 per 100 woman years
before ejaculation is known as: Rhythm method calculates length of individual’s
a. Coitus interruptus woman previous mens
b. Rhythm method Billing method must abstain from intercourse from
c. Billing’s method onset of intercourse to cervical mucus symptoms
d. Symptothermal method Symptothermal method involves cervical
secretions change sin BBT and own calendar
calculations
325. Mechanism of action of intrauterine device: Prostaglandin formation
a. Prevents fertilization of ovum Endometrial atrophy
b. Induces local inflammation of sperm Local inflammation reaction
c. Captures and holds the seminal fluid Cellular and Humoral components expressed in
d. Blocks the entry of the sperm tissue and fluid fill the uterine cavity

326. In reproductive process w/c is TRUE?


A. GNRH…
B. GNRH regulate FSH and TSH only
C. LH regulate growth and development of ovarian follicle
D. FSH influence the ovary to secrete progesterone in luteal phase
327. In the first half of ovarian cycle, estradiol 17 beta is secreted by increasing w/c of the ff.:
A. theca cells of endometrial stroma
B. granulosa cells of the dominant follicle
C. residual…
D. AOTA
328. In the second part of the ovarian cycle what hormone is produced by what structure?
A. Estrogen, by the corpus luteum
B. Inhibin, by the…
C. Progesterone, in high amount of corpus luteum
D. AOTA
E. A & C
329. What is the histologic features/characteristics of proliferative phase of menstrual cycle?
A. Appearance of subnuclear vacuoles
B. increase stromal edema
C. Progressive proliferation

330 The long feedback loop associated with:


A. GnRH
B. FSH
C. Estrogen
D. AOTA

“Never design your LIFE like a GARDEN Where anyone can walk, rather Design your LIFE like the SKY Where everyone desires
to REACH”

Special Credits to: | Darna | Totoro | LittleSnoozy | Alma | RatioTeam | RecallsTeam | Astral | SKCCSLAS | Katz
Disclaimer: Some of the items and answers here are not mine but was verified so, If you find discrepancies please feel free to
correct, this serves only as a guide for easy and efficient reviewing.
God bless!
36 | P a g e Philippians 4:13 jpmrnmd
GYNECOLOGY – Mastery Review
“Master your Ultra Instinct”
Ultimate Reviewer ~800 items for Major Exams | Revalida | Board Exams
331. Secretions of the pituitary that is produced by gland:
A. FSH
B. Prolactin
C. TSH
D. Vasopressin
332. The frequency and amplitude of GnRH secretion is during the:
A. Early follicular stage
B. Late follicular stage
C. both
D. neither
333. An initial release of gonadotrophins is expected:
A. GnRH agonist
B. GnRH antagonist
C. both
D. neither
334. Structurally FSH is related to:
A. TSH
B. Prolactin
C. LH
D. HCG
335. Prolactin release is associated with:
A. Drugs
B. Stress
C. Exercise
D. AOTA

336. Initial follicular development is primarily due to:


A. Transforming growth factors
B. FSH
C. LH
D. AOTA
337. FSH effects include:
A. increase LH receptors in the granulosa cells
B. increase aromataze enzyme for increase estradiol secretion
C. Expand the oocyte
D. AOTA
338. Estradiol is primarily secreted by the:
A. Granulosa cells
B. Theca cells
C. both
D. neither
339. What secretory cells of the anterior pituitary Acidophilic - prolactin and growth hormone
secrete GH and prolactin? Basophil - secretes the GNRH
a. Basophilic
b. Acidophilic
c. Neutral staining chomophobes
d. All of the above

“Never design your LIFE like a GARDEN Where anyone can walk, rather Design your LIFE like the SKY Where everyone desires
to REACH”

Special Credits to: | Darna | Totoro | LittleSnoozy | Alma | RatioTeam | RecallsTeam | Astral | SKCCSLAS | Katz
Disclaimer: Some of the items and answers here are not mine but was verified so, If you find discrepancies please feel free to
correct, this serves only as a guide for easy and efficient reviewing.
God bless!
37 | P a g e Philippians 4:13 jpmrnmd
GYNECOLOGY – Mastery Review
“Master your Ultra Instinct”
Ultimate Reviewer ~800 items for Major Exams | Revalida | Board Exams
340. What level of feedback to the hypothalamus Short - from pituitary hormones
is composed of endocrine input from circulating Ultra short - from hypothalamic secretions
hormones? Long - from circulating hormones
a. Short
b. Ultra short
c. Long

341. What make/s the GNRH unique? Hypothalamic gonadotropin- releasing hormone
a. It must be secreted in a pulsatile (GNRH) simultaneously regulates both luteinizing
fashion to be effective. hormone (LH) and follicle-stimulating hormone
b. It regulates LH and FSH (FSH) in the pituitary, and does so by being
simultaneously secreted in a pulsatile manner. The pulse
c. The pulsatile release of GNRH frequency determines the relative amounts of LH
influences the release of LH and FSH and FSH secretions.
d. All of the above
e. A and B

342. Continual exposure of the pituitary GNRH agonists - leads to persistent activation of
gonadotroph to GNRH results in a phenomenon GNRH receptor -> down regulation and decrease
called? in GNRH Rc.
a. “Upregulation” an increase of GNRH
receptor
b. “Down regulation” and decrease of
GNRH receptor

343. Which of the 3 major classes of endogenous Endorphins - are named for their endogenous
opiods produces behavioral effects and exhibit a morphinelike activity. These substances are
high analgesic potency? produced in the hypothalamus from the precursor
a. Enkephalins proopiomelanocortin (POMC) and have diverse
b. Dynorphins activities, including regulation of temperature,
c. Endorphins appetite, mood and behavior.
d. All of the above Enkephalins - are the most widely distributed
opiod peptides in the brain, and they function
primarily in regulation of the autonomic nervous
system. Proenkephlain A is the precursor for the
two enkephalins of primary importance:
methionine-enkephalins and leucine-enkephalin.
Dynorphins - are endogenous opioids produced
from the precursor of proenkephalin B that serve
a function similar to that of the endorphins,
producing behavioral effects and exhibiting a high
analgesic potency.

“Never design your LIFE like a GARDEN Where anyone can walk, rather Design your LIFE like the SKY Where everyone desires
to REACH”

Special Credits to: | Darna | Totoro | LittleSnoozy | Alma | RatioTeam | RecallsTeam | Astral | SKCCSLAS | Katz
Disclaimer: Some of the items and answers here are not mine but was verified so, If you find discrepancies please feel free to
correct, this serves only as a guide for easy and efficient reviewing.
God bless!
38 | P a g e Philippians 4:13 jpmrnmd
GYNECOLOGY – Mastery Review
“Master your Ultra Instinct”
Ultimate Reviewer ~800 items for Major Exams | Revalida | Board Exams
344. Which of the following is/are true about The gonadotropins FSH and LH are produced by
gonadotrophs? the anterior pituitary gonadotroph cells and are
a. LH and FSH are responsible for responsible for ovarian follicular stimulation.
ovarian follicular stimulation Structurally, there is great similarity between FSH
b. Structurally, LH and FSH have the and LH. They are both glycoproteins that share
same B subunit identical a subunit and differ only in the structure
c. Thyroid stimulating hormone and of their B subunit, which confer receptor
placental HCG also share identical B subunits specificity.
d. All of the above Thyroid-stimulating hormone and placental human
chorionic gonadotropin (HCG) also share identical
a subunits with the gonadotropins.
345. During the luteal phase of the ovarian cycle, After ovulation, the remaining follicular shell is
what is the main physiological change? transformed into the primary regulator of the luteal
a. Corpus luteum formation phase: the corpus luteum.
b. Folliculogenesis
c. Regression of corpus luteum
d. Ovulation
e. All of the above

346. What hormone/s is/are produced during the Luteal phase - from ovulation to the onset of
luteal phase? menses has an average length of 14 days.
a. Estrogen The presence of estradiol and progesterone in the
b. Progesterone luteal phase results in negative feedback on both
c. Both FSH and LH secretions. Because of this negative
d. Neither feedback, the levels of FSH and LH are relatively
low in the luteal phase.
347. What layer/s of the endometrium is shed In the absence of implantation, glandular
during menstruation? secretion ceases, and an irregualr breakdown of
a. Basalis the decidua functionalis occurs. The resultant
b. Functionalis shedding of this layer of the endometrium is
c. Both termed menses. The destruction of the corpus
d. Neither luteum and its production of estrogen and
preogesterone is the presumed cause of the
shedding.
There is a profound spiral artery vascular spasm
that leads to endometrial ischemia,
simultaneously, there is a breakdown of
lysosomes and a release of proeolytic enzymes,
which further promote local tissue destruction
leaving the decidua basalis as the source of
subsequent endometrial growth.
348. What is/are the main change/s in the The proliferative phase is characterized by
endometrium during the proliferative phase of the progressive mitotic growth of the decidua
menstrual cycle? functionalis in preparation for implantation of the
a. Leucocytic infiltration embryo in response to rising circulating levels of
b. Glandular proliferation and mitosis estrogen.
c. Glandular secretions
d. Corpus luteum regression

“Never design your LIFE like a GARDEN Where anyone can walk, rather Design your LIFE like the SKY Where everyone desires
to REACH”

Special Credits to: | Darna | Totoro | LittleSnoozy | Alma | RatioTeam | RecallsTeam | Astral | SKCCSLAS | Katz
Disclaimer: Some of the items and answers here are not mine but was verified so, If you find discrepancies please feel free to
correct, this serves only as a guide for easy and efficient reviewing.
God bless!
39 | P a g e Philippians 4:13 jpmrnmd
GYNECOLOGY – Mastery Review
“Master your Ultra Instinct”
Ultimate Reviewer ~800 items for Major Exams | Revalida | Board Exams
The predominant change seen during this time is
evolution of the initially straight, narrow, and short
endometrial glands into longer, tortous stuctures.
From a low columnar pattern to a pseudostratified
pattern before ovulation.

349. Inadequate or improperly timed endometrial LUTEAL PHASE INSUFFICIENCY


development at potential implantation site ● Also called Luteal Phase Defects (LPD)
A. Luteal phase defect ● Characterized by inadequate luteal milestones
B. DM ● Inadequate or improperly timed endometrial
C. Hyperprolactinemia development at potential implantation sites.
D. Hypothyroidism *Meaning on the day of ovulation there will be
released of eggs then the egg meets with the
sperm causing fertilization. From the day of
ovulation that was fertilized, they will need one
week to traverse or to go back to the endometrial
cavity to implant as blastocyst. The endometrium
must be ready to implant the blastocyst but
sometimes it will not happen because of luteal
phase insufficiency. There is a delay of the
development of the endometrium which the
endometrium is still not ready for the implantation
of the blastocyst.
● Associated with hypersecretion of LH
● Abnormal LH secretion may have direct effects
on the developing oocyte (premature aging), on
the endometrium (dyssynchronous maturation), or
both

350. Most common single anembryonic Monosomy (45 X)


conceptuses ● The most common single chromosomal
A. Monosomy abnormality especially among anembryonic
B. Single gene defect conceptuses.
C. Balance translocation
D. Trisomy
351. In post-conception evaluation, serial serum Serial Serum B-Hcg Levels
BhCG is performed until ● From the time of missed menstrual period until
A. 500 the level is approximately 1,200 to 1,500 mlU/mL
B. 1000 ● In this level, you can see pregnancy through
C. 1200 ultrasound
D. 200,000
352. Placental abnormalities highly associated with reproductive failure
A. Circumvallate placenta
B. Apoptosis
C. Placenta previa
D. AOTA
353. Alternative approaches for couples who have robertsonian translocation Sperm/oocyte donor

“Never design your LIFE like a GARDEN Where anyone can walk, rather Design your LIFE like the SKY Where everyone desires
to REACH”

Special Credits to: | Darna | Totoro | LittleSnoozy | Alma | RatioTeam | RecallsTeam | Astral | SKCCSLAS | Katz
Disclaimer: Some of the items and answers here are not mine but was verified so, If you find discrepancies please feel free to
correct, this serves only as a guide for easy and efficient reviewing.
God bless!
40 | P a g e Philippians 4:13 jpmrnmd
GYNECOLOGY – Mastery Review
“Master your Ultra Instinct”
Ultimate Reviewer ~800 items for Major Exams | Revalida | Board Exams
Hysterotomy of uterus Antithrombin therapy
354. When equal segments of chromosomal Reciprocal Translocation – healthy individuals but
material are swapped – such that no has 2 chromosomes that exchanges genetic
chromosomal material has been lost or gained A. materials. There is no loss or gain of genetic
Insertion materials, just a reciprocal translocation.
B. Balance translocation
C. Deletion
D. A&C
355. Carriers of balanced translocation are Balanced Chromosomal Translocation – patient is
healthy individuals, suspicion of chromosomal usually normal. The translocation can happen in
abnormalities: any of the chromosomes. They are genetically
A. Positive test for lupus anticoagulant looking normal but they are carrier of balanced
B. Positive test for anticardiolipin chromosomal translocation if they have history of
C. Recurrent abortion the following:
D. All of the above a) Infertility
b) Recurrent Miscarriage
c) Child with birth defects and/or learning
difficulties

356. One or more preterm birth at less than 34 wks AOG secondary to severe preeclampsia or
placental insufficiency
A. Hypothyroidism
B. Hyperprolactinemia
C. Balanced translocation
D. APAS
357. Infertility History of subfertility or infertility – inability to
A. After at least 5 years of unprotected conceive after 12 months of unprotected
intercourse intercourse
B. After 1 year of unprotected intercourse
C. At least 5 months of unprotected intercourse
D. Undetermined number of coitus
358. Treatment of recurrent pregnancy loss secondary to thrombophilic disorder includes:
A. Unfractioned heparin
B. Low molecular wt heparin
C. Coumadin
D. aspirin

359. Androgen resistance ANDROGEN RESISTANCE (Testicular


A. (-) B, (+) U Feminization) → Rarest cause of primary
B. (+) B, (-) U amenorrhea
C. (-) B, (-) U
D. (+) B, (+) U

360. Agonadism
A. (-) B, (+) U
B. +) B, (-) U
C. (-) B, (-) U

“Never design your LIFE like a GARDEN Where anyone can walk, rather Design your LIFE like the SKY Where everyone desires
to REACH”

Special Credits to: | Darna | Totoro | LittleSnoozy | Alma | RatioTeam | RecallsTeam | Astral | SKCCSLAS | Katz
Disclaimer: Some of the items and answers here are not mine but was verified so, If you find discrepancies please feel free to
correct, this serves only as a guide for easy and efficient reviewing.
God bless!
41 | P a g e Philippians 4:13 jpmrnmd
GYNECOLOGY – Mastery Review
“Master your Ultra Instinct”
Ultimate Reviewer ~800 items for Major Exams | Revalida | Board Exams
D. (+) B, (+) U

361. Turner’s syndrome


A. (-) B, (+) U
B. (+) B, (-) U
C. (-) B, (-) U
D. (+) B, (+) U

362. Imperforate hymen Imperforate hymen is associated with Mullerian


A. (-) B, (+) U agenesis also known as Mayer-Rokitansky-Kuster-
B. (+) B, (-) U Hauser (MRKH) syndrome
C. (-) B, (-) U
D. (+) B, (+) U

363. Craniopharyngioma
A. (-) B, (+) U
B. (+) B, (-) U
C. (-) B, (-) U
D. (+) B, (+) U

364. The difference between mullerian and androgen insensitivity


A. FSH level
B. LH
C. Testosterone
D. Progesterone

365. Which of the following has a primary GONADAL DYSGENESIS


ovarian failure? ● Hypergonadotropic hypogonadism (hyper-hypo)
A. Gonadal dysgenesis → ovarian failure
B. Kallman syndrome
C. Craniopharygioma
D. Mullerian agenesis

366. Androgen insufficiency Androgen insensitivity, yata ito? Kasi sa androgen


A. Presence of female ovaries insensitivity you have:
B. Presence of male internal genitalia ● Testes
C. Absence of pubic/axilla hair ● Absent axillary & pubic hair
D. Absence of breast development
Kindly refer to the table at #6 na lang :)
367. What is the most common cause of primary Most common cause → Gonadal failure
amenorrhea? 2nd most common → Congenital absence of
A. Uterine agenesis uterus/Meyer-Rokitansky-Kuster-Hauer (MRKH)
B. Testicular feminization syndrome
C. Polycystic ovaries Rarest cause → Androgen resistance/Testicular
D. Gonadal failure feminization

“Never design your LIFE like a GARDEN Where anyone can walk, rather Design your LIFE like the SKY Where everyone desires
to REACH”

Special Credits to: | Darna | Totoro | LittleSnoozy | Alma | RatioTeam | RecallsTeam | Astral | SKCCSLAS | Katz
Disclaimer: Some of the items and answers here are not mine but was verified so, If you find discrepancies please feel free to
correct, this serves only as a guide for easy and efficient reviewing.
God bless!
42 | P a g e Philippians 4:13 jpmrnmd
GYNECOLOGY – Mastery Review
“Master your Ultra Instinct”
Ultimate Reviewer ~800 items for Major Exams | Revalida | Board Exams
368. Which of the following is associated with Turner’s syndrome
A. Gonadectomy is done after puberty
B. McIndoe procedure for sexually active female
C. Prepubertal
D. Congenital

369. Give the histologic Criteria for the diagnosis Histopathologic criteria of at least plasma cell/ x
of endometritis 120 field of endometrial
stroma combined w/ ≥ 5 neurtophils in the
superficial endometrial epithelium / x 400 field
370. Give the minimum criteria for the diagnosis Minimum Criteria:
of PID ● Lower Abdominal tenderness
a. Uterine tenderness ● Adnexal tenderness
b. Adnexal tenderness ● Cervical Motion Tenderness
c. Cervical motion tenderness

371. Give atleast 1 diagnostic criterion for PID Definitive Criteria for Diagnosing PID
a. Endometrial biopsy with ● Histopathologic evidence of endometritis on
histopathologic evidence of endometritis endometrial biopsy
b. Transvaginal US or MRI showing ● Transvaginal Sonography or MRI showing
thick fluid-filled tubes thickened- fluid filled tubes with or without free
c. ...uptake?!.. Abnormalities consistent pelvic fluid or tubo-ovarian complex
with PID ● Laparoscopic abnormalities consistent with PID

372. The Fitz-Hugh-Cutis Syndrome is found in Women with antibodies to chlamydial heat shock
women who develop antibodies to protein are more likely to develop tubal scarring
and fitz-hugh-curtis syndrome
373. Which of the following are recommended for initial treatment for regiment of PID

374. What is the most common cause of PID Colonization of bacterial flora found at the lower
genital tract
375. Actinomyces Israelii is more commonly IUD for more than 8 years
associated with women using what type of
contraception and for how long?
376. What is the laparoscopic abnormalities seen Distal ends of the oviducts remain
in the fallopian tubes in women with pelvic TB? everted (“tobacco pouch” appearance

377. Gonococcal PID is associated with the D. Pain occurs during ovulation
following EXCEPT:
a. Acute onset of pain that increases with
movement
b. Purulent vaginal discharge
c. Nausea and vomiting
d. Pain occurs during ovulation

“Never design your LIFE like a GARDEN Where anyone can walk, rather Design your LIFE like the SKY Where everyone desires
to REACH”

Special Credits to: | Darna | Totoro | LittleSnoozy | Alma | RatioTeam | RecallsTeam | Astral | SKCCSLAS | Katz
Disclaimer: Some of the items and answers here are not mine but was verified so, If you find discrepancies please feel free to
correct, this serves only as a guide for easy and efficient reviewing.
God bless!
43 | P a g e Philippians 4:13 jpmrnmd
GYNECOLOGY – Mastery Review
“Master your Ultra Instinct”
Ultimate Reviewer ~800 items for Major Exams | Revalida | Board Exams

378. In PID the following are indications for


hospitalization EXCEPT:
a. Pregnancy
b. IUD
c. Uncertain diagnosis
d. syncope

379. The average age of menopause for Filipina T


is 47-48 years of age. 51 years old is the average of menopause
Genetic is a primary determinate of age of
menopause

380. The age of menopause is NOTgenetically F


yprogrammed.

381. Estradiol is reduced to greater extent than T


estrone in menopause patients. Estradiol or Serum E2( ave. 15 pg/ml range 10-
25pg/ml)
382. Nocturnal sleep disruption is the hallmark F
feature of declining estrogen. Hot flashes referred to as vasomotor episode is
the hallmark of declining estrogen status.
Nocturnal sleep disruption is consequence or
results from hot flashes
383. Recurrent urinary tract infection in a T
postmenopausal patient may be related to
estrogen deficiency.

384. Loss of cortical bone is greater with estrogen F


deficiency than loss of trabecular bone.
385. Osteopenia is defined by T score of -1 to -2.5 T
standard deviations.
386. Taking bisphosphonates have a significant T
effect on prevention of osteoporosis.
387. Older asymptomatic women will benefit in T
the quality of life with the use of hormone
replacement therapy.

“Never design your LIFE like a GARDEN Where anyone can walk, rather Design your LIFE like the SKY Where everyone desires
to REACH”

Special Credits to: | Darna | Totoro | LittleSnoozy | Alma | RatioTeam | RecallsTeam | Astral | SKCCSLAS | Katz
Disclaimer: Some of the items and answers here are not mine but was verified so, If you find discrepancies please feel free to
correct, this serves only as a guide for easy and efficient reviewing.
God bless!
44 | P a g e Philippians 4:13 jpmrnmd
GYNECOLOGY – Mastery Review
“Master your Ultra Instinct”
Ultimate Reviewer ~800 items for Major Exams | Revalida | Board Exams
388. Which of the following is considered a Post-menopausal women not taking hormones -
normal endometrial thickness for a normal EM = 4mm
postmenopausal woman?
A. 1 cm
B. 3 mm
C. 4 cms
D. 5 mm
389. What is the best time to do Hysterosonography, also called
sonohysterography? sonohysterography, uses sound waves to
A. Day 7-10 in a 28 day menstrual cycle produce pictures of the inside of a woman’s
B. Day 14 in a 28 day menstrual cycle uterus and help diagnose unexplained vaginal
C. Day 20-24 in a 28 day menstrual cycle bleeding. Hysterosonography is performed very
D. During menstruation much like a gynecologic exam and involves the
insertion of the transducer into the vagina after
you empty your bladder. Using a small tube
inserted into the vagina, your doctor will inject a
small amount of sterile saline into the cavity of the
uterus and study the lining of the uterus using the
ultrasound transducer. Ultrasound does not use
ionizing radiation, has no known harmful effects,
and provides a clear picture of soft tissues that
don’t show up well on x-ray images. It is best to
perform hysterosonography one week after
menstruation to avoid the risk of infection. Little or
no special preparation is required for this
procedure. Inform your doctor if there’s a
possibility you are pregnant. Leave jewelry at
home and wear loose, comfortable clothing. You
may be asked to wear a gown.
- Instilling saline (5-30mL) in uterine cavity
- Alternative To office hysterectomy
- Ibuprofen before the procedure

CI: Active cervical or uterine infection


Done in proliferative phase - EM lining is at its
lowest level
390. Associated with tamoxifen therapy - Endometrial polyps
Localized overgrowths of endometrial glands and
stroma that project beyond the surface of EM.
• SoftPliable
• Can be single or multiple
• Mostlyarise–fundus
• Broad base (sessile)
• Slender Pedicale (pedunculated)
• All age grp; peak = 40-49 years old - 20-25%
prevelance in reproductive age group
• Cause: Unknown

“Never design your LIFE like a GARDEN Where anyone can walk, rather Design your LIFE like the SKY Where everyone desires
to REACH”

Special Credits to: | Darna | Totoro | LittleSnoozy | Alma | RatioTeam | RecallsTeam | Astral | SKCCSLAS | Katz
Disclaimer: Some of the items and answers here are not mine but was verified so, If you find discrepancies please feel free to
correct, this serves only as a guide for easy and efficient reviewing.
God bless!
45 | P a g e Philippians 4:13 jpmrnmd
GYNECOLOGY – Mastery Review
“Master your Ultra Instinct”
Ultimate Reviewer ~800 items for Major Exams | Revalida | Board Exams
- Often associated with EM hyperplasia,
unopposed estrogen

• Majority: Asymptomatic
Chronic administration of non steroidal anti-
estrogen (Tamoxifen therapy)
- Polyps20-35%
- EM hyperplasia2-4%
- EMCA-1-2%

391. Most common presenting symptom of uterine


myoma 1. Pressure symptom
A. Dysmenorrhea - most common
B. Abnormal uterine bleeding - Urinary frequency and urgency = most common
C. Pressure symptom than bowel symptoms
D. Infertility - Extremely large and broad ligament M -
Hydroureter

2. Pain
- acquired dysmenorrhea - most frequent
- Severe pain = vascular compromise, torsion
- Mild = edematous swelling = dull aching
sensation

3. AUB - 30%
- menorrhagia = most common; intermenstrual
bleeding and disruption of normal pattern

392. Most common benign solid tumor of the Fibroma


ovary - most common benign solid neoplasm of the
A. Fibroma ovary
B. Dermoid cyst - <1% malignant potential
C. Brenner tumor - 5% of benign ovarian tumir, approx. 20% of all
D. Adenofibroma solid tumors
- Size vary from small nodules to huge pelvic
tumor weighing 50lbs
- Extremely slow growing tumor
- 90% unilateral

Differential Dx:
- non neoplastic thecomas
- Stromal hyperplasia
- Fibrosarcoma
- Brenner tumor

Tx:

“Never design your LIFE like a GARDEN Where anyone can walk, rather Design your LIFE like the SKY Where everyone desires
to REACH”

Special Credits to: | Darna | Totoro | LittleSnoozy | Alma | RatioTeam | RecallsTeam | Astral | SKCCSLAS | Katz
Disclaimer: Some of the items and answers here are not mine but was verified so, If you find discrepancies please feel free to
correct, this serves only as a guide for easy and efficient reviewing.
God bless!
46 | P a g e Philippians 4:13 jpmrnmd
GYNECOLOGY – Mastery Review
“Master your Ultra Instinct”
Ultimate Reviewer ~800 items for Major Exams | Revalida | Board Exams
TAHBSO - postmenopausal - resolution of
symptoms

393. MC tumor of the ovary in pre-pubertal age Desmoid Cyst


394. Type of uterine myoma that would present as Submucosal
AUB -most troublesome clinically
A. Subserosal -associated with abnormal uterine bleeding or
B. Intramural distortion pf uterine cavity
C. Submucosal -produce infertility & abortion
D. Parasitic Subserous = knobby contour
W/ continued growth = pedunculated myoma
wandering peritoneal cavity “parasitic myoma”
Broad Ligament Myoma = lateral growth
395. MC complication of Dermoid cyst? Torsion- 11%, most frequent complication
A. Rupture Rupture- 4-6%, but has more serious complication
B. Torsion like Chemical peritonitis
C. Malignant Ovarian tumor Uncommon are infection, hemorrhage &
D. Malignant degeneration malignant degeneration

396. Which of the ff is assoc. with Torsion? Hydrosalpinx


A. Usually involves the left ovary Dermoid cyst most frequently reported with
B. Hydrosalpinx torsion includes Parovarian cyst, solid benign
C. Malignant ovarian tumor tumors, serous cyst
D. Usually seen in normal ovaries of Usually affects the RIGHT ovary
premenopausal women Torsion of malinant ovarian tumor is very rare
Most frequent in children
397. Associated with masculinization during Luteoma -asooc with either normal endocrine
pregnancy funcion or prolonged secretion of Progesterone
A. Corpus Luteum cyst
B. Brenner tumor
C. Luteoma
D. Malignant teratoma

MATCHING TYPE:
A. Urethrocele/cystocele
B. Enterocele
C. Uterine prolapse
D. Rectocele
398. Assoc with urgency and stress incontinence
Ans. A. Urethrocele/cystocele
399. Tumor bulging out of introitus- uterine prolapse.
Ans. C. Uterine prolapse

“Never design your LIFE like a GARDEN Where anyone can walk, rather Design your LIFE like the SKY Where everyone desires
to REACH”

Special Credits to: | Darna | Totoro | LittleSnoozy | Alma | RatioTeam | RecallsTeam | Astral | SKCCSLAS | Katz
Disclaimer: Some of the items and answers here are not mine but was verified so, If you find discrepancies please feel free to
correct, this serves only as a guide for easy and efficient reviewing.
God bless!
47 | P a g e Philippians 4:13 jpmrnmd
GYNECOLOGY – Mastery Review
“Master your Ultra Instinct”
Ultimate Reviewer ~800 items for Major Exams | Revalida | Board Exams
400. Incomplete emptying and constipation
Ans. D. Rectocele
401. Usually contains small intestine
Ans. B. Enterocele
402. Transillumination is the key to diagnosis
Ans. B. Enterocele

Choices for # 6-10


A. Le Fort
B. McCall
C. Burch Procedure
D. Anterior Colporrhaphy
403. Management of choice for urethrocoele:
Ans. D. Anterior Colporrhaphy
404. Procedure involving repair of the Coopers Ligament
Ans. C. Burch Procedure
405. Repair of enterocele neck
Ans. B. McCall
406. Surgery for old female that does not desire sexual activity?
Ans. A. Le fort colpocleisis
407. repair of pubocervical fascia in pt with cystocele
Ans. D. Anterior Colporrhapy

408. Ovulation inducing medication such as The incidence of Ectopic pregnancy following
clomiphene citrate and gonadotrophins plus infertility treatment is much higher compared with
tubal surgery to enhance patency, has shown that in spontaneous pregnancies.
to increase this complication 2-3 fold in the Preliminary results of a case-control study suggest
general population: that CC or factors closely associated with CC therapy
A. Pregnancy failure (or both) may increase the risk of ectopic pregnancy.
B. Abortion ● From NCBI journal
C. Ectopic pregnancy
D. Preterm labor

409. If anovulation is primarily caused by Dopamine agonists are the primary treatment of
hyperprolactinemia, the drug of choice to Hyperprolactinemia, if associated with a pituitary
induce ovulation is: lesion and <10mm adenoma. Surgical therapies
A. Dopamine agonists should only be considered with prolactin-secreting
B. Clomiphene citrate adenomas resistant to medical therapy.
C. Urinary gonadotropin If the tumor size is > 10mm, the bromocriptine use is
D. Recombinant FSH and LH advised during pregnancy to avoid significant tumor
growth.

“Never design your LIFE like a GARDEN Where anyone can walk, rather Design your LIFE like the SKY Where everyone desires
to REACH”

Special Credits to: | Darna | Totoro | LittleSnoozy | Alma | RatioTeam | RecallsTeam | Astral | SKCCSLAS | Katz
Disclaimer: Some of the items and answers here are not mine but was verified so, If you find discrepancies please feel free to
correct, this serves only as a guide for easy and efficient reviewing.
God bless!
48 | P a g e Philippians 4:13 jpmrnmd
GYNECOLOGY – Mastery Review
“Master your Ultra Instinct”
Ultimate Reviewer ~800 items for Major Exams | Revalida | Board Exams
410. Best time to measure serum Ovulation is expected 5 to 10 days after last tablet
progesterone in women taking clomiphene ● Natural progesterone is measure in the midluteal
citrate? phase
A. Exactly at midluteal phase
B. 1 week after last intake clomiphene citrate
C. 2 weeks after last intake clomiphene
citrate
D. 3 days after increase of basal body
temperature
411. Local explanation for women with failed to ovulate treated with high dose of CC
A. Negative feedback response from pituitary
B. Inability of HPA axis to respond
C. Lack of ovarian response to raised gonadotropin levels
D. Unresponsive higher center to high GnRH
412. The use of gonadotropin therapy is indicated for ovulation induction when:
A. Estrogen concentration or levels are too low
B. Lack of withdrawal bleeding after progesterone bleeding
C. Clomiphene citrate and letrozole fails
D. A and B
E. A, B and C
413. what procedure will you do if ovulation will not occur after 6-12 months
A. Salpingotomy
B. HSG
C. Laparoscopy
D. Surgery
414. IVF embryo transfer is the last step in algorithm of infertile couple but can be primary
management in case of
A. Bilateral tubal obstruction
B. Severe male factor
C. Immunological cause of infertility
D. A and B
E. A, B, C
415. The highest percentage of potential risk complication in singleton pregnancy after IVF is:
A. Preterm birth
B. SGA
C. Neonatal ICU admission
D. Low birth weigh

416. Initial treatment for couples with unknown cause of


infertility
A. Ovulation
induction
B. Intrauterine
insemination
C. IVF
D. A and B
E. A, B, C

“Never design your LIFE like a GARDEN Where anyone can walk, rather Design your LIFE like the SKY Where everyone desires
to REACH”

Special Credits to: | Darna | Totoro | LittleSnoozy | Alma | RatioTeam | RecallsTeam | Astral | SKCCSLAS | Katz
Disclaimer: Some of the items and answers here are not mine but was verified so, If you find discrepancies please feel free to
correct, this serves only as a guide for easy and efficient reviewing.
God bless!
49 | P a g e Philippians 4:13 jpmrnmd
GYNECOLOGY – Mastery Review
“Master your Ultra Instinct”
Ultimate Reviewer ~800 items for Major Exams | Revalida | Board Exams

417. Best therapy offered in cases of proximal and distal obstruction of the tube is:
A. IUI
B. IVF
C. Ovulatory drug
D. Operative reconstructive therapy
418. The diagnosis of luteal phase deficiency is made with serum progesterone levels consistently
below 10ng/ml
A. 1 week after menstruation
B. Mid luteal phase
C. 3 days after menstruation
D. Menstruation
419. Prerequisite for clomiphene citrate adequate estrogen production
therapy?
420. Major side effect of clomiphene citrate The potential side effects of Clomid are related to
A. Formation of ovarian cyst its antiestogen effects: hot flushes, abdominal
B. Hyperovarian stimulation distention and bloating, emotional liability,
C. Abdominal pain/ bloating depression, and visual changes. These SE are
D. Vasomotor flushes mostly mild and disappears after discontinuation.
Multiple gestation pregnancy is a major SE of
ovulation induction and ARTs. 8% are Clomid-
induce pregnancies and 20% from Gonadotropins.
The other major complication of OI with
gonadotropins is ovarian hyperstimulation
syndrome (OHSS).
421. Tubal embryo transfer (TET) is similar to zygote intrafallopian transfer (ZIFT) except that in TET
the embryo are transferred how many hours after fertilization:
A. 8-72 hours
B. 24-72
C. 36-72
D. 46-72 hours
422. In women w/ unexplained infertility, laparoscopy can identify previous unsuspected pathology in
30-50% of patient, the most common condition diagnosed is
A. Adhesions
B. Endometriosis
C. Polycystic ovaries
D. Congenital Mullerian Abnormality

423 Basal Body Temp, an obvious temperature elevation occurs due to the thermogenic effect of this
hormone
A. Estrogen
B. Progesterone
C. Luteal hormone
D. FSH
424. The m/c cause of female hyperandrogenism and present with the m/c symptom of infertility.
A. Congenital Adrenal Hyperplasia (Answer??)
B. Hypothyroidism

“Never design your LIFE like a GARDEN Where anyone can walk, rather Design your LIFE like the SKY Where everyone desires
to REACH”

Special Credits to: | Darna | Totoro | LittleSnoozy | Alma | RatioTeam | RecallsTeam | Astral | SKCCSLAS | Katz
Disclaimer: Some of the items and answers here are not mine but was verified so, If you find discrepancies please feel free to
correct, this serves only as a guide for easy and efficient reviewing.
God bless!
50 | P a g e Philippians 4:13 jpmrnmd
GYNECOLOGY – Mastery Review
“Master your Ultra Instinct”
Ultimate Reviewer ~800 items for Major Exams | Revalida | Board Exams
C. PCOS
D. DM

425. Person who lacks oocyte (ovarian failure) and desire pregnancy may be a candidate for:
A. Adoption
B. Oocyte donation
C. Gonadotropin Therapy
D. Steroid hormone therapy
426. Major complication associated with induction Multiple gestation pregnancy is a major SE of
of ovulation with gonadotropins ovulation induction and ARTs. 8% are Clomid-
A. Ovarian Hyperstimulation induce pregnancies and 20% from
B. Multiple pregnancies Gonadotropins. The other major complication of
C. Abdominal pain/bloating OI with gonadotropins is ovarian hyperstimulation
D. A and B syndrome (OHSS).
E. A and C
20. Sonohysterography appears to be superior to HSG in the detection of uterine malformation,
correctly identifying 90% of abnormalities in infertile patients
A. True
B. False

427. In the latest classification of HMB, one of the


following is not a structural cause of abnormal
bleeding
A. Ovulatory dysfunction
B. Polyp
C. Hyperplasia
D. myoma

428. An estrogen secreting tumor that can cause Estrogen Producing tumors:
AUB is - Granulosa Cell Tumor
A. Serous cystadenoma - Thecoma
B. GCT
C. EST
429. A regular menstrual cycle is a TRUE
progesterone withdrawal effect. Menstruation is the occurrence of bleeding when
progesterone is withdrawn from an estrogen- and
progestin-primed uterus
430. More than 80 points of blood loss is TRUE
considered to be menorrhagia.
431. Von Willebrand disease is the second most TRUE
common cause of AUB in adolescents. Most common: Infection of the upper genital tract –
PID
432. In a reproductive age woman, the first TRUE
consideration is she has AUB is pregnancy.

“Never design your LIFE like a GARDEN Where anyone can walk, rather Design your LIFE like the SKY Where everyone desires
to REACH”

Special Credits to: | Darna | Totoro | LittleSnoozy | Alma | RatioTeam | RecallsTeam | Astral | SKCCSLAS | Katz
Disclaimer: Some of the items and answers here are not mine but was verified so, If you find discrepancies please feel free to
correct, this serves only as a guide for easy and efficient reviewing.
God bless!
51 | P a g e Philippians 4:13 jpmrnmd
GYNECOLOGY – Mastery Review
“Master your Ultra Instinct”
Ultimate Reviewer ~800 items for Major Exams | Revalida | Board Exams
433. Among all the age group, adolescents have TRUE
the highest chance of AUB due to genital tract Reproductive Tract Dse:
infection. 3. Infection of the upper genital tract - PID
(MC in Adolescents)
434. Ovulatory AUB will not slough uniformly TRUE
The endometrium will grow and proliferate because
there is still ESTROGEN produced. But Since there
is no PROGESTERONE release, some area of
Endometrium will not slough off uniformly
435. In anovulatory AUB, it is caused by FALSE
prostaglandin secretion. In ovulatory DUB, during menstrual cycle, normally
even before the patient bleeds, there is
Prostaglandin that will be secreted.
436. Endometrial biopsy must be done in all FALSE
patients.
437 There is no need to perform CBC if the bleeding of the patient is not profuse
438. GnRH agonist can be used to stop the Estrogens, progestins, androgens, nonsteroidal
bleeding in anovulatory AUB. anti-inflammatory drugs (NSAIDs), ergot
derivatives, antifibrinolytics, and gonadotropin-
releasing hormone (GnRH) agonists have been
used to treat abnormal uterine bleeding (AUB).

439. It results from an alteration in the


neuroendocrinologic function of a woman
A. Blood coagulation disorder
B. Abnormality in the production of
PGF2a
C. Anovulation
D. Pregnancy

440. One of the following causes ovulatory type of


DUB
A. Ectopic pregnancy
B. Prothrombin deficiency
C. ITP
D. Increase uterine prostacyclin
production

441. Uterine evaluative techniques to diagnose


submucous myoma include the ffg EXCEPT
A. Laparoscopy
B. Hysteroscopy
C. Hysterography
D. TVS

“Never design your LIFE like a GARDEN Where anyone can walk, rather Design your LIFE like the SKY Where everyone desires
to REACH”

Special Credits to: | Darna | Totoro | LittleSnoozy | Alma | RatioTeam | RecallsTeam | Astral | SKCCSLAS | Katz
Disclaimer: Some of the items and answers here are not mine but was verified so, If you find discrepancies please feel free to
correct, this serves only as a guide for easy and efficient reviewing.
God bless!
52 | P a g e Philippians 4:13 jpmrnmd
GYNECOLOGY – Mastery Review
“Master your Ultra Instinct”
Ultimate Reviewer ~800 items for Major Exams | Revalida | Board Exams
442. It is best to exclude a diagnosis of Endometrial biopsy
endometrial cancer by endometrial biopsy on the - age >35 years
ffg patients - Long standing history of excessive menstrual
A. 29 day old neonate observed to have bleeding
blood stains on diaper pads - Endometrial thickness >8mm
B. 49 y.o. Nulligravid with a history of
menstural interval of 24 days
C. 49 y.o. G2P2 on her 5th day of
menses with TVS findings of 4mm endometrial
thickness
D. 51 y.o. G3P3 menopause for 3 yrs with
endometrial thickness of 6mm

443. Medroxy progesterone acetate is a medical Cyclic Progestogen (Medroxyprogesterone


management for the ffg women with AUB acetate)
A. At risk for thrombosis ● marketed as Provera 10 mg for 10days each
B. Adolescent nulligravid month for a few months.
C. Having regular but profuse ● For adolescent with HMB with increased risk for
menstruation thrombosis
D. AOTA ● Expect bleeding for total of 3mos-6mos wait till
maturation of HPO Axis.
● Given to adolescent waiting for maturation of
HPO

444. This medical management is ideal for Danazol - 60 % reduction in MBL


ovulatory DUB for it offers 80% reduction of blood NSAID - Reduce the mean MBL by approximately
loss after 3 months of use 20% to 50%
A. Danazol GnRH agonists - Markedly reduced MBL from 100
B. NSAIDS to 200 mL per cycle to 0 to 30 mL per cycle
C. LNG-IUS
D. GnRH agonists

445. 30 y.o nulligravid with AUB due to


submucous myoma is best managed by
A. NSAIDS
B. Hysteroscopic myomectomy
C. Endometrial ablation
D. Total abdominal hysterectomy

456. 30 y.o. G0, with history of thrombophilia,


having acute heavy menstrual bleeding but
hemodynamically stable, may be managed by
A. Intravenous 25mg conjugated equine
estrogen
B. High dose OCPs
C. High dose progestin
D. AOTA

“Never design your LIFE like a GARDEN Where anyone can walk, rather Design your LIFE like the SKY Where everyone desires
to REACH”

Special Credits to: | Darna | Totoro | LittleSnoozy | Alma | RatioTeam | RecallsTeam | Astral | SKCCSLAS | Katz
Disclaimer: Some of the items and answers here are not mine but was verified so, If you find discrepancies please feel free to
correct, this serves only as a guide for easy and efficient reviewing.
God bless!
53 | P a g e Philippians 4:13 jpmrnmd
GYNECOLOGY – Mastery Review
“Master your Ultra Instinct”
Ultimate Reviewer ~800 items for Major Exams | Revalida | Board Exams

457. Oocyte meiosis arrested at prophase 1 Reduction division (division I or meiosis I) – initiates in
from fetal period until time of OVULATION. the fetal ovary but is then arrested and completed at
the time of ovulation.
458. Fertilizatiin occur at what part of the A uterine tube contains 3 parts. The first segment,
fallopian tube closest to the uterus, is called the isthmus. The second
segment is the ampulla, which becomes more dilated
in diameter and is the most common site for
fertilization. The final segment, located farthest from
the uterus, is the infundibulum.
459. First cell division leading to two-cell During the first 12 hours after conception, the fertilized
embryo takes how many hours? egg cell remains a single cell. After approximately 30
A. 5 hours, it divides from 1 cell into 2 and 15 hours later,
B. 10 the 2 cells divide into 4. And at the end of 3 days, the
C. 15 fertilized egg cell has become a berry-like structure
D. 20 made up of 16 cells. This structure is called a morula,
which is Latin for mulberry.

460. 3 days after embryo enter the uterus, By the time the embryo enters the uterus, it will be
implantation occurs when trophoblastic cells between the 32-cell and blastocyst stages of
contact endometrium and burrow beneath the development. Implantation occurs when trophoblastic
surface by enzymatic action: cells contact endometrium and burrow beneath the
A. Incorrect surface by enzymatic action. This generally takes
B. Correct place 3 days after the embryo enters the uterus
461. The earliest fetal epithelium to develop: The earliest fetal epithelium to develop is the
A. Ectoderm ectoderm, the second is the endoderm, and the third
B. Endoderm is the mesoderm
C. Mesoderm
D. A and C
461. HCG is secreted by synctiotrophoblast at Responsible for maintaining the corpus luteum
about the time of implantation. It doubles Detected in the peripheral blood of the mother as
quantity every 1.2 to 2 days until? early as 6 days after ovulation, but it is always seen
Ans 7-9wks gestation by day 12.
Concentration doubles every 1.2 to 2 days
Its highest point at 7 to 9 weeks of pregnancy
462. Vagina develops from the sinovaginal The vaginal canal is not patent throughout its entire
bulbs which are outgrowths of the urogenital length until the sixth month
sinus. The vagina develops from the sinovaginal bulbs,
Correct which are outgrowths of the urogenital sinus.
The sinovaginal bulbs then canalize to form the
vagina.
Failure of these bulbs to form leads to agenesis of
the vagina.
463. Feminism of undifferentiated external TRUE
genitalia happens in the absence of androgen In the absence of a Y chromosome or in the absence
stimulation of a gonad, development will be female in nature.

“Never design your LIFE like a GARDEN Where anyone can walk, rather Design your LIFE like the SKY Where everyone desires
to REACH”

Special Credits to: | Darna | Totoro | LittleSnoozy | Alma | RatioTeam | RecallsTeam | Astral | SKCCSLAS | Katz
Disclaimer: Some of the items and answers here are not mine but was verified so, If you find discrepancies please feel free to
correct, this serves only as a guide for easy and efficient reviewing.
God bless!
54 | P a g e Philippians 4:13 jpmrnmd
GYNECOLOGY – Mastery Review
“Master your Ultra Instinct”
Ultimate Reviewer ~800 items for Major Exams | Revalida | Board Exams
467. AMH (anti mullerian hormone) acts locally TRUE.
to regress the mesonephric duct and Paramesonephric duct is suppressed because of the
testosterone systemically giving rise to the secretion and action of AMH. with the absence sertoli
paramesonephric duct. cells and the anti-mullerian Hormone (AMH) is not
produced, the paramesonephric duct system
develops into a phenotypic female configuration, and
the mesonephric duct system is suppressed.
468. In the presence of ovaries or of gonadal Female genital tract
agenesis, the mesonephric duct regress, and
the paramesonephric ducts develop into
469. Early in the embryo life, there are two TRUE
sets of paired genital ducts that develop in
each sex. These are the Wolffian ducts and
the Mullerian ducts.
470. The sperm are attracted to an egg Chemotaxis
through the process known as The sperm are attracted to an egg through the
process known as chemotaxis, which is related to
capacitation of the sperm. Aided by the binding of
progesterone to a surface receptor on the sperm
Allows an increase in intracellular calcium ion
concentration, which increases sperm motility
(chemokinesis)

471. Angiogenesis, or blood vessel formation, Angiogenesis is seen by day 15 or 16.


is seen in the extraembryonic of the yolk sac Embryonic heart function begins in the third week of
by day gestation, forming a primitive cardiovascular system.
A. 13-14 Blood circulation starts about this time, and the
B. 15-16 cardiovascular system becomes the first functioning
C. 17-18 organ system within the embryo.
D. 19-20
472. Teratogen exposure after 49th day of TRUE
gestation may injure or kill the embryo or
cause developmental and growth retardation
but usually will not be responsible for specific
malformations.
473. The period of embryonic development is The period of embryonic development is said to be
said to be complete when the embryo attains a complete when the embryo attains a crown– rump
crown rump length of length of 30 mm
A. 10mm
B. 20mm
C. 30mm
D. 40mm
474. Fetal intrauterine produces urine TRUE
throughout pregnancy that contribute to The fetus produces urine throughout all the periods
amniotic fluid of gestation, but the placenta handles the excretory
functions of the fetus.

“Never design your LIFE like a GARDEN Where anyone can walk, rather Design your LIFE like the SKY Where everyone desires
to REACH”

Special Credits to: | Darna | Totoro | LittleSnoozy | Alma | RatioTeam | RecallsTeam | Astral | SKCCSLAS | Katz
Disclaimer: Some of the items and answers here are not mine but was verified so, If you find discrepancies please feel free to
correct, this serves only as a guide for easy and efficient reviewing.
God bless!
55 | P a g e Philippians 4:13 jpmrnmd
GYNECOLOGY – Mastery Review
“Master your Ultra Instinct”
Ultimate Reviewer ~800 items for Major Exams | Revalida | Board Exams
The urine produced by the fetus contributes to the
amniotic fluid.
The fetus may swallow the amniotic fluid and
recirculate it through the digestive system.
475. X chromosome is responsible for the FALSE
development of testes. Testis development is directed by a gene located on
the short arm of Y—testis- determining factor (TDF),
also called sex-determining region (SRY).
476.41 y/o G3P2, 16 wk AOG scheduled for All general anaesthetic drugs cross the placenta and
cholecystectomy. Generalized ET anes affect there is no optimal general anaesthetic technique.
the baby? Neither is there convincing evidence that any
A. No,.... particular anaesthetic drug is toxic in humans. There
B. Yes,..... Crosses the placenta is weak evidence that nitrous oxide should be
C. Yes, .... Need trophoblastic receptor. avoided in early pregnancy due to a potential
D. Yes,.... association with pregnancy loss with high exposure.
477. Intra abdominal remnant of umbilical vein. Obliteration of the intra‐abdominal umbilical vein at
birth produces a hepatic remnant termed the
ligamentum teres.
478. A 36 yr old primigravida consulted at 27 Pulmonary hypoplasia is a substantial cause of
wks gestation. Transabdominal ultrasound death in newborn infants, and oligohydramnios is one
revealed low amniotic fluid volume. Which of of the most commonly associated abnormalities.
the following is the most likely effect of Oligohydramnios reduces the intrathoracic cavity
oligohydramnios to the fetus? size, thus disrupting fetal lung growth and leading to
A. Congenital megacolon pulmonary hypoplasia. The exact mechanism by
B. Tracheoesophageal atresia which oligohydramnios alters the respiratory system
C. Pulmonary hypoplasia structure and the effect of oligohydramnios on long-
D. Ventricular septal defect term respiratory outcomes remain unknown.
479. A 21 y/o G2P1 (1001) 20 wks AOG Congenital abnormalities that impair normal
followed up with an UTZ result of absence of development or function of the fetal kidneys generally
amniotic fluid . Previous UTZ result result in little or no amniotic fluid (oligohydramnios or
normal amniotic fluid at 14 wks. AOG . Cause anhydramnios)
of Oligohydramnios?
A. Tracheoesophageal fistula
B. ventriculomegaly
C. Renal agenesis
D. Imperforate anus

480. A 25 year old clerk consulted because of A. The risk is increase in 1st trimester
erythematous rash initially on the face then
spread to the other parts of the
body with low grade fever and body
malaise. One day prior to consult, the
patient is positive for pregnancy test. What
is
the risk:

“Never design your LIFE like a GARDEN Where anyone can walk, rather Design your LIFE like the SKY Where everyone desires
to REACH”

Special Credits to: | Darna | Totoro | LittleSnoozy | Alma | RatioTeam | RecallsTeam | Astral | SKCCSLAS | Katz
Disclaimer: Some of the items and answers here are not mine but was verified so, If you find discrepancies please feel free to
correct, this serves only as a guide for easy and efficient reviewing.
God bless!
56 | P a g e Philippians 4:13 jpmrnmd
GYNECOLOGY – Mastery Review
“Master your Ultra Instinct”
Ultimate Reviewer ~800 items for Major Exams | Revalida | Board Exams
481. gabernaculum testis Round ligament
One week later, they reach the urogenital sinus.
At that time, the two müllerian ducts fuse to form a
single canal at the level of the inguinal crest.
This crest gives rise to the gubernaculum, which is
the primordium of the round ligament.
482. When does meiosis II completed? Penetration of sperm
If sperm penetration occurs, then meiosis II is
completed with extrusion of the second polar yielding
a haploid oocyte, entered by a haploid sperm

TRUE OR FALSE FALSE


483. 65 y/o female diagnosed with CIN III
undergo hysterectomy and requires papsmear
there after for the vagina.
TRUE OR FALSE FALSE
484. Lugol’s solution as an adjunct…produces
brown solution in abnormal cells
TRUE OR FALSE TRUE
485. Primary site of vaginal cancer is at the ● Primary vaginal cancer usually occur as squamous
upper third. cell carcinoma (90%)
● Most common location of SCC: upper third

TRUE OR FALSE TRUE


486. Clear cell carcinoma exhibiting hobnail Clear cell carcinoma has 3 histologic subtypes:
cells may be treated with wide local excision ● Tubulocystic (hobnail)
followed by lymphadenectomy ● Solid pattern
● Papillary pattern

Treatment:
Stage I to early stage II → radical hysterectomy with
partial or complete vaginectomy
Advance stage → wide local excision followed by
pelvic lymphadenectomy
TRUE OR FALSE FALSE**
487. Most vulvar atypias are located at labia Wala akong mabasa about this :(
minora Pero sa if Lichen Sclerosus ang tanong: CLITORAL
in location ang most common
TRUE OR FALSE TRUE
488. Cancer at the middle aspect of the labia
minora that has extended to the perineal body
is stage 2

“Never design your LIFE like a GARDEN Where anyone can walk, rather Design your LIFE like the SKY Where everyone desires
to REACH”

Special Credits to: | Darna | Totoro | LittleSnoozy | Alma | RatioTeam | RecallsTeam | Astral | SKCCSLAS | Katz
Disclaimer: Some of the items and answers here are not mine but was verified so, If you find discrepancies please feel free to
correct, this serves only as a guide for easy and efficient reviewing.
God bless!
57 | P a g e Philippians 4:13 jpmrnmd
GYNECOLOGY – Mastery Review
“Master your Ultra Instinct”
Ultimate Reviewer ~800 items for Major Exams | Revalida | Board Exams
TRUE OR FALSE FALSE
489. Imiquimod cream is discontinued due to 5 - FU cream is the one discontinued due to its
its severe burning effect severe burning effect
**Management of VIN III:
● SKINNING VULVECTOMY → superficial skin,
preserves clitoris
● CO2 LASER ABLATION → for hairy area such as
mons pubis
● SURGICAL EXCISION → for raised lesions and
older women
● IMIQUIMOD
● 5 - FU CREAM → + severe burning
● HPV VACCINE → for young patients

TRUE OR FALSE TRUE


490. Vulvar cancer at fourchette spreads If at MIDLINE → bilateral
bilaterally If at INGUINOFEMORAL → ipsilateral
TRUE OR FALSE FALSE
491. Nodular vulvar melanoma carries better Vaginal melanoma has worse prognosis than vulvar
prognosis than vaginal melanoma melanoma; it is usually fatal
**Pero note lang guys may two types of vulvar
melanoma: superficial and nodular. If the question is
alin ang mas worse prognosis between the two:
nodular has worse prognosis because it is more
invasive than the superficial typ

492. The major effect of synthetic progestin Synthetic Progesterone


component of OCP is: Major Effects:
A. Makes cervical mucus slippery and elastic a. Inhibit ovulation
B. Maintains the endometrium b. Makes the cervical mucus thick
C. Inhibit ovulation Synthetic Estrogen
D. Prevent unscheduled bleeding Major Effects:
a. Maintain endometrium to prevent unscheduled
bleeding para di mag bleed ng mag bleed ang
patient
b. Prevent follicular maturation or development
493. What is the estrogen component used in Ethinyl estradiol
contraceptive preparations that is orally active stable, orally absorbed, and an active agent
and highly potent? Mestranol- could not be absorbed orally
A.17b - estradiol - should undergo methylation first before it can be
B.Mestranol transformed into an active agent, Ethinyl estradiol.
C. Ethinyl estradiol
D. estrone valerate
494. This OCP formulation contains a low Daily Progestin Formulations
progestin without any estrogen: ● Also known as minipills
A. monophasic ● Consist of tables containing a low dose of
B. minipill progestin without any estrogen and

“Never design your LIFE like a GARDEN Where anyone can walk, rather Design your LIFE like the SKY Where everyone desires
to REACH”

Special Credits to: | Darna | Totoro | LittleSnoozy | Alma | RatioTeam | RecallsTeam | Astral | SKCCSLAS | Katz
Disclaimer: Some of the items and answers here are not mine but was verified so, If you find discrepancies please feel free to
correct, this serves only as a guide for easy and efficient reviewing.
God bless!
58 | P a g e Philippians 4:13 jpmrnmd
GYNECOLOGY – Mastery Review
“Master your Ultra Instinct”
Ultimate Reviewer ~800 items for Major Exams | Revalida | Board Exams
C. biphasic
D. triphasic

495. The following symptoms are related to METABOLIC EFFECTS


estrogen component of OCP except: ● Nausea, breast tenderness and fluid retention
A. nausea ● Insignificant reductions in circulating levels of
B.breast tenderness vitamin B complex and C and increase vitamin A
C. fluid retention ● Melasma - hyperdiscoloration
D. wt gain ● Changes in mood and depression
● Adverse androgenic effects (acne)
● Weight gain dose related anabolic effect of
progestins
● Unscheduled (breakthrough) bleeding
● Increased frequency of headache

496. What does WHO category 4 means in WHO MEDICAL ELIGIBILITY CRITERIA (MEC)
WHO medical eligibility criteria: WHO 1: Can use the method. No restriction use.
A. can used the method without restriction WHO 2: Can use the method. Advantages generally
B.advantage generally outweigh theoretical or outweigh theoretical or proven risk
proven risk WHO 3: Should not use method unless a doctor or
C.theoretical or proven risk usually outweigh nurse makes a clinical judgment that the client can
the advantages safely use OCP.
D.condition represents an unacceptable WHO 4: Cannot use the method. Condition
health risk if method is used represents an unacceptable health risk.

497 Which of the following is relative Relative Contraindications


contraindication for OCP used: ● Migraine or vascular headaches
A. Gestational DM ● Cardiac or renal dysfunction
B. Undiagnosed AUB ● Gestational DM or prediabetes – is not a
C. suspected pregnancy contraindication, unless they have vascular
D. impaired liver fxn involvement = ABSOLUTE CONTRAINDICATION
● Undiagnosed cause of amenorrhea or genital
bleeding
● Women with galactorrhea or amenorrhea
● Varicose veins
● Heavy smoker < 35 years old
● Sickle cell or Sickle β-thalassemia disease
● Cholestatic jaundice during pregnancy
● Hepatitis or mononucleosis
● Asthma
● Ulcerative colitis
498. How is contraceptive patch used: Once patch each week for 3 consecutive weeks and
A.one patch each week for 4 weeks 1 week free
B. one patch each week for 3 weeks
C. One patch is used for 1 month
D. One patch every other week for 1 month

“Never design your LIFE like a GARDEN Where anyone can walk, rather Design your LIFE like the SKY Where everyone desires
to REACH”

Special Credits to: | Darna | Totoro | LittleSnoozy | Alma | RatioTeam | RecallsTeam | Astral | SKCCSLAS | Katz
Disclaimer: Some of the items and answers here are not mine but was verified so, If you find discrepancies please feel free to
correct, this serves only as a guide for easy and efficient reviewing.
God bless!
59 | P a g e Philippians 4:13 jpmrnmd
GYNECOLOGY – Mastery Review
“Master your Ultra Instinct”
Ultimate Reviewer ~800 items for Major Exams | Revalida | Board Exams
499. What is the dose of DMPA used for A single 150 IM dose will suppress ovulation for 14
contraception? weeks
A. 150ug
B. 150mg
C. 150gm
D. 150kgs
500. The following are the effects of subdermal Subdermal implants a/e s/e:
implant: Irregular bleeding & headache
A.Endometrial injury Acne, weight gain, mastalgia, mood change,
B. Infertility depression, hyperpigmentation, hirsutism,
C.Irregular bleeding galactorrhea
D.AOTA
E.A and C only
501. The ff contraceptive could be given for a except Subdermal implant
pt taking anticonvulsant drugs, except: Contraception for women with Chronic illness:
A.DMPA Epilepsy
B.IUD Anti-seizure drugs decrease the efficacy of OCs &
C Subdermal implant Norplant
D. Low dose OCP Ocs with 50 (miu/g) Estrogen, DMPA, IUDs

501. Revised Rotterdam 2003 Criteria of Rotterdam


PCOS -Oligoovulation/anovulation/menstrual irregularity
-Hyperandrogenism
-Polycystic ovaries
NIH
- Oligo-ovulation
- Hyperandrogenism and/or hyperandrogenemia

502. Genetic difference in ______ activity 5a-reductase


likely explains the discrepancy in the racial Elevated androgen play a major in determining the
incidence of hirsutism in women with PCOS type and distribution of hair. Within a hair follicle,
testosterone is converted by the enzymes 5a-
reductase to dihydrostestosterone (DHT). Although
both testosterone and DHT convert short, sofet vellus
hair to coarse terminal hair, DHT is markedly more
effective than testosterone. COnversion is
irreversible, and only hairs in androgen- sensitive
areas are changed in this manner to terminal hairs.
As a result the most common areas affected with
excess hair growth in women with PCOS include
upper lip, chin, sideburns,chest, and linea alba of the
lower abdomen.
503.This compartment is the most consistent Ovarian
contributor of androgen In PCOS patient, you have ovarian source of
androgen in the form of testosterone and your
adrenals also will contribute to that.
MARKERS OF ANDROGEN PRODUCTION

“Never design your LIFE like a GARDEN Where anyone can walk, rather Design your LIFE like the SKY Where everyone desires
to REACH”

Special Credits to: | Darna | Totoro | LittleSnoozy | Alma | RatioTeam | RecallsTeam | Astral | SKCCSLAS | Katz
Disclaimer: Some of the items and answers here are not mine but was verified so, If you find discrepancies please feel free to
correct, this serves only as a guide for easy and efficient reviewing.
God bless!
60 | P a g e Philippians 4:13 jpmrnmd
GYNECOLOGY – Mastery Review
“Master your Ultra Instinct”
Ultimate Reviewer ~800 items for Major Exams | Revalida | Board Exams
Source::Marker
Ovary: Testosterone
Adrenal gland: DHEAS
Periphery: 3a-dial-G

504. Dysregulation of this enzyme may be one CYP17


of the central pathogenic mechanisms
underlying hyperandrogenism
505. Most common cause of insulin resistance obesity
and compensatory hyperinsulinemia
505. Characterized by insulin Metabolic syndrome/dysmetabolic syndrome x
resistance/hyperinsulinemia This syndrome is characterized by insulin resistance,
obesity, atherogenic dyslipidemia, and hypertension.
506. Ultrasonographic criteria for PCOS Enlarged ovaries (>10 cm3)
10 or more more perihpherally oriented cystic
structures (2-8mm) surrounding a dense stroma
Although Rotterdam is not strict about this,
sometimes they consider >7.
507. Unpposed increase in estrogen levels Endometrial cancer
uninterrupted by progesterone increased the
risk of ?
508. Obese patients w/ PCOS, the initial Weight reduction
recommendation is
509. Administration of 100mg/day of this Cyproterone acetate
______ on days 5 to 15 and ethinyl estradiol
30-50mg/day on cycle days 5 to 26 allow
regular menstrual bleeding, excellent
contraception, and is effective in severe
hirsutism and acne.

510. Dysregulation of this enzyme in both the CYP17


adrenals and the ovary…
a. CYP17
b. 5-a reductase
c. 7-B HSD
d. aromatase

511. This is a reliable marker of insulin Acanthosis nigricans


resistance and is characterized by thickened, This skin condition is characterized by thickened,
pigmented, velvety skin lesion found in the gray-brown velvety plaques seen in areas of flexure
vulva, axilla,nape, below the breast and on such as the back of the neck, axillae, the crease
inner thigh beneath the breast, waist and groin. Thought to be a
a. Squamous hyerplasia cutaneous marker of insulin resistance, acanthosis
b. Acanthosis nigricans nigricans may be found in individuals with or without
c. Melanoma PCOS. Insulin resistance leads to hyperinsulinemia,
d. Fox-Fordyce disease which is believed to stimulate keratinocyte and

“Never design your LIFE like a GARDEN Where anyone can walk, rather Design your LIFE like the SKY Where everyone desires
to REACH”

Special Credits to: | Darna | Totoro | LittleSnoozy | Alma | RatioTeam | RecallsTeam | Astral | SKCCSLAS | Katz
Disclaimer: Some of the items and answers here are not mine but was verified so, If you find discrepancies please feel free to
correct, this serves only as a guide for easy and efficient reviewing.
God bless!
61 | P a g e Philippians 4:13 jpmrnmd
GYNECOLOGY – Mastery Review
“Master your Ultra Instinct”
Ultimate Reviewer ~800 items for Major Exams | Revalida | Board Exams
dermal fibroblast growth, producing the
characteristics skin changes.

512. Patient with PCOS who have chronic Endometrial , Ovary


anovulation, persists by elevated, estrogen Long Term Risk:
levels increase the risk of which ones cancer? - Metabolic syndrome
a. Endometrial,cervical,breast - Endometrial cancer
b. Colon,pancreas,ovary - Ovarian cancer - 2-3 fold increase
c. endometrial , ovary
d. Skin, breast,pancreas

513. In the treatment of obese women with Weight reduction


PCOS, this is the initial recommendation For obese women with PCOS, lifestyle change and
because it reduces insulin, SHBG and antigen focused on diet and exercise are paramount to
levels and may restore ovulation. treatment at each stage of life. Even a modest
a. GNRH analogues amount of weight loss (5% of BW) can result in
b. Cyporterone acetate restoration of normal ovulatory cycles in some
c. Insulin sensitize women. This improvement result from reduction in
d. Weight reduction insulin and androgen levels, the latter mediated
e. Laparoscopic electrocautery through increases in SHBG levels.
Insulin sensitizing agents
Metformin: improves peripheral insulin sensitivity by
reducing hepatic glucose production and increasing
target tissues sensitivity to insulin.It also decreases
androgens in both lean and obese women, leading to
increased rates of spontaneous ovulation.
514. Rotterdam criteria for diagnosis of PCOS include the following EXCEPT
A. Insulin resistance
B. Menstrual irregularity
C. Clinical or biochemical hyperandrogenism
D. Ultrasound finding of more than 10 peripherally located cysts in the ovarian stroma
515. Nonhirsute women with PCOS have elevated levels of the ff androgens EXCEPT
A. Testosterone
B. DHEAS
C. 3a diol G (clinical manifestation: hirsutism)
D. None. All are elevated

516. Unopposed estrogen renders a patient with PCOS especially at risk for
A. Endometrial hyperplasia
B. B. Hyperandrogenism
C. C. Obesity (can cause insulin resistance)
D. Acanthosis nigricans (due to increase of insulin in the system)
517. For diagnosis of DM in PCOS patient which of the ff is least reliable
71. A. HbA1c
72. B. FBS
C. OGTT

“Never design your LIFE like a GARDEN Where anyone can walk, rather Design your LIFE like the SKY Where everyone desires
to REACH”

Special Credits to: | Darna | Totoro | LittleSnoozy | Alma | RatioTeam | RecallsTeam | Astral | SKCCSLAS | Katz
Disclaimer: Some of the items and answers here are not mine but was verified so, If you find discrepancies please feel free to
correct, this serves only as a guide for easy and efficient reviewing.
God bless!
62 | P a g e Philippians 4:13 jpmrnmd
GYNECOLOGY – Mastery Review
“Master your Ultra Instinct”
Ultimate Reviewer ~800 items for Major Exams | Revalida | Board Exams
518. The ff are long term complications Other complication; abnormal bleeding, cardiovascular
of PCOS, EXCEPT? disease, lipid abnormalities and hypertension
A. Ovarian cancer
B. Infertility
C. Cervical cancer
D. DM
519. Drug of choice in PCOS particularly useful for overweight and obese women to achieve better
metabolic control prior to pregnancy and for those who might have a more casual approach to their
fertility
A. Clomiphene (first drug for induction of ovulation)
B. Metformin
C. Letrozole ( alternative for clomiphene)
D. GnRH agonist
520. For failed medical management in PCOS a surgical alternative would be
A. Hysteroscopy
B. Oophorocystectomy
C. Oophorectomy
D. Ovarian diathermy (or ovarian drilling)
521. Hyperandogenismisclinicallymanifestedby the ff, EXCEPT
A. Increased hair growth in peripheral areas i.e. Upper and lower extremities
B. Acne
C. Male pattern hair distribution in a woman
D. Thinning of hair
522. In PCOS, Increased GnRH pulse amplitude or increased pituitary sensitivity to GnRH, the ff
abnormalities result in which of?
A. Tonically elevated levels of LH
B. Increased ovarian androgen production
C. Decreased S HBG-BC
D. AOTA
523. Which of the ff statements is correct about PAO or PCOM
A. A normal reproductive age woman with no symptoms or signs of PCOS but have PCO
B. Confers increased risk for other features of PCOS such as insulin resistance and cardiovascular risk
factors
C. May develop into a full blown PCOS
D. AOTA

“Never design your LIFE like a GARDEN Where anyone can walk, rather Design your LIFE like the SKY Where everyone desires
to REACH”

Special Credits to: | Darna | Totoro | LittleSnoozy | Alma | RatioTeam | RecallsTeam | Astral | SKCCSLAS | Katz
Disclaimer: Some of the items and answers here are not mine but was verified so, If you find discrepancies please feel free to
correct, this serves only as a guide for easy and efficient reviewing.
God bless!
63 | P a g e Philippians 4:13 jpmrnmd
GYNECOLOGY – Mastery Review
“Master your Ultra Instinct”
Ultimate Reviewer ~800 items for Major Exams | Revalida | Board Exams
MATCHING TYPE:
524. Thigh 1. Thigh → Vascular metastasis
Vascular metastasis
525. Ovary Hematogenous dissemination is the best theory to
Retrograde menstruation explain endometriosis of the forearm and thigh, as
526. Episiotomy well as multiple lesions in the lung
Iatrogenic 2. Ovary → Retrograde menstruation
527. Pelvic lymph node
Lymphatic metastasis Retrograde menstruation
528. Lung → most popular theory
Vascular metastasis → secondary to implantation of endometrial cells
shed during menstruation
A. Retrograde menstruation → most frequently discovered in areas immediately
B. Lymphatic metastasis adjacent to tubal ostia or in dependent areas of the
C. Vascular metastasis pelvis.
D. Iatrogenic 3. Episiotomy → Iatrogenic

Iatrogenic dissemination is the theory to explain the


episiotomy scar discovered in women after a
cesarean delivery
4. Pelvic LN → Lymphatic metastasis

Endometriosis is observed in the pelvic LN of


approx. 30% of women with the disease
5. Lung → Vascular metastasis

MATCHING TYPE: TREATMENT OF ENDOMETRIOSIS:


529. Danazol Two primary short term goals:
- Androgen ★ Relief of pain
530. Leuprolide ★ Promotion of fertility
- GNRH Agonist
531. Mefenamic acid Primary long term goal:
- NSAIDs
★ Prevent progression or recurrence of the disease
532. Diegonest
process
- Progestogen
533. Medroxyprogesterone acetate
1. Danazol → Androgen
- Progestogen
Side effects of Danazol:
A. NSAIDs
B. Progestogen ❏Hypoestrogenic and hyperandrogenic effect
C. GNRH Agonist ❏Atrophic changes in the endometrium
D. Androgen ❏Mild elevation of liver enzymes

2. Leuprolide → GNRH Agonist

“Never design your LIFE like a GARDEN Where anyone can walk, rather Design your LIFE like the SKY Where everyone desires
to REACH”

Special Credits to: | Darna | Totoro | LittleSnoozy | Alma | RatioTeam | RecallsTeam | Astral | SKCCSLAS | Katz
Disclaimer: Some of the items and answers here are not mine but was verified so, If you find discrepancies please feel free to
correct, this serves only as a guide for easy and efficient reviewing.
God bless!
64 | P a g e Philippians 4:13 jpmrnmd
GYNECOLOGY – Mastery Review
“Master your Ultra Instinct”
Ultimate Reviewer ~800 items for Major Exams | Revalida | Board Exams
Types of GNRH Agonist:
● Injectable - Leuprolide acetate (Lupron)
● Intranasal - Nafarelin acetate (Synarel)
● Subcutaneous implant - Goserelin acetate
(Zoladex)

Clinical use of GNRH agonists produces


“medical oophorectomy”
Side effects are similar to menopause: hot
flushes, vaginal dryness and insomnia
3. Mefenamic acid → NSAIDs

NSAIDs are beneficial for pain relief


4. Dienogest → Progestogen

Dienogest is a selective progestogen that


causes anovulation, has an antiproliferative
effect on endometrial cells, and may inhibit
cytokine secretion
5. Medroxyprogesterone acetate →
Progestogen

534. Endometriosis is the presence of growth of TRUE


endometrial gland and stroma in an aberrant or
heterotrophic location
535. The most common theory of etiology of TRUE
endometriosis is the retrograde menstruation
536. Cause of endometriosis in CS scar is by FALSE
vascular metastasis theory Iatrogenic Dissemination is the theory that explains
CS scar. While Vascular Metastasis is the best
theory to explain endometriosis of the forearm,
thigh, and lungs.
537. Natural Killer cells have decreased TRUE
cytotoxicity against endometrial and Also, peritoneal fluid of women with endometriosis
hematopoietic cells in women with has less influence of NK activity.
endometriosis
538. Classic signs of endometriosis are cyclic FALSE
pelvic pain and infertility These are SYMPTOMS not SIGNS. Other
symptoms include: secondary dysmenorrhea,
dyspareunia, and abnormal bleeding.
The classic pelvic finding of endometriosis (SIGN)
is a fixed retroverted uterus.

“Never design your LIFE like a GARDEN Where anyone can walk, rather Design your LIFE like the SKY Where everyone desires
to REACH”

Special Credits to: | Darna | Totoro | LittleSnoozy | Alma | RatioTeam | RecallsTeam | Astral | SKCCSLAS | Katz
Disclaimer: Some of the items and answers here are not mine but was verified so, If you find discrepancies please feel free to
correct, this serves only as a guide for easy and efficient reviewing.
God bless!
65 | P a g e Philippians 4:13 jpmrnmd
GYNECOLOGY – Mastery Review
“Master your Ultra Instinct”
Ultimate Reviewer ~800 items for Major Exams | Revalida | Board Exams
539. Extent of pelvic pain is dependent on FALSE
amount of endometriosis in female pelvis The extent of pelvic pain is often inversely related
to the amount of endometriosis.

540. Medical treatment of endometriosis aims to FALSE


suppress symptoms. Provide progression and Medical therapy usually suppresses
provide long term cure. symptomatology and prevents progression of
endometriosis but it does NOT provide long term
cure.
541. GnRH agonist when taken will cause TRUE
decrease in bone density “Add back” hormone therapy is combined with
chronic GnRH agonist regimen to overcome
demineralization of bone.

542. A 53 year old, who on inspection, there is A. Lichen Sclerosus


thinning of vulvar skin and white lesion on the -color white, skin thins out
labia majora -premalignant
A. Lichen Sclerosus -‘cigarette paper appearance’
B. Squamous Hyperplasia -drug allergies
C. Contact Dermatitis
D. Fungal Vaginitis
543. 25 yo female consulted with curd-like A. Fungal vaginitis : Candidiasis, trichomonas,
vaginal discharge. Vulva is slightly edematous bacterial vaginosis
and with excoriation. Candidiasis:
A. Fungal vaginitis White discharge + pruritus
B. Squamous Hyperplasia -Curd-like
C. Contact dermatitis -inflamed vulva
D.Lichen Sclerosus -scoration
544. 1 year old child with red perineum and C. Contact dermatitis
redness follows the diaper pattern -diaper pattern
A. Lichen sclerosus
B. Squamous Hyperplasia
C. Contact dermatitis
D. Fungal dermatitis
545. 66 year old female with non tender mass in B.urethral caruncle
the urethral orifice associated with urinary -vulvar lesion
urgency and frequency mChoices: -small, fleshy outgrowth of the distal end of the
A. cystocoele urethra
B.urethral caruncle -soft, smooth, bright red
C. Uterine prolapse -sessile or pedunculated -1-2 cm
-arise from ectropion of posterior urethral wall
-growth is secondary to chronic irritation or infection
Symptoms: asymptomatic
dysuria,frequency,urgency
Ulcerative lesion-spotting on contact

“Never design your LIFE like a GARDEN Where anyone can walk, rather Design your LIFE like the SKY Where everyone desires
to REACH”

Special Credits to: | Darna | Totoro | LittleSnoozy | Alma | RatioTeam | RecallsTeam | Astral | SKCCSLAS | Katz
Disclaimer: Some of the items and answers here are not mine but was verified so, If you find discrepancies please feel free to
correct, this serves only as a guide for easy and efficient reviewing.
God bless!
66 | P a g e Philippians 4:13 jpmrnmd
GYNECOLOGY – Mastery Review
“Master your Ultra Instinct”
Ultimate Reviewer ~800 items for Major Exams | Revalida | Board Exams
546. Drug induced Hemolytic Anemia B. Cephalosporin
A. Isoniazid Drug induced Hemolysis
B. Cephalosporin - Must be differentiated from the other hemolytic
C. Macrolides anemias
D. Sulfonamides - Milder hemolysis
- Resolves with drug withdrawal
- Penicillins and Cephalosporins
- Probenecid, rifampicin, quinidine and thiopenthal.

547. Vulvar lesion - premalignant? A. Dysplastic nevi


A. Dysplastic nevi -malignant transformation is 15x higher than
B. general population
C. Bartholins cyst -produces malignant melanoma
D. Endometriosis - >5 mm

548.Vaginal bleeding no laceration


a. sexual abuse,
b. foreign body
c, pinworm
d. vulvovaginitis
549. 23 year old Female presented with multiple c.genital warts
small non tender lesions on the vulva feels -multiple small non tender
rough to touch. pe on the Husband there were -rough to touch
similar lesions on the penis. probable diagnosis. -6,11(benign)
A. Herpes infect - very painful, small ulceration, -16,18(malignant)
multiple
b. Primary syph- Painless, heals on its own;
Treponema Pallidum
c.genital warts- HPV
d. c. Inguinale.
550. T/F :Prominent vulvar lesion which cause Pruritus in long term can lead to VIN (Squamous
prolonged pruritus is squamous hyperplasia Hyperplasia) then vulvar cancer
551.Postmenopausal, Cystocele TRUE
552. 30 y/o non-tender cyst at the anterior labia Skene’s Duct Cyst
majora?
12. 60 y/o G8P8, delivered at home, vagial Uterine prolapse
itroitus mass when walking

553. This tumor contains glycogen & hobnail Clear Cell (Mesonephroid) Tumor contain cells with
A. Mucinous tumor abundant glycogen and so called “hobnail cells”
B. Serous tumor
C. Clear cell
D. D. Epitheloid

“Never design your LIFE like a GARDEN Where anyone can walk, rather Design your LIFE like the SKY Where everyone desires
to REACH”

Special Credits to: | Darna | Totoro | LittleSnoozy | Alma | RatioTeam | RecallsTeam | Astral | SKCCSLAS | Katz
Disclaimer: Some of the items and answers here are not mine but was verified so, If you find discrepancies please feel free to
correct, this serves only as a guide for easy and efficient reviewing.
God bless!
67 | P a g e Philippians 4:13 jpmrnmd
GYNECOLOGY – Mastery Review
“Master your Ultra Instinct”
Ultimate Reviewer ~800 items for Major Exams | Revalida | Board Exams
554. Non-epithelial tumor that can grow into Mucinous Tumor - large and reach sizes of 30cm
humongous size and more.
A. Mucinous tumor Possible complications: Perforation & rupture
B. Serous tumor
C. Clear cell

D. Epitheloid
555. Psamomma bodies are commonly seen in Serous Cystadenoma
what tumor Gross :
A. Serous cystadenoma o Papiliary projections on the surface
B. Mucinous cystadenoma o Inner cyst wall mostly smooth •
C. Clear cell Microscopic :
D. Endometioid o Low columnar epithelium with occasional cilia
o Psammoma bodies
- small granules, end product of degeneration of
papillary implants
- indicative of functional immunologic

556. True of CA125 Specificity appears to be better for increased values


A. Commonly expressed in mucinous in post menopausal patients
tumor less frequently by mucinous tumor
B. Specificity is higher in post- >35 U/mL is considered increased
menopausal
C. Specific for serous malignant
epithelial ovarian tumor
D. All

557. According to FIGO Guidelines, what is the Stage IC = tumor either stage IA or IB but with
stage where it involvrs both ovaries, ruptured tumor on surface of one or both ovaries; or with
capsule and malignant cell capsule ruptured; or ascites; (+) peritoneal wash
A. 1B
B. 1C
C. IIA
D. IIB

558. Histologic type cervical malignancy that is Adenocarcinoma


not affected by the usual sexual factors but the - do not appear to be affected by usual sexual
risk of developing it is increased by the use of factors
OCPs - increased risk in:
A. Adenocarcinoma 1. HPV infection
B. Squamous cell Ca 2. Oral contraceptive use
C. Adenosquamous Ca 3. Lack of cervical cytologic screening
D. Glassy cell Ca
Squamous cell Ca
- affected by usual sexual factors
Adenosquamous Ca

“Never design your LIFE like a GARDEN Where anyone can walk, rather Design your LIFE like the SKY Where everyone desires
to REACH”

Special Credits to: | Darna | Totoro | LittleSnoozy | Alma | RatioTeam | RecallsTeam | Astral | SKCCSLAS | Katz
Disclaimer: Some of the items and answers here are not mine but was verified so, If you find discrepancies please feel free to
correct, this serves only as a guide for easy and efficient reviewing.
God bless!
68 | P a g e Philippians 4:13 jpmrnmd
GYNECOLOGY – Mastery Review
“Master your Ultra Instinct”
Ultimate Reviewer ~800 items for Major Exams | Revalida | Board Exams
- mixed carcinoma, consists of SqCa and AdenoCa
frequently found in PREGNANT women
Glassy Cell Ca
- mixed carcinoma
- undifferentiated tumor consisting of large cells
containing cytoplasm with a ground glass
appearance
- mets early to lymph node
559. Histologic type of cervical ca that is most Adenosquamous Ca
frequently seen in pregnant women - mixed carcinoma, consists of SqCa and AdenoCa
A. Squamous cell Ca frequently found in PREGNANT women
B. Adenosquamous Ca Adenocarcinoma
C. Adenocarcinoma - do not appear to be affected by usual sexual
D. Endometriod cervical Ca factors
- increased risk in:
HPV infection
Oral contraceptive use
Lack of cervical cytologic screening
Squamous cell Ca
- affected by usual sexual factors
Endometriod cervical Ca
- A variant of Adenocarcinoma
- mimics endometrium and resembles the
endocervix which contains little or no mucin
560. The finding of hydronephrosis in cervical ca is what stage?
A. III A
B. III B
C. III C
D. III A2

561. A clinically visible lesion limited to the cervix is


A. Stage 1-A2
B. Stage 1-B2
C. Stage II A
D. Stage II B

562. A barrel shaped cervix is associated with Endophytic


A. Verrucous carcinoma growth - Asymptomatic, delay in diagnosis
B. Exophytic growth - Barrel shaped cervix lesion in the endocervical
C. Endophytic growth canal
D. AOTA - Deeply invasive when diagnosed

Exophytic
- cauliflower -like appearance extruding from the
cervix

“Never design your LIFE like a GARDEN Where anyone can walk, rather Design your LIFE like the SKY Where everyone desires
to REACH”

Special Credits to: | Darna | Totoro | LittleSnoozy | Alma | RatioTeam | RecallsTeam | Astral | SKCCSLAS | Katz
Disclaimer: Some of the items and answers here are not mine but was verified so, If you find discrepancies please feel free to
correct, this serves only as a guide for easy and efficient reviewing.
God bless!
69 | P a g e Philippians 4:13 jpmrnmd
GYNECOLOGY – Mastery Review
“Master your Ultra Instinct”
Ultimate Reviewer ~800 items for Major Exams | Revalida | Board Exams
- Produces abnormal bleeding and staining

Verrucous carcinoma growth


- Warty tumors
- Appear in large bulbous mass
- Rarely metastasize

563. Biopsy of this node is frequently performed Distant Metastasis: Left supracervical & Left
in the assessment of advanced cervical scalene nodes
carcinoma to clarify whether the tumor has
spread outside the abdomen
A. Internal Iliac node
B. External Iliac node
C. Inguinal node
D. Left Scalene node
564. Mode of spread of cervical malignancy to Hematogenous - lung, liver and bone (less
the liver, lung and bone is via: frequently)
A. Hematogenous route Lymphatics
B. Lymphatics - Primary path of distant spread
C. Local infiltration - Obturator LN - most frequently involved node,
D. NOTA – Metastasis of cervical ca does not sentinel node
involve these sites Local infiltration
- Initially a locally infiltrating cancer from cervix →
vagina → paracervical and parametrial areas
565. Staging of cervical carcinoma is dependent Staging PRIMARILY DEPENDS ON PELVIC EXAM
primarily on which of the ff May also add:
A. Pelvic Exam General PE
B. Operative findings Chest X-ray
C. CT/MRI findings IVP
D. CXR findings CT scan
-is NOT changed based on Operative findings
566. A 28 y/o G1P0 is diagnosed to have Intraepithelial neoplasia or carcinoma in situ
carcinoma in situ at 20wks AOG. Management - Observed and delivered
plan is? - Final evaluation and therapy completed 6 weeks
A. Observe and deliver, with final evaluation after delivery
and therapy completed approx. 6 weeks after
delivery If carcinoma is diagnosed in the 1st trimester of
B. Immediate initiation of Teletherapy w/ early trimester (BEFORE 20 weeks)
subsequent brachytherapy ff abortion - Immediate treatment → delay → progression or
C. Immediate radical hysterectomy w/ pelvic spread
lymphadenectomy
D. Chemoradiation If diagnosed BEYOND 20 weeks of gestation
- Therapy is delayed until fetal viability (monitor
lung maturity)
- Delivery by CS; then
- Therapy is completed by surgery or radiation

Chemoradiation

“Never design your LIFE like a GARDEN Where anyone can walk, rather Design your LIFE like the SKY Where everyone desires
to REACH”

Special Credits to: | Darna | Totoro | LittleSnoozy | Alma | RatioTeam | RecallsTeam | Astral | SKCCSLAS | Katz
Disclaimer: Some of the items and answers here are not mine but was verified so, If you find discrepancies please feel free to
correct, this serves only as a guide for easy and efficient reviewing.
God bless!
70 | P a g e Philippians 4:13 jpmrnmd
GYNECOLOGY – Mastery Review
“Master your Ultra Instinct”
Ultimate Reviewer ~800 items for Major Exams | Revalida | Board Exams
- Stage IB2
- post -op chemoradiation improves survival of (+)
LN and margin
- Cisplatin-containing chemotherapy
- Reduce local and distant recurrence and
progression and improved

567. Meigs-Wertheim hysterectomy is also known as


A. Class I Hysterectomy
B. Class II Hysterectomy
C. Radical Hysterectomy
D. Modified radical hysterectomy

568. Which of the ffg risk factor is NOT associated C. Early and frequent sexual contact
with cervical adenocarcinoma?
A. HIV
B. OCP use
C. Early and frequent sexual contact
D. Lack of cytologic screening

569. How would you stage carcinoma of the B. By Clinical Findings


cervix? STAGING depends primarily on the PELVIC
A. By operative findings EXAMINATION
B. By clinical findings
C. By CT scan
D. By IVP

570. A young patient with Stage IB1 cervical D. Superiorly and laterally
carcinoma was considered to be at high risk for
adjuvant pelvic irradiation, where should the
ovaries be transposed to preserve ovarian
function?
A. Inferiorly and medially
B. Inferiorly and laterally
C. Superiorly and medially
D. Superiorly and laterally

571. Which of the ffg secondary node would D. Inguinal node


become a primary node when there is
involvement of the lower third of the vaginal in
cervical malignancy?
A. Common iliac
B. Obturator
C. Paracervical

“Never design your LIFE like a GARDEN Where anyone can walk, rather Design your LIFE like the SKY Where everyone desires
to REACH”

Special Credits to: | Darna | Totoro | LittleSnoozy | Alma | RatioTeam | RecallsTeam | Astral | SKCCSLAS | Katz
Disclaimer: Some of the items and answers here are not mine but was verified so, If you find discrepancies please feel free to
correct, this serves only as a guide for easy and efficient reviewing.
God bless!
71 | P a g e Philippians 4:13 jpmrnmd
GYNECOLOGY – Mastery Review
“Master your Ultra Instinct”
Ultimate Reviewer ~800 items for Major Exams | Revalida | Board Exams
D. Inguinal node

572. Which of the ffg is associated with D. Pelvic exenteration can be done on central
recurrence of cervical CA? pelvic recurrences for cure
A. Early central recurrences would
present as weight loss, malaise, and loss of
appetite
B. Most recurrences would develop within
1st 2 years
C. Recurrence of adenocarcinomas are
commonly seen at the vagina or cervix
D. Pelvic exenteration can be done on
central pelvic recurrences for cure

573. Which of the ffg. Is associated with Meigs- C. Uterine arteries are ligated at its origin from the
Wertheim Hysterectomy? hypogastric artery
A. ¾ of the vagina is resected
B. Uterosacral ligaments are ligated
midway towards their attachment to the sacrum
C. Uterine arteries are ligated at its
origin from the hypogastric artery
D. Used to manage patients with stage
IIB cervical cancer

574. Glassy cell CA Pregnant women


575. Adenoma Malignum Peutz-Jeghers syndrome
576. Clear cell Carcinoma DES exposure
577. Adenoid cystic Cylindroma of the salivary gland
carcinoma

578. Papsmear have shown to reduce the incidence Pap smear = yo reduce incidence of dse by at
of cervical cancer by at least 70% least 70%
A. True = ONLY screening procedure for Cervical
B. False cancer
Screening test: Pap test
Alternative: HPV testing
579. Early age of coitus is closely linked to the closely linked to sexual activity:
development of CIN Early age of first intercourse
A. True (together with multiple sex partner) multiple sexual partners
B. False
580. ACOG recommendation to start Pap testing is ACOG: recommendation fpr women age 21
at age 21 Frequency of the test remains controversial
A. True
B. False

“Never design your LIFE like a GARDEN Where anyone can walk, rather Design your LIFE like the SKY Where everyone desires
to REACH”

Special Credits to: | Darna | Totoro | LittleSnoozy | Alma | RatioTeam | RecallsTeam | Astral | SKCCSLAS | Katz
Disclaimer: Some of the items and answers here are not mine but was verified so, If you find discrepancies please feel free to
correct, this serves only as a guide for easy and efficient reviewing.
God bless!
72 | P a g e Philippians 4:13 jpmrnmd
GYNECOLOGY – Mastery Review
“Master your Ultra Instinct”
Ultimate Reviewer ~800 items for Major Exams | Revalida | Board Exams
581. Annual testing with cytology alone is Annual testing with cytology alone is acceptable
acceptable although not necessary for most women but not necessary For most women
A. True
B. False
582. When a woman undergoes hysterectomy for Current recommendation: Pap testing should be
myoma uteri, there is no need to perform pap test in stopped after having total hysterectomy
the future
A. True
B. False
583. Infection cannot be detected in Pap smear Pap Smear Test has low sensitivity thus = many
result false negatives
A. True used for screening ONLY of cervical ca
B. False
584. CIN II is consistent with Mild Dysplasia CIN I – Mild dysplasia
A. True CIN II – Moderate dysplasia
B. False CIN III – Severe dysplasia
585. Metaplasia is an abnormal process induced by it is a normal process occurring at the
trauma or hormonal changes transformation zone
A. True Normal transformation zone area where the
B. False columnar epithelium is replaced by squamous
epithelium
586. Persistent CIN I for more than 12 months in more than 24 months for women younger than
women younger than 21 years need to be treated 21, if older than 21y.o. its more than 12 months)
A. True
B. False
587. An endocervical margin that is positive is an Hysterectomy or immediate repeat excision is
indication for hysterectomy or immediate repeat only indicated for CIN remains for 4-6months
excision after the procedure
A. True
B. False (endocervical margin that is positive is
an indication for endocervical sampling and
coloscopy in 36months)

MATCHING TYPE: CIN I


588. LSIL
589. Severe dysplasia CIN III
590. HPV infection ALL ( cin I II III)
591. Moderate dysplasia CIN II
592. Carcinoma in situ CIN III
593. Squamous lesion but there are few cells present, Atypical Squamous cells of undetermined
not consistent with a more precise report significance (ASC-US)
594. Cells present with worrisome for significant Atypical Squamous cells, cannot exclude a
lesion, but are few in numbers higher grade lesion (ASC-H)
595. Women report often have CIN2 or CIN3 High grade Squamous intraepithelial lesion
(HSIL)

“Never design your LIFE like a GARDEN Where anyone can walk, rather Design your LIFE like the SKY Where everyone desires
to REACH”

Special Credits to: | Darna | Totoro | LittleSnoozy | Alma | RatioTeam | RecallsTeam | Astral | SKCCSLAS | Katz
Disclaimer: Some of the items and answers here are not mine but was verified so, If you find discrepancies please feel free to
correct, this serves only as a guide for easy and efficient reviewing.
God bless!
73 | P a g e Philippians 4:13 jpmrnmd
GYNECOLOGY – Mastery Review
“Master your Ultra Instinct”
Ultimate Reviewer ~800 items for Major Exams | Revalida | Board Exams
596. Consistent with histology report of CIN1, HPV, Low grade Squamous intraepithelial lesion
or mild dysplasia (LSIL)
597. Classified by site of origin Abnormal glandular lesions

598. Risk factor for developing endometrial ca, except?


A. Nulliparity
B. Obesity
C. HPN
D. Hypothyroidism
E. Postmenopausal

599. What stage is the px if the tumor invades the cervix


A. 1
B. 2
C. 3
D. 4

600. Surgical staging with endometrial CA is not possible, except?


A. Gross cervical involvement
B. Increased bleeding
C. More than ½ myometrial involvement
D. Distant mets

601. Mixture of sarcoma and carcinoma


602. What type of endometrial SIMPLE HYPERPLASIA
hyperplasia is considered to be weakly ● A term that defines endometrium with dilated glands
premalignant containing some outpouching and abundant stroma
A. Simple w/ atypia ● Cystic glands with round to irreg shapes, an increased
B. Simple w/o atypia crowding, no atypia
C. Complex w/ atypia ● Considered to be weakly premalignant
D. Complex w/o atypia

603. In postmenopausal women with ENDOMETRIAL THICKNESS: cut of 5mm


vaginal bleeding, which endometrial < 5 mm = no endometrial pathologic secondary to atrophic
stripe has almost 100% negative endometrium (thinning)
predictive value of finding and > 5mm = inc. chance of endometrial pathology, need further
endometrial patho? investigation
A. 2cm
B. 1cm
C. 5cm **(mm dapat ang unit
nito)
D. 0.4cm

“Never design your LIFE like a GARDEN Where anyone can walk, rather Design your LIFE like the SKY Where everyone desires
to REACH”

Special Credits to: | Darna | Totoro | LittleSnoozy | Alma | RatioTeam | RecallsTeam | Astral | SKCCSLAS | Katz
Disclaimer: Some of the items and answers here are not mine but was verified so, If you find discrepancies please feel free to
correct, this serves only as a guide for easy and efficient reviewing.
God bless!
74 | P a g e Philippians 4:13 jpmrnmd
GYNECOLOGY – Mastery Review
“Master your Ultra Instinct”
Ultimate Reviewer ~800 items for Major Exams | Revalida | Board Exams
604. 35 y/o G2P2 underwent For women with simple hyperplasia or complex hyperplasia
endometrial biopsy for abnormal WITHOUT atypia:
bleeding. Histopath shows complex ➔Low risk of developing endometrial ca, 1% and 3%
hyperplasia without atypia. What respectively
should be advised? ➔Diagnostic D&C can also be therapeutic
A. 50% chance to recur as
➔Progestins or combi OCPs will likely be effective
carcinoma in 5yrs
B. Perform hysterectomy
Cyclical progestogens should not be used because they are
C. Ovulation induction is
less effective in inducing regression of endometrial
contraindicated
hyperplasia without atypia compared with continuous oral
D. Cyclic progestin should
progestogens orthe LNG-IUS.
prompt withdrawal

605. A 53 y/o woman undergoes Adenocanthoma- squamous epithelium


curettage for postmenopausal Mucinous carcinoma - (+) intraplasmic mucin
bleeding. Histopath revealed The over-all microscopic picture is that of an
malignant glandular epithelium with adenocarcinoma of the endometrial glands. Sections of the
areas of metaplasia in endometrium. tumor show hyperplasia of the endometrial epithelium and in
What is the likely dx? some areas micro-cyst formation. These cells are more like
A. Atypical adenomatous the normal gland epithelium but have large, oval, open faced,
hyperplasia hypochromic nuclei; the cells tend to invade the
B. Adenocanthoma subendometrium.
C. Mucinous carcinoma
D. Endometrial ca

606. Endometrial ca invasion but without involvement of any structures. What FIGO classification.
A. IA
B. IB
C. IIA
D. IIB

607. The ff histologic type is assoc with poor Poor prognosis:


prognosis, except: ★ Clear cell ca
A. Endometrial ca ★ Papillary serous ca
B. Clear cell ca
C. Papillary serous ca Assoc with good prognosis:
D. Poorly differentiated
★ Simultaneous tumor of the endometrium
and ovary
★ Mucinous carcinoma

608. An ovarian tumor is often associated w/ excessive


estrogen production, w/c may cause pseudo precocious A. Granulosa- theca cell
puberty, post menopausal bleeding and menorrhagia.. tumor
A. granulosa - theca cell tumor
B. sertoli - leydig cell tumor Clinical manifestations:

“Never design your LIFE like a GARDEN Where anyone can walk, rather Design your LIFE like the SKY Where everyone desires
to REACH”

Special Credits to: | Darna | Totoro | LittleSnoozy | Alma | RatioTeam | RecallsTeam | Astral | SKCCSLAS | Katz
Disclaimer: Some of the items and answers here are not mine but was verified so, If you find discrepancies please feel free to
correct, this serves only as a guide for easy and efficient reviewing.
God bless!
75 | P a g e Philippians 4:13 jpmrnmd
GYNECOLOGY – Mastery Review
“Master your Ultra Instinct”
Ultimate Reviewer ~800 items for Major Exams | Revalida | Board Exams
C. gonadoblastoma Pubertal- 75% associated with sexual
D. immature teratoma pseudoprecocity
Reproductive age- menstrual
irregularity due to amenorrhea and
cystic hyperplasia
Postmenopausal- AUB
609. This non-epithelial ovarian tumor, histologically B. Sertoli - Leydig cell tumor
resembles fetal testis, producing significant amount of (highly malignant)
androgen mimicking tumor of adrenal glands 3rd-4th decade of life
A. granulosa - theca cell tumor Produces androgens and clinical
B. sertoli - leydig cell tumor virilization in 70 to 85%
C. gonadoblastoma - Testosterone, androsterone,
D. immature teratoma DHEAS

Micro: tubules were solid or hollow


-delicate septa occasionally seen
-the cells usually had pale to
occasionally densly eosinophilic
cytoplasms
610. Women w/ this tumor have very high level testosterone(>200ng/dl) and rapidly develop virilizing
characteristic such as temporal balding, clitoral hypertrophy, voice deepening and breast atrophy.
A. dysgerminoma
B. sertoli - leydig cell tumor
C. granulosa - theca cell tumor
D. endodermal sinus tumor

611. True regarding dysgerminoma. 75% occurs between ages 10 and 30 years
A. The peak age is over 45y/o 25% cases are metastatic sites include: bones, lungs, liver,
B. CA-125 is elevated in 50% of brain, mediastinum, supraclavicular lymph nodes
cases Mgt for chemotherapy AFP & Bhcg are tumor markers used for
C. they are mainly solid rather monitoring
than cystic in nature Gross appearance solid slightly bosselated capsuled mass
D. they can cause a rise in alpha-
fetoprotein
612. The cystic teratoma can undergo malignant transformation and estimated to occur in <2% of
these tumor, the origin of malignant proliferation usually occur in what area or portion of the dermoid
element
A. Endodermal Layer
B. Mesodermal Layer
C. Ectodermal Layer
D. Squamous epithelial component of the tumor
613. This is the most common primitive germ cell tumor,it accounts 20-30% of ovarian Ca encountered
during pregnancy and believed to be the counterpart of testicular seminoma in males.
A. endodermal sinus tumor
B. dysgerminoma
C. immature teratoma
D. androblastoma

“Never design your LIFE like a GARDEN Where anyone can walk, rather Design your LIFE like the SKY Where everyone desires
to REACH”

Special Credits to: | Darna | Totoro | LittleSnoozy | Alma | RatioTeam | RecallsTeam | Astral | SKCCSLAS | Katz
Disclaimer: Some of the items and answers here are not mine but was verified so, If you find discrepancies please feel free to
correct, this serves only as a guide for easy and efficient reviewing.
God bless!
76 | P a g e Philippians 4:13 jpmrnmd
GYNECOLOGY – Mastery Review
“Master your Ultra Instinct”
Ultimate Reviewer ~800 items for Major Exams | Revalida | Board Exams
614. Histologic typing of ovarian germ cell tumor of primary primitive germ cell tumor includes the
following
A. Non-gestational choriocarcinoma
B. Embryonal carcinoma
C. Immature teratoma
D. A & B
E. A, B, C
615. For cases of germ cell tumor part of the dx work-up is requesting for a karyotype obtained pre-
operatively for all premenarchial girls because of the propensity of these tumor to arise in dysgenetic
gonads.
A. True
B. False
616. Minimal surgical operation for ovarian dysgerminoma
A. Oophorocystectomy
B. Oophorectomy
C. TAHBSO
D. TAHBSO w/ BLND

617. Mixed germ cell malignancies of the ovary contains 2 Mixed germ cell tumors: Dysgerminoma
or more elements of the lesions, the most frequent + Endodermal Sinus tumor
combination of this malignancy is Immature Teratoma : consists of
A. Dysgerminoma & Immature teratoma immature embryonic structures admixed
B. Dysgerminoma & Endodermal sinus tumor with mature elements
C. Mature & Immature teratoma
D. Mature teratoma & Endodermal sinus tumor
618. Most important prognostic feature of immature teratoma
A. Stage of disease
B. Extent of tumor
C. Grading of lesions
D. degree of metastasis
619. True regarding endodermal sinus tumor of ovary
A. unilateral in 10%
B. Biopsy of Opposite of ovary in young is contraindicated
C. Most endodermal sinus tumor secrete AFP
D. A&C –
E. ABC
620. Mixed germ cell tumor malignancies of ovary contains Mixed germ cell tumors: Dysgerminoma
2 or more elements, most common combination + Endodermal Sinus tumor
A. Dysgerminoma and immature teratoma Immature Teratoma : consists of
B. Dysgerminoma and Endodermal sinus Tumor immature embryonic structures admixed
C. Mature and immature teratoma with mature elements
D. Mature teratoma and endodermal sinus tumor
621. In krukenberg tumor, the primary tumor is frequently PRIMARY TUMOR:
located in 1. Most common: STOMACH
A. Stomach 2. Less frequent: colon, breast biliary
B. Breast tract
C. Colon 3. Rare; cervix and bladder

“Never design your LIFE like a GARDEN Where anyone can walk, rather Design your LIFE like the SKY Where everyone desires
to REACH”

Special Credits to: | Darna | Totoro | LittleSnoozy | Alma | RatioTeam | RecallsTeam | Astral | SKCCSLAS | Katz
Disclaimer: Some of the items and answers here are not mine but was verified so, If you find discrepancies please feel free to
correct, this serves only as a guide for easy and efficient reviewing.
God bless!
77 | P a g e Philippians 4:13 jpmrnmd
GYNECOLOGY – Mastery Review
“Master your Ultra Instinct”
Ultimate Reviewer ~800 items for Major Exams | Revalida | Board Exams
D. Biliary tract Usually bilateral

622. Destruction of pituitary cells due to hypotension NOT related to pregnancy


A. Sheehans
B. Simmonds
C. Celiac
D. Asherman
623. This lesion is extremely rare and accounts fo less Sertoli Leydig Tumor
than .2% of ovarian cancer Typically produce androgen - 3rd to 4th decade of life
and clinical virilization in 70-80% - <0.2% of ovarian cancers
A. Granulosa Stromal cell Ca - Produces androgens and clinical
B. Sertoli Leydig Tumor virilization in 70 to 85% testosterone,
C. Gyandrogenblastoma androstenedione, DHES
D. Krukenberg - Low grade, poorly differentiated
- <1% cases are bilateral

Granulosa stromal cell tumor


- 75% associated with sexual
pseudoprecocity
- Menstrual irregularity due to
amenorrhea and cystic hyperplasia
- (+) callexner bodies
- The nucleus is connect finely granular
or hyperchromic with a groove
resembling “coffee bean”

Krukenberg tumor
- 30%-40% metastatic cancer to ovaries
- Arises in the ovarian stroma
- Composed of mucin filled, Signet-Ring
Cells

624. malignant change in benign cystic teratoma has been recorded occuring 0.5 % to 2% of the
cases, the most common malignant teratoma is???
A. Adenocarcinoma
B. AdenoSquamous Carcinoma
C. Squamous CA
D. Androblastoma
625. These lesion is seen in premenarchail girls with pseudopuberty and with elevated AFP and HCG
A. Choriocarinoma of the ovary
B. Polyembryonal CA
C. Embryonal CA
D. Mixed germ cell tumor
626. Ovulation can be induced 90-95% of anovulation patient except those elevated
A. FSH
B. LH

“Never design your LIFE like a GARDEN Where anyone can walk, rather Design your LIFE like the SKY Where everyone desires
to REACH”

Special Credits to: | Darna | Totoro | LittleSnoozy | Alma | RatioTeam | RecallsTeam | Astral | SKCCSLAS | Katz
Disclaimer: Some of the items and answers here are not mine but was verified so, If you find discrepancies please feel free to
correct, this serves only as a guide for easy and efficient reviewing.
God bless!
78 | P a g e Philippians 4:13 jpmrnmd
GYNECOLOGY – Mastery Review
“Master your Ultra Instinct”
Ultimate Reviewer ~800 items for Major Exams | Revalida | Board Exams
C. Estrogen
D. Progesterone
627. f the patient is asked to come back for follow up, when should it be
A. 3months after
B. 6months after
C.12 months after
D.Never
628. The risk of malignant transformation in benign cystic teratoma is markedly increased in this age
group of patient
A. Pre-menarchal
B. Reproductive age group women
C. Pre-menopausal
D. Post-menopausal (but malignant transformation occur in this age group)
629. The malignant germ cell tumors are analogous to seminoma in the male testes
A. Dysgerminoma
B. Yolk sac tumor
C. Embryonal CA (lack of trophoblast)
D. Mixed germ cell tumor (combination of dysgerminoma + EST)
630. The specific tumor markers if Endodermal Sinus Tumor used in identifying and follow-up the
course of the tumor clinically is
A. hCG (choriocarcinoma)
B. Placental alkaline phosphatase ( dysgerminoma)
C. Alpha feto protein
D. Lactic dehydrogenase ( dysgerminoma)
631. The tumor of the ovary that consists or contained both malignant cytotrophoblasts and
syncitiotrophoblasts and hCG is a useful tumor marker
A. Gestational choriocarcinoma
B. Embryonal CA
C. Mixed germ cell tumor
D. Choriocarcinoma
632. The most frequent sites of origin of tumor metastatic to the ovary is
A. Breast
B. GIT (stomach)
C. Lungs
D. Lower reproductive organs

633. In pre-adolescent females both benign and malignant ovarian tumors are usually unilateral,
routine biopsy of the contralateral ovary should be avoided EXCEPT for cases of
A. Immature teratomas (bilateral)
B. Endodermal sinus tumor
C. Dysgerminoma (bilateral)
D. A & B only
E. A & C only

“Never design your LIFE like a GARDEN Where anyone can walk, rather Design your LIFE like the SKY Where everyone desires
to REACH”

Special Credits to: | Darna | Totoro | LittleSnoozy | Alma | RatioTeam | RecallsTeam | Astral | SKCCSLAS | Katz
Disclaimer: Some of the items and answers here are not mine but was verified so, If you find discrepancies please feel free to
correct, this serves only as a guide for easy and efficient reviewing.
God bless!
79 | P a g e Philippians 4:13 jpmrnmd
GYNECOLOGY – Mastery Review
“Master your Ultra Instinct”
Ultimate Reviewer ~800 items for Major Exams | Revalida | Board Exams
634. Dermoids are usually asymptomatic in most cases, a rare complication of this tumor possess risk
for pregnant patient with this tumor, presenting with severe abdominal pain that mimics uterine
contraction is
A. Ovarian cyst torsion ( and sebaceous material perforate the cyst wall)
Dermoids = teratoma
B. Hemorrhagic formation of the cyst
C. Perforation leading to reacting peritonitis
D. Tumor previa
635. This tumor secretes alpha fetoprotein, which is a specific marker useful for identifying and
following up this tumor clinically
A. Dysgerminoma (PAP + LDH)
B. Immature teratomas ( no marker)
C. Endodermal sinus tumor
D. Embryonal CA (AFP+HCG)
636. True regarding Struma ovarii
A. Considered as one of the two specialized germ cell tumors
B. Contains dermoid or tissues of immature tereatoms
C. The thyroid tissue confined can be functional
D. A & C only
E. AOTA
637. This type of ovarian tumors are considered sex cord-stromal germ cell tumors that usually arise in
dys____ gonads in patient with Y chromoses
A. Granulosa cell tumor (inhibin is the marker)
B. Gonadoblastoma
C. Gynandroblastoma
D. Leydic cell and Hilus cell tumor

638. The absence of menses regardless of PRIMARY AMENORRHEA


presence of secondary sexual ★ Absence of menses and secondary sexual
characteristics by age of 16 is: characteristics by age 14
A. Primary amenorrhea ★ Absence of menses regardless of presence of
B. Secondary amenorrhea secondary sexual characteristics by age 16
C. Precocious puberty
D. Menopause SECONDARY AMENORRHEA
★ In a woman who has been menstruating
★ Amenorrhea of a total of at least 3 previous cycles or
6 months of amenorrhea
★ Absence of menses for an arbitrary time period -> 6
to 12 months

639. Pertains to the onset of female breast THELARCHE →onset of female breast development
development RUBARCHE →appearance of sexual hair
A. Menarche ADRENARCHE →onset of androgen-dependent body
B. Thelarche changes such as growth of axillary and pubic hair, body
C. Adrenarche odor and acne
D. Rubarche MENARCHE →onset of menstruation

“Never design your LIFE like a GARDEN Where anyone can walk, rather Design your LIFE like the SKY Where everyone desires
to REACH”

Special Credits to: | Darna | Totoro | LittleSnoozy | Alma | RatioTeam | RecallsTeam | Astral | SKCCSLAS | Katz
Disclaimer: Some of the items and answers here are not mine but was verified so, If you find discrepancies please feel free to
correct, this serves only as a guide for easy and efficient reviewing.
God bless!
80 | P a g e Philippians 4:13 jpmrnmd
GYNECOLOGY – Mastery Review
“Master your Ultra Instinct”
Ultimate Reviewer ~800 items for Major Exams | Revalida | Board Exams
640. Compartment 1 outflow obstruction Compartment 1: “The OUTFLOW TRACT”
which is affected ★ Uterine agenesis
A. Ovary ★ Testicular feminization
B. Uterus
★ Imperforate hymen
C. Pituitary
D. Hypothalamus ★ Asherman’s syndrome
★ Radiotherapy

Compartment 2: “The OVARIES”


★ Polycystic ovary syndrome
★ Ovarian failure (premature menopause)
★ Resistant overy syndrome
★ Ovarian agenesis/ dysgenesis
★ Chemo/ radiotherapy
★ Surgery

Compartment 3: “The ANTERIOR PITUITARY GLAND”


★ Tumor
➔Prolactinoma
➔Non-functional tumor
★ Infarction
➔Sheehan’s syndrome
★ Infection
➔Tuberculosis
★ Radiotherapy
★ Surgery

Compartment 4: “The HYPOTHALAMUS”


★ Congenital
➔Kallman’s syndrome
★ Acquired
➔Pituitary stalk disconnection syndromes
(craniopharyngioma)
➔Excessive weight loss
➔Extreme exercise
➔Cranial radiotherapy

641. Outflow tract obstruction leading to primary Compartment 1: “The OUTFLOW TRACT”
amenorrhea ★ Uterine agenesis
A. Perforated hymen **Imperforate yata ★ Testicular feminization
dapat ito
★ Imperforate hymen
B. Transversed vaginal septum
C. Hydro and/or pyosalpinx ★ Asherman’s syndrome
D. Cervicitis ★ Radiotherapy

“Never design your LIFE like a GARDEN Where anyone can walk, rather Design your LIFE like the SKY Where everyone desires
to REACH”

Special Credits to: | Darna | Totoro | LittleSnoozy | Alma | RatioTeam | RecallsTeam | Astral | SKCCSLAS | Katz
Disclaimer: Some of the items and answers here are not mine but was verified so, If you find discrepancies please feel free to
correct, this serves only as a guide for easy and efficient reviewing.
God bless!
81 | P a g e Philippians 4:13 jpmrnmd
GYNECOLOGY – Mastery Review
“Master your Ultra Instinct”
Ultimate Reviewer ~800 items for Major Exams | Revalida | Board Exams

642. The singer, Ms. Caren Carpenters, once had ANOREXIA NERVOSA is the most common
an extreme weight loss and distorted implaceable cause of amenorrhea
attitude towards eating. This psychiatric condition
is associated with amenorrhea is known as
A. Anorexia Nervosa
B. Bulimia
C. Both
D. Neither

643. Ischemia of pituitary gland leading to Compartment 3: “The ANTERIOR PITUITARY


necrosis of pituitary cells secondary to postpartum GLAND”
hemorrhage ★ Tumor
A. Sheehan’s syndrome ➔Prolactinoma
B. Simmond syndrome
➔Non-functional tumor
C. Asherman syndrome
D. Celiac disease ★ Infarction
➔Sheehan’s syndrome
★ Infection
➔Tuberculosis
★ Radiotherapy
★ Surgery

644. Congenital abnormality involving the Compartment 4: “The HYPOTHALAMUS”


hypothalamus causing idiopathic ★ Congenital
hypogonadotropic hypogonads with anosmia ➔Kallman’s syndrome
A. Kallman syndrome
★ Acquired
B. Sheehan’s syndrome
C. PCOS ➔Pituitary stalk disconnection syndromes
D. Asherman syndrome (craniopharyngioma)
➔Excessive weight loss
➔Extreme exercise
➔Cranial radiotherapy

645. The Mayer-Rokitansky-Kuster-Hauser Mullerian Agenesis (Mayer-Rokitansky-Kuster-


Syndrome is Hauser
A. Mullerian agenesis Syndrome)
B. Urogenital agenesis ● 10% of primary amenorrhea
C. Ovarian agenesis ● Normal secondary sexual characteristics
D. AOTA ● Absent vagina and cervix with normal or
rudimentary uterus →amenorrhea or
endometriosis
● 46,XX

“Never design your LIFE like a GARDEN Where anyone can walk, rather Design your LIFE like the SKY Where everyone desires
to REACH”

Special Credits to: | Darna | Totoro | LittleSnoozy | Alma | RatioTeam | RecallsTeam | Astral | SKCCSLAS | Katz
Disclaimer: Some of the items and answers here are not mine but was verified so, If you find discrepancies please feel free to
correct, this serves only as a guide for easy and efficient reviewing.
God bless!
82 | P a g e Philippians 4:13 jpmrnmd
GYNECOLOGY – Mastery Review
“Master your Ultra Instinct”
Ultimate Reviewer ~800 items for Major Exams | Revalida | Board Exams
646. In case of Premature Ovarian Insufficiency, Premature Ovarian failure
which is correct? ● Hypergonadotropic hypogonadism
A. Hypergonadotropic hypogonadism ● Dx: persistently high FSH values in menopausal
B. Ovaries cease to produce sufficient range
progesterone to stimulate endometrial growth
C. Occurs mostly before the onset of
physiologic menopause
D. Hypogonadotropic hypogonadism

647. Patient diagnosed with Asherman Syndrome Asherman’s syndrome (Uterine synechiae)
A. Whole abdomen utz ● Intrauterine adhesions (IUAs)
B. Hysterogram ● Diagnosis: HSG, hysteroscopy, cultures
C. Pelvic xray ● Treatment: ○ Hysteroscopic resection (scissors,
D. Pregnancy test laser, cautery)
○ Foley catheter
○ Progestogen + estrogen

648. The most important and probably most common cause of amenorrhea in adolescent girls is
A. PCOS
B. Anorexia Nervosa
C. Premature ovarian failure
D. Asherman’s syndrome
649. Gonadal failure is characterized by
A. Primary amenorrhea
B. X chromosome abnormality
C. Shorter than 63 inches in height
D. AOTA
650. The statement “Uterovaginal agenesis is The most common cause of primary amenorrhea is
the second most common cause of primary Gonadal failure 50%
amenorrhea is (15% of the cases) Most common cause of secondary amenorrhea is
A. True hypothalamic dysfunction
B. False
651. Caused by gonadal failure and hypogonadic hypogonadism is best established with measurement
of serum
A. FSH levels (it must be hypergonadotropic hypogonadism and gonadal failure) >30 mIU/ml
B. Prl
C. LH levels
D. Estradiol
652. The most frequent cause of intrauterine adhesion is
A. Curettage performed during pregnancy or shortly thereafter
B. Septic abortion secondary to catherization
C. PID
D. AOTA

“Never design your LIFE like a GARDEN Where anyone can walk, rather Design your LIFE like the SKY Where everyone desires
to REACH”

Special Credits to: | Darna | Totoro | LittleSnoozy | Alma | RatioTeam | RecallsTeam | Astral | SKCCSLAS | Katz
Disclaimer: Some of the items and answers here are not mine but was verified so, If you find discrepancies please feel free to
correct, this serves only as a guide for easy and efficient reviewing.
God bless!
83 | P a g e Philippians 4:13 jpmrnmd
GYNECOLOGY – Mastery Review
“Master your Ultra Instinct”
Ultimate Reviewer ~800 items for Major Exams | Revalida | Board Exams
653. When women lose 15% below ideal body 25% below ideal body weight – pituitary
weight, amenorrhea can occur because of gonadotropin dysfunction or abnormality
dysfxn at the
A. CNS-hypothalamic
B. Anterior pituitary
C. Ovaries
D. Uterus
654. When weight loss decreases below ___ of ideal body weight, pituitary gonadotrophism functions
can become abnormal
A. 15%
B. 20%
C. 25%
D. 30%
655. Individuals with anorexia nervosa have impaired peripheral conversion of T4 to T3, resulting in
normal T4 levels , decreased T3 levels, and increased reverse T3 levels
A. True
B. False
656. When uterine bleeding fails to occur after progestin is administered, E2 levels are:
A. Higher than 40 pg/mL
B. Lower than 40 pg/mL (<30-40pg/ml)
C. Normal
D. NOTA
657. Pituitary causes of amenorrhea can be associated with ACTH elevation and TSH deficiency
A. True
B. False

Matching Type:
A. Pap smear
B. Gram stain
C. KOH smear
D. NSS smear

658. An 18yo nulligravid, with several sexual B. Gram stain


partners, complaining of profuse yellow-green the patient is suffering from gonococcal infection
vaginal discharge ● yellow green and profuse vaginal discharge
● Gram stain - for Gonorrhea, Bacteria vaginosis

659. A 35.yo G1P1, postpartum for 1 year, A. Pap smear


coming for regular check-up. She has been she is prone of developing endometrial hyperplasia
taking OCP for 6 months due to OCP intake and postpartum state
660. A 45 yo complaing of profuse white curd- C. KOH Smear
like discharge which is causing intense pruritus Candidiasis
A. Pap smear ● fungal infection causing itching and vaginal
B. Gramstain discharge.
C. KOH smear ● The discharge is often thick, white and, curd-like
D. NSS smear (almost like cottage cheese)
● KOH smear - for fungal infection

“Never design your LIFE like a GARDEN Where anyone can walk, rather Design your LIFE like the SKY Where everyone desires
to REACH”

Special Credits to: | Darna | Totoro | LittleSnoozy | Alma | RatioTeam | RecallsTeam | Astral | SKCCSLAS | Katz
Disclaimer: Some of the items and answers here are not mine but was verified so, If you find discrepancies please feel free to
correct, this serves only as a guide for easy and efficient reviewing.
God bless!
84 | P a g e Philippians 4:13 jpmrnmd
GYNECOLOGY – Mastery Review
“Master your Ultra Instinct”
Ultimate Reviewer ~800 items for Major Exams | Revalida | Board Exams

Matching Type:
A. Cryotherapy
B. Colposcopy
C. Endometrial Biopsy
D. Cervical Biopsy

661. 45yo, with regular, profused, prolonged C. Endometrial biopsy


vaginal bleeding for the past 3 months Based on the age of the patient and she has
abnormal bleeding
Endometrial Biopsy
● Done in women with abnormal uterine bleeding,
sampling and histologic evaluation for hyperplasia
or neoplasia
● ACOG (2000) recommends for endometrial
assessment in any women older than 35 years old
with abnormalities
● Younger than 35 years who are suspected of
having annovulatory uterine bleeding refractory to
medical management

Indications for Endometrial Biopsy


● AUB/post-menopausal bleeding
● Endometrial dating
● Follow-up of previously diagnosed endometrial
hyperplasia
● Evaluation of patient’s with 1 yr. amenorrhea
● Evaluation of infertility
● Pap smear with atypical cells favoring endometrial
origin

662. 25yo, with Pap smear result showing CIN a. Cryotherapy, most commonly used for patient
II with CIN
Cryotherapy should be considered acceptable
therapy
when the following criteria are met:
• Cervical Intraepithelial Neoplasia, grade 1 to 2
• Small lesion
• Ectocervical location only
• Negative endocervical sample
• No endocervical gland involvement or biopsy
663. 33yo, with colposcopy finding of a focal a. Cryotherapy
area of CIN II on the anterior lip of the cervix
measuring 5x5

“Never design your LIFE like a GARDEN Where anyone can walk, rather Design your LIFE like the SKY Where everyone desires
to REACH”

Special Credits to: | Darna | Totoro | LittleSnoozy | Alma | RatioTeam | RecallsTeam | Astral | SKCCSLAS | Katz
Disclaimer: Some of the items and answers here are not mine but was verified so, If you find discrepancies please feel free to
correct, this serves only as a guide for easy and efficient reviewing.
God bless!
85 | P a g e Philippians 4:13 jpmrnmd
GYNECOLOGY – Mastery Review
“Master your Ultra Instinct”
Ultimate Reviewer ~800 items for Major Exams | Revalida | Board Exams
664. 27yo, with aceto-white epithelium on D. Cervical biopsy, you should get 2-3mm sample
routine vaginal inspection with acetic acid when patient has abnormal colposcopic findings
Abnormal findings:
- acetowhite epithelium
- Punctuation
- Mosaic
- Leukoplakia
- Abnormal blood vessels

665. 51yo, Postmenopause for 2 years C. Endometrial biopsy, based on the age of the
complaining of vaginal spotting, and with finding patient and she has polyps which is an indication
of endometrial polyps on TVS for endometrial biopsy
666. 18yo with findings of punctuation on D. Cervical Biopsy
colposcopy (+) Abnormal colposcopic findings do biopsy -
A. Cryotherapy obtain tissue sample 2-3 mm in size
B. Colposcopy
C. Endometrial Biopsy
D. Cervical Biopsy
667. A 63 year old with fungating friable mass D. Cervical Biopsy
on the posterior lip of the cervix

668. Solid ovarian tumor in pregnancy Luteoma of pregnancy


elaborating androgen - Unilateral / Bilateral SOLID enlargement
A. Luteoma
B. Hyperreactio lutelnalis Hyperreactio luteinalis
- Bilateral CYSTIC enlargement

669. Common ovarian tumor in postmenopause Hilus cell tumors


A. Sertoli Leydig Tumor - After menopause
B. Hilus Tumor - Small and NOT palpable
- TX: TAHBSO

Sertoli Leydig Cell tumors


- Develop during reproductive age
- Unilateral palpable tumor
- TX: Unilateral salpingooophorectomy

670. Associated w/ idiopathic hirsutism Diagnosis:


A. Increase testosterone ● Normal Testosterone
B. High levels of 3𝞪-diolG ● Normal Androgen
● Increased 3a-diol-G in 80% of women
● Increased levels of 5a reductase

671. Higher level of peripheral conversion of Testosterone is not elevated —-> but peripherally
testosterone converted
A. DHEAS Diagnosis:
B. Androstenedione ● Increased DHEAS (>8mg/ml)

“Never design your LIFE like a GARDEN Where anyone can walk, rather Design your LIFE like the SKY Where everyone desires
to REACH”

Special Credits to: | Darna | Totoro | LittleSnoozy | Alma | RatioTeam | RecallsTeam | Astral | SKCCSLAS | Katz
Disclaimer: Some of the items and answers here are not mine but was verified so, If you find discrepancies please feel free to
correct, this serves only as a guide for easy and efficient reviewing.
God bless!
86 | P a g e Philippians 4:13 jpmrnmd
GYNECOLOGY – Mastery Review
“Master your Ultra Instinct”
Ultimate Reviewer ~800 items for Major Exams | Revalida | Board Exams

672. Adrenal tumors assay Testosterone is not elevated —-> but peripherally
A. DHEAS = 12ng/ml converted
B. Testosterone = 2ng/ml Diagnosis:
● Increased DHEAS (>8mg/ml)

673. Can cause female fetus masculinization CAH


A. Luteoma - Congenital condition: causing female fetus
B. CAH masculinization
- Severe Form: causes sexual ambiguity
- Mild Form: manifest late until after puberty

674. Short statured Late Onset 21-Hydroxylase deficiency (LOHD)


A. LOHD - Does not produce physical signs until after
B. Cushing’s syndrome puberty
- S/Sx - Menstrual irregularity
- Anovulation (after menarche)
- History of prepubertal accelerated growth (6-8y)
with later decreased growth
- Short ultimate height (12-13y)

Cushing’s syndrome - increased ACTH secretion


(C.disease)
- S/Sx: Hirsuitism
- Menstrual irregularity
- Central obesity
- Dorsal neck fat pads
- Abdominal striae
- Muscle wasting and weakness

675. Elaborates both androgen and estrogen OVARIAN NEOPLASMS


A. Lipid cell tumor 1. Germ Cell Tumors
B Granulosa/Theca cell tumor ● Sertoli Leydig Cell tumors
● Hilus cell tumor

2. Lipoid
● Adrenal rest tumors

3. Granulosa / Theca Cell Tumors


● Increased ANDROGEN + ESTRADIOL

676. Familial or constitutional disorder IDIOPATHIC HIRSUTISM


A. LOHD ● Peripheral disorder of androgen metabolism
B. Idiopathic hirsutism ● AKA —-> Familial or Constitutional Hirsutism

“Never design your LIFE like a GARDEN Where anyone can walk, rather Design your LIFE like the SKY Where everyone desires
to REACH”

Special Credits to: | Darna | Totoro | LittleSnoozy | Alma | RatioTeam | RecallsTeam | Astral | SKCCSLAS | Katz
Disclaimer: Some of the items and answers here are not mine but was verified so, If you find discrepancies please feel free to
correct, this serves only as a guide for easy and efficient reviewing.
God bless!
87 | P a g e Philippians 4:13 jpmrnmd
GYNECOLOGY – Mastery Review
“Master your Ultra Instinct”
Ultimate Reviewer ~800 items for Major Exams | Revalida | Board Exams
677. Phases of hair development dependent on Phases of Hair Growth
the circulating androgen level 1. Anagen
A. Telogen ● Growth phase of hair**
B. Anogen ● Circulating Androgen

2. Catagen
● Transitional phase

3. Telogen
● Resting Phase
● Terminal hair —> Shedding**

MATCHING TYPE
A. subcapsular cyst in the ovary
B. elaborates androgen and estradiol
C. decreased growth at puberty
D. central obesity
E. increased level of 3 a diol G
678. Idiopathic hirsutism. E. Increased level of 3a-diol-G
679. Granulosa/theca lutein cysts B. elaborates androgen and estradiol
680. Cushing's Disease D. central obesity
681. LOHD C. decreased growth at puberty
682. PCOS A. subcapsular cyst in the ovary
683. Idiopathic hirsutism – familial or constitutional hirsutism it is a
peripheral disorder of androgen metabolism
684. Pheochromocytoma – secrets catecholamines ( skin sensation, flank
pain, increase in heart rate and BP)
685. Cushing’s syndrome – Central obesity, hirsutism, dorsal fat pads,
abdominal straie, fatigue, muscle wasting and
weakness = clinical features
686. LOHD – congenital hyperplasia >>> menstrual
irregularities
687. Addisons disease - TB, primary adrenal insufficiency and
hypocorticolism that is triggered by stress, injury,
infection and surgery
MATCHING TYPE
A. histologically appears as nest of theca cells
B. solid, palpable ovarian tumor at 35 y/o
C. small, non-palpable ovaries in post menopause
D. bilateral cystic enlargement of the ovaries in pregnancy
E. bilateral solid ovarian enlargement in pregnancy
688.Hyperreactio Luteinalis D. bilateral cystic enlargement of the ovaries in
pregnancy
689. Hilus tumor C. small, non-palpable ovaries in post menopause

“Never design your LIFE like a GARDEN Where anyone can walk, rather Design your LIFE like the SKY Where everyone desires
to REACH”

Special Credits to: | Darna | Totoro | LittleSnoozy | Alma | RatioTeam | RecallsTeam | Astral | SKCCSLAS | Katz
Disclaimer: Some of the items and answers here are not mine but was verified so, If you find discrepancies please feel free to
correct, this serves only as a guide for easy and efficient reviewing.
God bless!
88 | P a g e Philippians 4:13 jpmrnmd
GYNECOLOGY – Mastery Review
“Master your Ultra Instinct”
Ultimate Reviewer ~800 items for Major Exams | Revalida | Board Exams
690. Luteoma E. bilateral solid ovarian enlargement in
pregnancy
691. Sertoli/Leydig cell tumor B. solid, palpable ovarian tumor at 35 y/o
692. Stromal hyperthecosis A. histologically appears as nest of theca cells
693. Hyperreactio luteinalis – bilateral ovarian enlargement during pregnancy
due to theca lutein cyst associated with inc HCG
694. Luteoma - tumor of the ovaries due to inc. progesterone
and testosterone
695. Primary aldosteronism – Conn’s syndrome>>> poor vision and
headaches

696. Associated with infection above the waist. HSV-1 infects epithelium above the waist
A. HSV-1 HSV-2 causes ulceration below the waist
B. HSV-2 (Although clinically HSV-2 can appear in the
C. Both mouth and breast)
D. Neither
697. Known to be protective to other HSV?? infection affords some protection against HSV-1
A. HSV1
B. HSV2
C. BOTH
D. NEITHER
698. Serologic screening for syphilis used as an nonspecific nontreponemal test NSNT
index of treatment VDRL & RPR - screening tests
A. Rpr Specific antitreponemal antibody test SAAT
B. Fta-abs more sensitive but may also produce false (+)
C. Both results in cases of Lupus erythematous
D. Neither Standard test use: treponema immobilization test
(TP)
FTA-ABS & MHA-TP
699. Regression of primary syphilis heals within 2-6 weeks
A.1-4 weeks
B. 2-6 weeks
C.3-8 weeks
D. 4-10 weeks
700. Tumor cells with abundant glycogen and hobnail cell
Ans: Clear-cell
701. Type of tumor that resembles transitional epithelium
Ans: Brenner
702. CA-125 is elevated if the value is?
Ans. >35 U/mL
703. Incubation period of chancroid
A. 1-3 days
B. 3-6 days
C. 6-9 days
D. 9-11 days
ANS. B

“Never design your LIFE like a GARDEN Where anyone can walk, rather Design your LIFE like the SKY Where everyone desires
to REACH”

Special Credits to: | Darna | Totoro | LittleSnoozy | Alma | RatioTeam | RecallsTeam | Astral | SKCCSLAS | Katz
Disclaimer: Some of the items and answers here are not mine but was verified so, If you find discrepancies please feel free to
correct, this serves only as a guide for easy and efficient reviewing.
God bless!
89 | P a g e Philippians 4:13 jpmrnmd
GYNECOLOGY – Mastery Review
“Master your Ultra Instinct”
Ultimate Reviewer ~800 items for Major Exams | Revalida | Board Exams
704. Untreated chancroid is associated in the within 2 weeks if untreated develop acutely tender
development of bubbo within inguinal adenopathy (bubo) = 50%
A. 1 wk
B. 2 wks
C. 3 wks
D. 4 wks

705. HSV Virus associated with the lips


A. HSV 1
B. HSV 2
C. both
D. Neither
706. Nonspecific test for syphilis Syphilis testing can be divided into two
A. Vdrl categories. Treponemal assays (FTA, syphilis
B. Rpr IgG) measure antibodies that directly react with
C. Both the syphilis-causing organism T. pallidum, while
D. NOTA non-treponemal assays (RPR, VDRL) measure
antibodies against non-specific cardiolipin
antigens released during treponemal infections.

707. Most common area of involvement of cases LYMPHOGRANULOMA VENEREUM


of Lymph granuloma Venerum. Chronic infection of lymphatic tissue
A. Cervix Etiology: chlamydia trachomatis
B. Urethra Involves the vulva and other parts
C. Rectum INCUBATION PERIOD: 3-30 days
D. Vulva
708. drug of choice for granuloma inguinale GRANULOMA INGUINALE
a.acyclovir Management
b. tetracycline o Doxycycline 100mg BIDx3 weeks
c.doxycycline
d. doxycycline
709. Donovan bodies GRANULOMA INGUINALE
• • Donovanosis
• • Not highly contagious
• • Incubation period: 1-2 weeks
• • Etiology: K. granulomatis
• • Chronic ulcerative
• • Clinical presentation:

o Nodule
o Becoming painless
o Slowly progressing ulcer
o Surrounded by a highly vascular granulation
tissue with no adenopathies
o Untreated characterized by
- Scarring
- Lymphatic obstruction

“Never design your LIFE like a GARDEN Where anyone can walk, rather Design your LIFE like the SKY Where everyone desires
to REACH”

Special Credits to: | Darna | Totoro | LittleSnoozy | Alma | RatioTeam | RecallsTeam | Astral | SKCCSLAS | Katz
Disclaimer: Some of the items and answers here are not mine but was verified so, If you find discrepancies please feel free to
correct, this serves only as a guide for easy and efficient reviewing.
God bless!
90 | P a g e Philippians 4:13 jpmrnmd
GYNECOLOGY – Mastery Review
“Master your Ultra Instinct”
Ultimate Reviewer ~800 items for Major Exams | Revalida | Board Exams
- Vulvar enlargement

710. Ulcer as having foul smelling exudates CHANCROID


Clinical presentation
o Papule evolves into a pustule before ulceration
o Painful papule
o Ulcers described as shallow, rugged edge and
foul smelling
o Occurs in the
Vulvar vestibule
o If untreated within 2 weeks, may develop a bubo

“Never design your LIFE like a GARDEN Where anyone can walk, rather Design your LIFE like the SKY Where everyone desires
to REACH”

Special Credits to: | Darna | Totoro | LittleSnoozy | Alma | RatioTeam | RecallsTeam | Astral | SKCCSLAS | Katz
Disclaimer: Some of the items and answers here are not mine but was verified so, If you find discrepancies please feel free to
correct, this serves only as a guide for easy and efficient reviewing.
God bless!
91 | P a g e Philippians 4:13 jpmrnmd
GYNECOLOGY – Mastery Review
“Master your Ultra Instinct”
Ultimate Reviewer ~800 items for Major Exams | Revalida | Board Exams
711. 28y/o G1P1 with a history of OCP use d. A and B
e. AOTA
 Papsmear: Recommended for all
women starting age 21.  Rationale: Ch 7 Reproductive
Physiology, p. 142 – GnrH is unique
712. This use is for small focal CIN lesions and among releasing hormones in that it
act to kill by freezing intracellular water: simultaneously regulates the secretion
of 2 hormones – FSH & LH. It also is
 Cryotherapy: most commonly used unique among the body’s hormones
treatment for CIN lesions because it must be secreted in a
pulsatile fashion.
713. Patient discharge, the ff asked except
717. Which of the ff inhibits the release of
 Certain food or something-- doesnt prolactin secretion:
directly cause strong vaginal odor A. EGF
B. VIP
714: clinical breast examination? C. Gamma amino butyric acid
D. Disease of dopamine secretion
 Identify ca in women who are not E. All of the above
typically candidates for
mammography--- best at <50 >30 in  Rationale: Dopamine – previously
young women ultrasound are known as prolactin-inhibiting factor –
preffered inhibits its secretion
Stimuli that can elicit release of
715. The following is/are true of GnRH agonist? Prolactin:
 Breast manipulation
A. Stimulate the natural molecule in its  Drugs
intermittent effect on the gonadotrophins  Stress
B. Leads to persistent activation of GnRH  Exercise
receptor  Certain food
C. Increase GnRH receptor Hormones that may stimulate Prolactin
D. All of the above release
 Vasopressin
 Rationale: Ch7, p. 143 – Used clinically,  GABA
GnRH agonists are modifications of the
 B-endorphin
native molecule to either the increase
 VIP
receptor affinity or decrease regulation.
Their use leads to a persistent activation  Angiotensin II
of GnRH receptors.  TRH

716. Unique feature of GnRH 718. Which of the following is/are ture of
follicular phase of the ovarian cycle?
a. It regulates LH and FSH secretion
simultaneously A. Estrogen is secreted in increasing
b. It must be secreted in a pulsatile manner quantities by growing follicles
to be effective B. FSH begins to wane by midpoint of the cycle
c. GnRH has long half life C. LH initially decreases but increases
dramatically in late follicular phase

“Never design your LIFE like a GARDEN Where anyone can walk, rather Design your LIFE like the SKY Where everyone desires
to REACH”

Special Credits to: | Darna | Totoro | LittleSnoozy | Alma | RatioTeam | RecallsTeam | Astral | SKCCSLAS | Katz
Disclaimer: Some of the items and answers here are not mine but was verified so, If you find discrepancies please feel free to
correct, this serves only as a guide for easy and efficient reviewing.
God bless!
92 | P a g e Philippians 4:13 jpmrnmd
GYNECOLOGY – Mastery Review
“Master your Ultra Instinct”
Ultimate Reviewer ~800 items for Major Exams | Revalida | Board Exams
D. AOTA hypertrophy, take up increasing
E. A&C amounts of lipids, and acquire
organelles associated with
 Rationale: steroidogenesis. The hallmark of the
FOLLICULAR PHASE – promotes the human corpus luteum is its secretion
development of a single dominant primarily of progesterone.
follicle. Last for 10-14 days. (comprehensive gyne chapter 4 page
PRIMORDIAL PHASE 88)
 FSH assumes control of follicular
differentiation and growth 720. What phase of endometrial cycle is
 1ST sign of follicular phase : GROWTH characterized by progressive mitotic growth,
OF OOCYTE & MULTILAYERING OF dense stroma and initially straight narrow, short
THE GRANULOSA CELLS endometrial gland that grow into tortous
PRE-ANTRALL FOLLICLE structure?
 Zona pellucida appears
 granulosa continues to proliferate A. Proliferative phase
 theca cells in stroma near granulosa B. Early secretory phase
cells also proliferate C. Late secretory phase
 both cells > estrogen D. Premenstrual phase
PREOVULATORY FOLLICLE
 has a fluid filled antrum  Rationale:
Synthesis of DNA is increased, and
 Oocyte connected via cumulus
mitoses are numerous.Toward the late
oophorus
follicular phase, the straight glands
 LH SURGE effects:
become progressively more voluminous
o Luteinization of the granulosa
and tortuous. (Comprehensive gyne
cells
chapter 4 page 90)
o Progesterone production
o Ovulation
721. Involves premature maturation of HPO axis
OVULATION
that has normal menses, ovulation and
 LH SURGE CAUSES: possibility of pregnancy.
o Inc. prostaglandin and
proteolytic enzymes in the A. GnRH dependent precocious puberty-it
follicular wall involves prematurity maturation of the
o Weakening of the wall hypothalamic-pituitary-ovarian axis and includes
o 10-12 HOURS after LH SURGE normal menses, ovulation and the possibility of
pregnancy
719. Hormone that subsequently rises after B. GnRH independent precocious puberty- it
ovulation has occured involves premature female sexual maturation,
which may lead to estrogen-induced uterine
A. Estrogen stimulation and bleeding without any normal
B. Progesterone ovarian follicular activity.
C. Both C. Heterosexual precocious puberty- secondary
D. Neither sex characteristics are discordant with the
genetic phenotypic gender, this is a premature
 Rationale: virilization in the female child and includes
The corpus luteum is the result of two development of masculine secondary sexual
important events initiated at ovulation. characteristics
First, granulosa and theca cells

“Never design your LIFE like a GARDEN Where anyone can walk, rather Design your LIFE like the SKY Where everyone desires
to REACH”

Special Credits to: | Darna | Totoro | LittleSnoozy | Alma | RatioTeam | RecallsTeam | Astral | SKCCSLAS | Katz
Disclaimer: Some of the items and answers here are not mine but was verified so, If you find discrepancies please feel free to
correct, this serves only as a guide for easy and efficient reviewing.
God bless!
93 | P a g e Philippians 4:13 jpmrnmd
GYNECOLOGY – Mastery Review
“Master your Ultra Instinct”
Ultimate Reviewer ~800 items for Major Exams | Revalida | Board Exams
D. none  Rationale:
 Source: compre gyne page 830 Key points (pp 836)
Breast hyperplasia is a NORMAL phenomenon in
722. The most common GnRH independent neonate and may persists up to 6 MONTHS of
precocious puberty? age.

A Granulosa cell tumor 725. A 32 y/o, F, was diagnosed with non-


B theca cell tumor proliferative secretory tumor. It is best managed
C sertoli cell tumor by:
D leydig cell tumor A.Excision
B.Wt. loss
 Rationale: C. DMPA therapy
The most common cause of pseudo D. Estrogen Replacement
precocious puberty is an estrogen
secreting ovarian tumor. Granulosa cell
tumors are the most common type of  Rationale:
ovarian tumora acooounting for Excision (Non–prolactin-secreting
approximately 60% pituitary tumors should be surgically
excised, if possible. Those who have
723. A life threatening condition… lost weight should be advised to gain
weight. If strenuous exercise results in
A. Anorexia low estrogen levels (<30 pg/mL), the
B. Bulimia amount of exercise should be reduced
C. Both A and B or estrogen supplementation
D. neither administered to prevent possible
development of osteoporosis.)- Compre-
 Rationale: Gyne P828
Anorexia is a serious, potentially life
threatening eating disorder characterized 726. Secondary Ammenorhea: estradiol normal
by self starvation and EXCESSIVE and pelvic UTZ normal
WEIGHT LOSS.
Bulimia is an eating disorder A. PCO
characterized by binge eating combined B. Hyperandrogenism
with inappropriate compensatory C. POG
mechanisms to avoid weight gain. Men D. Hypothalamic Disorder
and women with bulimia are usually
NORMAL WEIGHT or SLIGHTLY
OVERWEIGHT. Being underweight while  Rationale:
purging might indicate a purging type of Hypothalamic Disorder - If a sensitive
anorexia. serum E2 value is above 30 to 40
pg/mL, and ultrasound confirms the
724. T/F breast hyperplasia is an abnormal presence of polycystic ovaries, the
phenomenon in neonates and persists up to 16 diagnosis of PCOS may be considered.
months of age. If there is no sonographic evidence of
polycystic ovaries and the woman has a
 False history of drug ingestion, stress, weight
loss, or strenuous exercise, she should
be told that hypothalamic-pituitary

“Never design your LIFE like a GARDEN Where anyone can walk, rather Design your LIFE like the SKY Where everyone desires
to REACH”

Special Credits to: | Darna | Totoro | LittleSnoozy | Alma | RatioTeam | RecallsTeam | Astral | SKCCSLAS | Katz
Disclaimer: Some of the items and answers here are not mine but was verified so, If you find discrepancies please feel free to
correct, this serves only as a guide for easy and efficient reviewing.
God bless!
94 | P a g e Philippians 4:13 jpmrnmd
GYNECOLOGY – Mastery Review
“Master your Ultra Instinct”
Ultimate Reviewer ~800 items for Major Exams | Revalida | Board Exams
dysfunction is present and the exact unrelated to pregnancy, it is called
cause cannot be determined with Simmond’s disease page826
current technology because frequent LH
sampling is costly and impractical. She 729. Risk factor for endometrial CA
should also be informed that
hypothalamic-pituitary dysfunction is a. premature ovarian failure
usually a self-limiting disorder and not a b. menopause hormone replacement therapy
serious threat to health or a cause of c. OCP
untreatable infertility. d. Polycystic ovarian disease
 Women with low E2 and low FSH levels
have a CNS lesion or hypothalamic-  Rationale: Menopause hormone
pituitary failure. Women with low E2 and replacement therapy. More historical
than clinically relevant, unopposed
elevated FSH levels (>30 mIU/mL) have
estrogen stimulation is strongly
POI.
associated with endometrial cancer,
increasing the risk by four to eight times
727. Estradiol <30-40ng/dl. Cause?
for a woman using estrogen alone for
A. PCO
menopausal replacement therapy. The
B. Hyperandrogensim
risk increases with higher doses of
C. POG
estrogen (>0.625 mg conjugated
D. Hypothalamic disorder
estrogens), and more prolonged use but
can be markedly reduced with the use of
 Rationale: Hypothalamic disorder
progestin. Similarly, combination
Ratio: when sufficient GnRH is produced
(progestin containing) oral
to facilitate gonadotropin stimulation of
contraceptives decrease the risk. As
the ovaries producing E2 levels
noted by Grimes and Economy,
sufficient to proliferate the endometrium
combination oral contraceptives protect
(30-40 pg/ml), the term is hypothamalic-
against endometrial cancer, with most
pituitary dysfunction is used to
studies showing a relative
characterize this disorder page 825
risk reduction to approximately 0.5. The
protection begins after 1 year of use and
728. Secondary amenorrhea due to necrosis of
lasts approximately 15 years after
the pituitary cells due to hypotensive episode
discontinuation. Other conditions leading
related to postpartum hemorrhage
to long-term estrogen stimulation of the
endometrium, including the polycystic
a. Sheehan’s syndrome
ovary syndrome (Stein-Leventhal
b. simmond’s disease
syndrome) and the much more rare
c. Asherman’s syndrome
feminizing ovarian tumors, are also
d. Celiac disease
associated with increased risk of
endometrial carcinoma (Comprehensive
 Rationale: Sheehans - Pituitary cells can
Gynecology 6th ed., 2012., p. 713).
also become damaged or necrotic as a
result of anoxia, thrombosis or
730. What type of endometrial hyperplasia is
hemorrhage. When pituitary cell
considered to be weakly
destruction occurs as a result of a
premalignant
hypertensive episode during pregnancy,
the disorder is called SHEEHAN’S
A. Simple with atypia
SYNDROME. When a disorder is
B. Simple without atypia

“Never design your LIFE like a GARDEN Where anyone can walk, rather Design your LIFE like the SKY Where everyone desires
to REACH”

Special Credits to: | Darna | Totoro | LittleSnoozy | Alma | RatioTeam | RecallsTeam | Astral | SKCCSLAS | Katz
Disclaimer: Some of the items and answers here are not mine but was verified so, If you find discrepancies please feel free to
correct, this serves only as a guide for easy and efficient reviewing.
God bless!
95 | P a g e Philippians 4:13 jpmrnmd
GYNECOLOGY – Mastery Review
“Master your Ultra Instinct”
Ultimate Reviewer ~800 items for Major Exams | Revalida | Board Exams
C. Complex with atypia B. Perform hysterectomy
D. Complex without atypia Page 717: Older patients with moderate
or severe atypical hyperplasia generally require
 Rationale: Simple without atypia. The hysterectomy. In addition, those who fail
term cystic hyperplasia has been used progestin therapy, and especially those with
to describe dilation of the endometrial severe cytologic atypia, should also be
glands, which often occurs in a considered for hysterectomy. So dapat progestin
hyperplastic endometrium in a therapy muna then if that doesn’t work proceed
menopausal or postmenopausal woman with your hysterectomy
(cystic atrophy). It is considered to be
weakly premalignant (Comprehensive C. Ovulation induction is contraindicated
Gynecology 6th ed., 2012., p. 715). Page 717: Studies have shown that
Simple hyperplasia had a 1% rate of younger patients with chronic anovulation and
progression to cancer, complex hyperplasia who desire children may also be
hyperplasia without atypia had a 3% rate treated by induction of ovulation with clomiphene
of progression to cancer, and complex citrate. Since 35 y/o na si patient and may
atypical hyperplasia had a 29% rate of dalawa na syang anak, hindi na applicable tong
progression to cancer. option na to

731. In postmenopausal women with vaginal D. Cyclic progestin should prompt withdrawal
bleeding, which endometrial stripe has almost Page 718 Figure 32-5: According sa
100% negative predictive value of finding an diagram, pag under reproductive age ka pa and
endometrial pathology? meron kang complex endometrial atypia,
A. 2cm continuous high-dose progestin therapy ang
B. 1cm intervention mo
C. 5cm
D. 0.4cm 733. Classic cause of acute PID is:

 Rationale: Page 717: In A. E. coli


postmenopausal women with any B. C. trachomatis
vaginal bleeding, Gull and colleagues C. group B streptococcus
have found that an endometrial stripe D. Pneumoccocus
less than 4mm has a 100% negative
predictive value. A finding of endometrial  Rationale: Trachomatis and N.
thickness less than 4mm is a reasonable gonorrhea comprises more than 90% of
predictor of lack of endometrial cases. They can coexist in the same
pathology, even in postmenopausal individual. Chlamydia is more prevalent.
woman with bleeding  ***C. trachomatis- smelly fishy odor,
cause PID even without symptoms,
732. 35y/o G2P2 underwent endometrial biopsy atypical
for abnormal bleeding. Histopath shows complex  N. gonorhea-yellowish-greenish copious
hyperplasia without atypia. What should be discharge
advised?

A. 50% chance to occur as carcinoma in 5 years


Page 717: 3% and 29% yung nasa
book. Basta walang 50% dun

“Never design your LIFE like a GARDEN Where anyone can walk, rather Design your LIFE like the SKY Where everyone desires
to REACH”

Special Credits to: | Darna | Totoro | LittleSnoozy | Alma | RatioTeam | RecallsTeam | Astral | SKCCSLAS | Katz
Disclaimer: Some of the items and answers here are not mine but was verified so, If you find discrepancies please feel free to
correct, this serves only as a guide for easy and efficient reviewing.
God bless!
96 | P a g e Philippians 4:13 jpmrnmd
GYNECOLOGY – Mastery Review
“Master your Ultra Instinct”
Ultimate Reviewer ~800 items for Major Exams | Revalida | Board Exams
734. Salpingitis is the most common • laparoscopic abnormalities
characteristic and component in PID. consisted with PID (GOLD STANDARD)

 True 738. Procedure associated with PID

 Rationale: Many authors prefer the term a. pap smear


SALPINGITIS because infection of the b. speculum
oviducts is the most characteristic and c. internal examination
common component of PID. d. Dilation and curettage
 The terms acute SALPINGITIS and
PELVIC INFLAMMATORY DISEASE  Rationale: Comprehensive gynecology
are used synonymously to describe an 6E 2012 page 544 - 15% of acute PID
acute infection. (Compre gyne by Katz develop following procedures that break
chapter 23, page 544) the cervical mucus barrier, allowing
vaginal flora that opportunity to colonize
735. 90% of the cases is associated with that upper genital tract. These
ascending infection procedures include endometrial biopsy,
curettage, IUD insertion,
 False hysterosalpingography, and
hysteroscopy.
 Rational: Acute PID results from
ascending infection from the bacterial 739. Fits-hugh-curtis syndrome is more likely to
flora of the vagina and cervix in MORE be present in PID caused by:
THAN 99% of cases. (Compre. Gyne by
Katz chapter 23, page 544) a. N. Gonorrhea
 *actually, the answer can also be A. b. C. trachomatis
True…pero kasi nasa book MORE c. Both
THAN 99%... d. Neither

736. PID is extremely rare in women with  Rationale: Comprehensive gynecology


amenorrhea. 6E 2012 page 546 - C. Trahomatis may
lead to an autoimmune response that
A. TRUE causes sevre tubal damage, even if C.
B. FALSE trachomatis is no longer present.
Immunologically sensitized studies have
737. Most accurate in diagnosing PID demonstrated that women with
antibodies to Chamydial heat shock
A. Endometrial biopsy protein are more likely to have severe
B. MRI showing fluid in cul de sac tubal damage and Fitz-hugh Curtis
C. Lap direct visualization of infection syndrome.
D. (+) culture
740. Delayed treatment of PID for___ hours may
 Definitive diagnostic criteria: endometrial lead to infertility or ectopic pregnancy.
biopsy with histopathologic evidence of
endometritis transvaginal sonography or A.>24
MRI showing thick fluid filled tubes B.>36
C.>48
D.>72

“Never design your LIFE like a GARDEN Where anyone can walk, rather Design your LIFE like the SKY Where everyone desires
to REACH”

Special Credits to: | Darna | Totoro | LittleSnoozy | Alma | RatioTeam | RecallsTeam | Astral | SKCCSLAS | Katz
Disclaimer: Some of the items and answers here are not mine but was verified so, If you find discrepancies please feel free to
correct, this serves only as a guide for easy and efficient reviewing.
God bless!
97 | P a g e Philippians 4:13 jpmrnmd
GYNECOLOGY – Mastery Review
“Master your Ultra Instinct”
Ultimate Reviewer ~800 items for Major Exams | Revalida | Board Exams
 Rationale: pp 864
 Rationale: Women who are not treated  Metformin had been used as a first-line
in the first 72 hours following the onset treatment for infer- tility, although not all
of symptoms are three times as likely to women with PCOS will respond more re-
develop tubal infertility or ectopic cent randomized trials with a focus on
pregnancy as those whobare treated live births as an end point have
early in the disease process. suggested that clomiphene is superior to
Page 553 (compre gyne) metformin for first - line therapy.

741. Anaerobic infection is best treated with:  Clomiphene has been the mainstay for
ovulation induction. Most pregnancies
A. Cefotetan-Doxycycline occur within the first few cycles.
B. Cefoxitin-Doxycycline Accordingly, it is reasonable to use
C. Clindamycin-Gentamicin clomiphene, with or without metformin,
D. Ampicillin-sulbactam-Doxycycline as an initial approach, after obtaining a
semen analysis, but not for more than
 Rationale: C.Clindamycin-Gentamicin three or four ovulatory cycles before a
Compre Gyne page555 more compre- hensive workup is
Regimen Bbis a combination of undertaken. Letrozole (2.5 to 5 mg/day,
clindamycin and gentamicin 5 days) has proved to be efficacious as
(aminoglycoside). It has the advantage an alternative to clomiphene, and is
of providing excellent coverage for particularly suited for women who have
anaerobic infections and facultative side effects with clomiphene. However,
gram negative rods. there are no long-term data to date.
 Ampicillin-sulbactam plus doxycycline is
an alternative inpatient regimens and 744. 1st line of treatment for skin disorder
they have excellent anaerobic coverage secondary to hyperandrogenism in PCOS.
and would be a good choice for women
with with tubo ovarian complex. The A. Flutamide
alternative regimen has less extensive B. Ketoconazole
clinical trials. C. GnRH agonist + Estrogen
 Cefoxitin and doxycycline provide D. Non-androgenic progestin in an OC
excellent coverage for N.gonorrhoeae,
C.trachomatis and penicillinase-  P865. Non androgenic progestogen (e.g
producing N.gonorrhoeae desogestrel, norgetimate, drosperinone)
in an OC, in combination with
742. First line for ovulation induction? spironolactone (100mg to 200mg), is
suggested as 1st line treatment.
 CLOMIPHENE CITRATE
745. Treatment for metab sx in pcos: ocp insulin,
743. Patient who has adverse effect in diet+ metformin, bariatric surgery
clomiphene, the alternative drug
A. Metformin  Diet+metformin
B. OCP
C. Letrozole  Rationale: P866. Metabolic syndrome
D. Spironolactone treated by diet and metformin. 6-
12month therapy has been shown to
reduce weight 5-7% as well as reduce

“Never design your LIFE like a GARDEN Where anyone can walk, rather Design your LIFE like the SKY Where everyone desires
to REACH”

Special Credits to: | Darna | Totoro | LittleSnoozy | Alma | RatioTeam | RecallsTeam | Astral | SKCCSLAS | Katz
Disclaimer: Some of the items and answers here are not mine but was verified so, If you find discrepancies please feel free to
correct, this serves only as a guide for easy and efficient reviewing.
God bless!
98 | P a g e Philippians 4:13 jpmrnmd
GYNECOLOGY – Mastery Review
“Master your Ultra Instinct”
Ultimate Reviewer ~800 items for Major Exams | Revalida | Board Exams
insulin resitance and improve metabolic C. Previous CS
parameters. D. Smoking

746. Although therapy for a woman with PCOS  Rationale: pagte 801
should be directed at woman specific complaint Cigarette Smoking. The relative risk of
which of the ff should be the mainstay of placenta previa is increased at least
treatment twofold in women who smoke (Ananth,
2003a; Usta, 2005). It has been
A. Improvement of lifestyle variables postulated that carbon monoxide
including weight reduction and fitness hypoxemia causes compensatory
B. Metformin placental hypertrophy and more surface
C. OCP area. Smoking may also be related to
D. Finasteride decidual vasculopathy that has been
implicated in the genesis of previa.
 Rationale: COMPRE GYNE CH40 pg.
864 - Before ovulation induction, it is 749. Ovarian testosterone is converted to
necessary to normalize overt estrogen through:
abnormalities in glucose tolerance and
to encourage weight loss if the BMI is  Aromatization
excessive >28
750. Pcos is also known as:
747. PCOS is worst in obese women due to?
 Stein and Leventhal
A. Infection
B. Insulin resistance  Rationale: PCOS was originally
C. Acne described in 935 by Stein and Leventhal
D. Hirsutism as a syndrome consisting amenorrhea,
hirsutism, and obesity in association
 Rationale: COMPRE GYNE CH40 pg with enlarged polycystic ovaries. (page
858- Women with PCOS have 853)
characteristic lipid and lipoprotein
abnormalities including the presence of 751. Histologic Type of ____ associated with
abnormal lipoprotein particles which add poor prognosis and high rate of relapse and
to a long list of abnormalities that then to metastasis.
increase cardiovascular risk, including
hypertension and diabetes as women A. Adenocarcinoma
approach menopause, these risks B. AdenoSquamous
pertain to a women with more classic C. Squamous cell CA
feature of PCOS particularly obesity. D. All have equal prognosis.
 Obesity is one of the major factors
leading to these risks and and  Rationale: page 662 tends to
development of metabolic syndrome. metastasize early to lymp nodes as well
as to distant sites, fatal outcome.
748. Risk factor for placenta previa because of
placental hypertrophy is due to?

A. Increase maternal age


B. Multiparity

“Never design your LIFE like a GARDEN Where anyone can walk, rather Design your LIFE like the SKY Where everyone desires
to REACH”

Special Credits to: | Darna | Totoro | LittleSnoozy | Alma | RatioTeam | RecallsTeam | Astral | SKCCSLAS | Katz
Disclaimer: Some of the items and answers here are not mine but was verified so, If you find discrepancies please feel free to
correct, this serves only as a guide for easy and efficient reviewing.
God bless!
99 | P a g e Philippians 4:13 jpmrnmd
GYNECOLOGY – Mastery Review
“Master your Ultra Instinct”
Ultimate Reviewer ~800 items for Major Exams | Revalida | Board Exams
752. Most common chromosomal abnormality  Chapter 28, page 653 compre gyne
among anembryonic conceptus. (lentz, lobo, gershenson)

A. Trisomy 755. ACOG recommendation for cervical cancer


B. Monosomy screening for age:
C. Single gene defect
D. Balanced translocation. A. 12
B. 18
753. Tthe most effective cervical screening C. 21
Method ever developed
 Rationale: in 2009 American College of
a. Colposcopy Obstetriticans and Gynecologists
b. Pap test (ACOG) practice Bulletin has
c. Visualization on acetic acid recommended Pap testing for all women
d. Mammography beginning at age 21- chapter 28, page
654 compre gyne (lentz, lobo,
 Rationale: The cervical cytology(pap gershenson,)
test) became available in many
developed countries in the 1950s after 756. This/these may be the reason for an
the studies of Papaniculaou had shown unsatisfactory result in The Bethesda System
that by examining a properly prepared
and stained cellular sample scraped A. Lack of label
from uterine cervix, the presence of B. Loss of transport medium
cancer and its precursors could be C. Scant cellularity
identified. The 1941, monograph D. All of the above
Papaniculaou and Traut remains one of
the breakthroughs in the history of  Rationale: Reasons for unsatisfactory
preventive medicin.-- Chapter 28, page report include such items as lack of
654 compre gyne (lentz, lobo, labe, loss of transport medium, scant
gershenson,katz) cellularity, and contamination by foreign
material -chapter 28, page 655 compre
754. Whent is became possible to discover this gyne (lentz, lobo, gershenson)
organism, the true cause of cervical CA is:
757. 25 years old, G1P1, cytology report (+)
a. chalymidia Atypical Cells of undetermined significance,
b. HSV what is the next dx procedure?
c. HPV
d. Gonorrhea a. HPV DNA
b. Repeat Pap Smear
 Rationalee: Although some studies c. Colposcopy
found associations with herpes simplex d. ALL
virus, Chlamydia and gonorrheal
infections, all these were ultimately
discarded. It was only when it became
possible to identify HUMAN
PAPILLOMA VIRUS (HPV) infection that
the true cause of cervical cancer was  Rationale: Management plan is
discovered. immediate colposcopy, repaeat cytology

“Never design your LIFE like a GARDEN Where anyone can walk, rather Design your LIFE like the SKY Where everyone desires
to REACH”

Special Credits to: | Darna | Totoro | LittleSnoozy | Alma | RatioTeam | RecallsTeam | Astral | SKCCSLAS | Katz
Disclaimer: Some of the items and answers here are not mine but was verified so, If you find discrepancies please feel free to
correct, this serves only as a guide for easy and efficient reviewing.
God bless!
100 | P a g e Philippians 4:13 jpmrnmd
GYNECOLOGY – Mastery Review
“Master your Ultra Instinct”
Ultimate Reviewer ~800 items for Major Exams | Revalida | Board Exams
in 4- 6 months, and do HPV DNA “After……… The predominant cause of DUB in
testing. the post……..in women with anovulatory DUB,
there is unopposed estradiol production without
758. 19 years old, undergo colposurgically corpus luteum formation and progesterone
guided biopsy. Result is CIN 1 production……..uniform slouching does not occur
with contributes to excessive uterine blood flow”
a. cryotherapy
b. thermoablation
c. excision 761. The pathophysiology of osteoporosis are
d. none the following except?

 Ratioanle: CIN 1 presents with mild A. low dietary calcium


dysplasia, most cases disappear B. high estrogen
spontaneously within weeks or months. C. aging
It should not be regarded as a serious D. hereditary
finding for it is a manifestation of
transient HPV.  Rationale: Estrogen DEFICIENCY has
been well established as a cause of
759. A 25 yo Gravida 1 Para 1, Pregnant Uterine bone loss - Compre gyne page 283
6 weeks consulted at your clinic. She underwent
cryotherapy for cervical intraepithelial lesion 762. The tx for early dysgerminoma is primarily
several years ago. What is your expected surgical, for women who wants to preserve repro
complication? organ the minimal surf procedure offered would
be?
a. Premature Birth
b. Premature Rupture of Membrane a. Unilateral oophorectomy
c. Preeclampsia B. Unilateral oophorectocystectomy
d. All of the above C. Unilateral salphingooophorectomy
D. Unilateral salphingooophorecystectomy
 Rationale: Comprehensive gyne 6th ed
(chapter 23, p663) - “Short term  Rationale: The minimal surgical
complications from the procedure include operation for ovarian dysgerminoma is a
the nuisance of the discharge and unilateral oophorectomy- Novak 15ed
occasional bleeding. Long term page 2421 ebook
complications include cervical stenosis
and a small increase in preterm labor” 763. Mc malignant germcell tumor?

760. Most common cause of postmenopausal A. Dysgerminoma


bleeding B. Immature teratoma
A. Endometrial ca C. Mature teratoma
B. Endometrial hyperplasia D. Endodermal sinus tumor
C. Endometrial polyp
D. Atrophic endometrium  Ratioanle: of the malignant germ cell
E. Cervical ca tumors, the most frequent is the
dysgerminoma, which accounts for
 Rationale: Comprehensive gyne 6th ed approximately 45% of malignant germ
(chapter 37 p806) cell tumors. Next in frequency are
immature teratomas

“Never design your LIFE like a GARDEN Where anyone can walk, rather Design your LIFE like the SKY Where everyone desires
to REACH”

Special Credits to: | Darna | Totoro | LittleSnoozy | Alma | RatioTeam | RecallsTeam | Astral | SKCCSLAS | Katz
Disclaimer: Some of the items and answers here are not mine but was verified so, If you find discrepancies please feel free to
correct, this serves only as a guide for easy and efficient reviewing.
God bless!
101 | P a g e Philippians 4:13 jpmrnmd
GYNECOLOGY – Mastery Review
“Master your Ultra Instinct”
Ultimate Reviewer ~800 items for Major Exams | Revalida | Board Exams
and then endodermal sinus tumors. C. embryonal- high HCG and AFP, no sx of
Dysgerminomas are the most common type of pseudopuberty
malignant germ D. mixed germ cell tumor- high AFP, HCG and
cell tumors. Page 762 and 764 Comprehensive LDH
gynecology 6th ed,
768. Ovulation can be induced 90-95% of
764. Dysgerminoma may be found in both sexes anovulation patieny except those elevated,
and may arise in both gonadal and extragonadal except:
sites which include
a.fsh
A. Midline from pineal gland b. LH
B. Mediastinum c. estrogen
C. Retroperitoneum d. progesterone
D. B & C
E. A, B, C 769. 1st line drug therapy for GERD

 Ratioanle: page 1506 Berek & Novak’s a) Oral antacids


Gynecology 14th ed b) Omeprazole
c) Pantoprazole
765. Most important prognostic feature of d) Esomeprazole
Immature Teratoma?
 Rationale: Page 1073 - Oral antacids are
 Grading first-line therapy. IF severe symptoms
persist, sucralfate is given with an H2-
 Rationale: (Gyne 6th Ed) the prognosis receptor antagonist such as Cimetidine
for patient with immature teratoma is or Ranitidine. IF these are not successful,
related to stage (FIGO) and grade of the commonly used Proton-pump inhibitors
tumor. The grade of tumor s based on such as Omeprazole or Pantoprazole are
the degree of Immaturity of various also safe for use in pregnancy. IF there is
tissues. still no relief, then endoscopy should be
considered. Misoprostol is
766. True regarding endometrial sinus tumor of contraindicated because it stimulates
ovary? labor.

Most endometrial sinus tumor secretes AFP 770. Which of the ff. increase risk for
monozygotic?
 Rationale: Gyne 6th ed The tumor a) Maternal age
secretes AFP, which s a specific marker b) Race
for identifying and following these c) Hereditary
tumors clinically. d) AOTA
e) NOTA
767. This lesion is seen in premenarchal girls
with pseudopuberty and with elevated AFP and  Rationale: Page 892 - Dizygotic twinning
HCG? is much more common than monozygous
splitting of a single oocyte, and its
A. choriocarcinoma- secrete HCG with isosexual incidence is influenced by race, heredity,
precocity maternal age, parity, and especially,
B. polyembryonal fertility treatment.

“Never design your LIFE like a GARDEN Where anyone can walk, rather Design your LIFE like the SKY Where everyone desires
to REACH”

Special Credits to: | Darna | Totoro | LittleSnoozy | Alma | RatioTeam | RecallsTeam | Astral | SKCCSLAS | Katz
Disclaimer: Some of the items and answers here are not mine but was verified so, If you find discrepancies please feel free to
correct, this serves only as a guide for easy and efficient reviewing.
God bless!
102 | P a g e Philippians 4:13 jpmrnmd
GYNECOLOGY – Mastery Review
“Master your Ultra Instinct”
Ultimate Reviewer ~800 items for Major Exams | Revalida | Board Exams
 By contrast, the frequency of d. colpopexy - an abdominal sacrocolpopexy is
monozygotic twin births is relatively usually necessary for resuspension and closure
constant worldwide – approximately one of the enterocele defect.
set per 250 births, and this incidence is
generally independent of race, heredity, 775. Next event following rapid rate of growth
age, and parity. A. Menarche
B.
771. Prolactin is liberated in a well decidualized C. Growth of pubic hair
endometrial stroma D. Growth of axillary hair
TRUE
FALSE  Rationale: Growth in height is most rapid
during several years prior to menarche,
772. If a woman presents with profuse yellow with onset of the growth spurt occurring
green discharge, the first etiologic agent which is between the ages of 10 - 14 years. Peak
considered should be height velocity occurs about one to one
A. Chlamydia and a half years before menarche.
B. Trichomonas vaginalis Additional height is gained after peak
C. Candida albicans height velocity (10.8 - 22.3 cm) and after
D. Neisseria gonorrhea menarche (7.4 - 10.6 cm). This additional
height may be gained over a period of 4.7
Chlamydia clear thick discharge + pain years after menarche (Tanner, 1972).
during urination
Trichomonas profuse, “frothy”, malodorous 776. A 25yo woman came in for her first check up
white gray/yellow/green complaining of excessive hair growth on her
(Compre, p536), strawberry upper lip and outer cheeks. Blood assay
cervix, clue cells (Berek) revealed:
Candida white-gray, pruritus (Compre, Testosterone: 0.05 mg
p538) DHEAS: 24 mg DHEA: 0.06 mg
N. milky discharge, burning, Androstenedione: 4 mg.
gonorrhoea itching The increase hair growth may be attributed to
increase production of androgen from:
773. Vaginal laceration from vaginal barrel up to
introitus? A. Ovary
B. Adrenals
A. 1st stage - proplase into the upper barrel of C. Peripheral compartment
the vagina D. A & B
B. 2nd stage E. All are correct
C. 3rd stage - cervix and uterus prolapses out
through introitus  Rationale: the ovaries secrete only
D. 4th stage - complete eversion of the uterus approximately 0.1 mg of
and cervix - procidentia or vaginal apex testosterone/day, mainly from the thecal
and stroma cells. Other androgens
774. Methods for pelvic floor strengthening secreted by the ovary are
except: androstenedione (1 to 2 mg/day) and
DHEA (<1 mg/day). The adrenal glands,
a. Kegels in addition to secreting large quantities of
b. vaginal cones DHEAS (6 to 24 mg/day), secrete
c. Electrical approximately the same daily amount of

“Never design your LIFE like a GARDEN Where anyone can walk, rather Design your LIFE like the SKY Where everyone desires
to REACH”

Special Credits to: | Darna | Totoro | LittleSnoozy | Alma | RatioTeam | RecallsTeam | Astral | SKCCSLAS | Katz
Disclaimer: Some of the items and answers here are not mine but was verified so, If you find discrepancies please feel free to
correct, this serves only as a guide for easy and efficient reviewing.
God bless!
103 | P a g e Philippians 4:13 jpmrnmd
GYNECOLOGY – Mastery Review
“Master your Ultra Instinct”
Ultimate Reviewer ~800 items for Major Exams | Revalida | Board Exams
androstenedione (1 mg/day) as the 780. Vaginal cancer is common among women
ovaries and less than 1 mg of DHEA/day. who have expose to diethylstilbestrol in utero?
The normal adrenal gland secretes little
testosterone, although some uncommon A. Sarcoma botyroids
adrenal tumors may secrete testosterone B. Yolk sac tumor
directly. Androstenedione and DHEA do C. DES tumor
not have strong androgenic activity but D. Clear cell tumor
are peripherally converted at a slow rate
to the biologically active androgen,
testosterone. Only approximately 5% of 781. The term complex hyperplasia is
androstenedione and a smaller
percentage of DHEA are converted to  Adenomatous hyperplasia - Novaks
testosterone. Williams, 24th.
782. The estrogen that makes up the largest
777. G1P0, 7 weeks AOG, palpitation, easy amount in posmenopausal women is?
fatigability. Thyroid studies reveal thyroid
peroxidase antibodies. Treatment prevents this  Estrone - Interwebs
complication:
783. In the 2009 FIGO surgical staging a patient
A. Preterm birth with endometrial carcinoma involving the
B. Hyperparathyroidism cervical stroma is stage as
C. Tetanic seizures
D. Placenta previa  Ia - Berek and Novaks

778. Cushing
a. Tachycardia 784. A 52 year old G1P1 patient came to your
b. Bilateral ovarian enlargement clinic for post menopausal bleeding. On pelvic
c. Central deposition of fat exam her uterus was slightly enlarged.
Transvaginal Ultrasound revealed a thickened
789. Dx aid to differentiate LOHD from PCOS endometrial lining of 1-2 cm. What will be your
IS? next step in the management of this case?

A. Ct scan  Endometrial Biopsy - Berek and


B. Serum 17-hydroxyprogesterone Novaks
C. Transvaginal ultrasound
D. Dexamethasone supression test (Cushing
Syndrome) 785. Which is the most common histologic
subtype in endometrial carcinoma?
 Rationale: To differentiate LOHD from
PCOS, measurement of basal (early  Adenocarcinoma - Berek and Novaks
morning) serum 17-
hydroxyprogesterone levels should be 786. Which histologic subtype of endometrial
performed. If basal levels are greater carcinoma has the poorest prognosis?
than 8 ng/mL, the diagnosis of LOHD is
established.  Sqamous Cell Carcinoma

“Never design your LIFE like a GARDEN Where anyone can walk, rather Design your LIFE like the SKY Where everyone desires
to REACH”

Special Credits to: | Darna | Totoro | LittleSnoozy | Alma | RatioTeam | RecallsTeam | Astral | SKCCSLAS | Katz
Disclaimer: Some of the items and answers here are not mine but was verified so, If you find discrepancies please feel free to
correct, this serves only as a guide for easy and efficient reviewing.
God bless!
104 | P a g e Philippians 4:13 jpmrnmd
GYNECOLOGY – Mastery Review
“Master your Ultra Instinct”
Ultimate Reviewer ~800 items for Major Exams | Revalida | Board Exams
787. The histologic criteria of diagnosing 799. Mucopurulent Gonorrhea
leiomyosarcoma is
discharge
A. more than 10 mitotic figures per 10
high power field
800. First Line Therapy for kegels exercise
788. The primary treatment for sarcoma of the overreactive bladder
uterus is?

 Surgery Berek and Novaks

789. Which of the following is a risk factor for the

|REMEMBER|
development of endometrial carcinoma?

 HPV infection

790. Bacteria Vaginosis Metronidazole 500mg


twice daily for 7 days
791. Vaginal Candidiasis Fluconazole 150mg single
dose Philippians 4:13
792. Trichomoniasis Metronidazole 2gm Single
dose “I can do all this through him who
793. Gonorrhea Azithromycin 1gm single
dose gives me strength”
794. Chlamydia Azithromycin 1gm single
dose

795. Clue Cells Bacterial Vaginosis

796. Hyphae and Active Candidiasis


Pseudohypae

797. Cottage Cheese-like Vaginal Candidiasis


discharge

798. Strawberry cervix Trichomoniasis

“Never design your LIFE like a GARDEN Where anyone can walk, rather Design your LIFE like the SKY Where everyone desires
to REACH”

Special Credits to: | Darna | Totoro | LittleSnoozy | Alma | RatioTeam | RecallsTeam | Astral | SKCCSLAS | Katz
Disclaimer: Some of the items and answers here are not mine but was verified so, If you find discrepancies please feel free to
correct, this serves only as a guide for easy and efficient reviewing.
God bless!

S-ar putea să vă placă și